Anaesthesia: Single Best Answer Mcqs in

Download as pdf or txt
Download as pdf or txt
You are on page 1of 212

SBA cover.

qxd 02/07/2010 15:30 Page 1

Single Best Answer MCQs in Anaesthesia


This book comprises six sets of single best answer practice papers. Each set

contains 30 single best answer questions on clinical anaesthesia, pain and

Single Best Answer MCQs in


intensive care. The scenarios are based on common peri-operative problems

encountered during anaesthetic practice and intensive care medicine. The

best possible answer to a given clinical scenario is substantiated by detailed

explanation drawn from recent review articles and textbooks in clinical

ANAESTHESIA
anaesthesia. These questions enable the candidates to assess their knowledge

and problem-solving ability.

This book is an ideal companion for candidates sitting postgraduate

examinations in anaesthesia and intensive care medicine. It will also be a

valuable educational resource for all trainees and practising anaesthetists.

Volume I Clinical Anaesthesia

ISBN 978-1-903378-75-5

9 781903 378755
tf m Cyprian Mendonca, Mahesh Chaudhari, Josephine James
Prelims MCQ book_Prelims MCQ book.qxd 22-04-2013 19:17 Page i

Single Best Answer MCQs in

ANAESTHESIA

Volume I Clinical Anaesthesia

Cyprian Mendonca, Mahesh Chaudhari, Josephine James


Prelims MCQ book_Prelims MCQ book.qxd 22-04-2013 19:17 Page ii

Single Best Answer MCQs in Anaesthesia

tfm Publishing Limited, Castle Hill Barns, Harley, Nr Shrewsbury, SY5


6LX, UK. Tel: +44 (0)1952 510061; Fax: +44 (0)1952 510192
E-mail: nikki@tfmpublishing.com; Web site: www.tfmpublishing.com

Design & Typesetting: Nikki Bramhill BSc Hons Dip Law


First Edition: © September 2010
Background cover image © Comstock Inc., www.comstock.com
Paperback ISBN: 978-1-903378-75-5

E-book editions: 2013


ePub ISBN: 978-1-908986-63-4
Mobi ISBN: 978-1-908986-64-1

ii
Web pdf ISBN: 978-1-908986-65-8

The entire contents of ‘Single Best Answer MCQs in Anaesthesia’ is


copyright tfm Publishing Ltd. Apart from any fair dealing for the purposes
of research or private study, or criticism or review, as permitted under the
Copyright, Designs and Patents Act 1988, this publication may not be
reproduced, stored in a retrieval system or transmitted in any form or by
any means, electronic, digital, mechanical, photocopying, recording or
otherwise, without the prior written permission of the publisher.

Neither the authors nor the publisher can accept responsibility for any
injury or damage to persons or property occasioned through the
implementation of any ideas or use of any product described herein.
Neither can they accept any responsibility for errors, omissions or
misrepresentations, howsoever caused.

Whilst every care is taken by the authors and the publisher to ensure that
all information and data in this book are as accurate as possible at the time
of going to press, it is recommended that readers seek independent
verification of advice on drug or other product usage, surgical techniques
and clinical processes prior to their use.

The authors and publisher gratefully acknowledge the permission granted


to reproduce the copyright material where applicable in this book. Every
effort has been made to trace copyright holders and to obtain their
permission for the use of copyright material. The publisher apologizes for
any errors or omissions and would be grateful if notified of any corrections
that should be incorporated in future reprints or editions of this book.

Printed by Gutenberg Press Ltd., Gudja Road, Tarxien, PLA 19, Malta.
Tel: +356 21897037; Fax: +356 21800069.
Prelims MCQ book_Prelims MCQ book.qxd 22-04-2013 19:17 Page iii

Contents

Page
Preface iv iii
Acknowledgements vi
Abbreviations viii

Set 1 Questions
Set 1
1
Answers 15

Set 2 Questions
Set 2
33
Answers 45

Set 3 Questions
Set 3
65
Answers 77

Set 4 Questions
Set 4
95
Answers 107

Set 5 Questions
Set 5
129
Answers 143

Set 6 Questions
Set 6
165
Answers 179
Prelims MCQ book_Prelims MCQ book.qxd 22-04-2013 19:17 Page iv

Preface

Single best answer type multiple choice questions are being introduced

iv
into anaesthetic postgraduate examinations. They are considered to be a
valuable way of assessing the trainee’s ability to apply knowledge to
clinical practice.

This book consists of six sets of single best answer practice papers.
Each set comprises 30 multiple choice questions drawn from clinical
anaesthesia, pain and intensive care medicine. Each question consists of
a stem describing a clinical scenario or problem followed by five possible
answer options. One of them is the best response for the given question.
Each question and answer is accompanied by supporting notes obtained
from peer-reviewed journal articles and anaesthesia textbooks.

The main objective of this book is to provide trainees with a series of


single best answer type questions that will prepare them for this format of
postgraduate examinations. Much emphasis has been placed on the
application of knowledge to solve common peri-operative problems
encountered during anaesthetic practice.

We hope that a thorough revision of this book will enable trainees to


improve their ability to apply knowledge to clinical practice. We believe this
book will not only be an invaluable educational resource for those who are
Prelims MCQ book_Prelims MCQ book.qxd 22-04-2013 19:17 Page v

preparing for postgraduate examinations, but will also be of benefit to any


practising anaesthetist.

Cyprian Mendonca MD, FRCA


Consultant Anaesthetist
University Hospitals Coventry and Warwickshire
Coventry, UK

Mahesh Chaudhari MD, FRCA, FFPMRCA


Consultant Anaesthetist
v
Worcestershire Royal Hospital
Worcester, UK

Josephine James FRCA


Consultant Anaesthetist
Heart of England Foundation Trust
Birmingham, UK
Prelims MCQ book_Prelims MCQ book.qxd 22-04-2013 19:17 Page vi

Acknowledgements

We are grateful to Nikki Bramhill, Director, tfm publishing, for critically

vi
reviewing the manuscript. We extend our thanks to the following who
contributed questions to this book:

Dr Thejas Bhari
Specialty Registrar, Warwickshire School of Anaesthesia

Dr Thomas Billyard
Specialty Registrar, Warwickshire School of Anaesthesia

Dr Narotham Burri
Specialty Registrar, Warwickshire School of Anaesthesia

Dr Shefali Chaudhari
Specialty Registrar, Warwickshire School of Anaesthesia

Dr Adrian Jennings
Specialty Registrar, Birmingham School of Anaesthesia

Dr Payal Kajekar
Specialty Registrar, Warwickshire School of Anaesthesia

Dr Seema Quasim
Consultant Anaesthetist, University Hospital, Coventry

Dr Mohan Ranganathan
Consultant Anaesthetist, George Eliot Hospital, Nuneaton
Prelims MCQ book_Prelims MCQ book.qxd 22-04-2013 19:17 Page vii

Dr Rajneesh Sachdeva
Specialty Registrar, Warwickshire School of Anaesthesia

Dr Rathinavel Shanmugam
Specialty Registrar, Warwickshire School of Anaesthesia

Dr Catherine Snaith
Specialty Registrar, Warwickshire School of Anaesthesia

Dr Joyce Yeung
Specialty Registrar, Warwickshire School of Anaesthesia

vii
Prelims MCQ book_Prelims MCQ book.qxd 22-04-2013 19:17 Page viii

Abbreviations

AAA
ACTH
Abdominal aortic aneurysm

viii ADH
Adrenocorticotrophic hormone

AF
Anti-diuretic hormone

AICD
Atrial fibrillation

ALS
Automatic implantable cardioverter defibrillator

ALT
Advanced life support

APACHE
Alanine transaminase

APTT
Acute Physiology and Chronic Health Evaluation

ARDS
Activated partial thromboplastin time

ASA
Acute respiratory distress syndrome

ASAS
American Society of Anesthesiologists

AST
Anterior spinal artery syndrome

BD
Aspartate transaminase

BE
Twice a day

BJR
Base excess

BP
Bezold-Jarisch reflex

CABG
Blood pressure

CAP
Coronary artery bypass grafting

CBT
Community-acquired pneumonia

CEA
Cognitive behavioural therapy

CK
Carotid endarterectomy

Cl
Creatine kinase

CNS
Chloride

CO
Central nervous system

COAD
Carbon monoxide

COHb
Chronic obstructive airway disease
Carboxy-haemoglobin
Prelims MCQ book_Prelims MCQ book.qxd 22-04-2013 19:17 Page ix

Abbreviations

COPD
CPAP
Chronic obstructive pulmonary disease

CPM
Continuous positive airway pressure

CPR
Central pontine myelinolysis

CPSP
Cardiopulmonary resuscitation

CRPS
Chronic post-surgical pain

CSE
Complex regional pain syndrome

CSF
Combined spinal epidural

CSWS
Cerebrospinal fluid

CT
Cerebral salt wasting syndrome

CVA
Computed tomography

CVP
Cerebrovascular accident

ix
DDAVP
Central venous pressure

DI
1-de-amino-8-D-arginine vasopressin

DIC
Diabetes insipidus

DKA
Disseminated intravascular coagulation

DLCO
Diabetic ketoacidosis

DPG
Diffusion lung capacity for carbon monoxide

DVT
2,3-diphosphoglycerate

ECG
Deep vein thrombosis

EEG
Electrocardiogram

EMG
Electro-encephalography

EMI
Electromyography

ESR
Electromagnetic interference

ETCO2
Erythrocyte sedimentation rate

FDP
End-tidal CO2

FEV
Fibrin degradation product

FFP
Forced expiratory volume

FRC
Fresh frozen plasma

FVC
Functional residual capacity

GA
Forced vital capacity

GABA
General anaesthesia

GCS
Gamma aminobutyric acid

GFR
Glasgow Coma Scale

GH
Glomerular filtration rate

GTN
Growth hormone

H
Glyceryl trinitrate
Hydrogen
Prelims MCQ book_Prelims MCQ book.qxd 22-04-2013 19:17 Page x

Single Best Answer MCQs in Anaesthesia

Hb
HCO3
Haemoglobin

Hct
Bicarbonates

HDU
Haematocrit

HELLP
High dependency unit

HIT
Haemolysis, elevated liver enzymes and low platelets

ICP
Heparin-induced thrombocytopaenia

ICU
Intracranial pressure

IDDS
Intensive care unit

INR
Implantable drug delivery system

IPH
International normalised ratio

ISS
Idiopathic pulmonary hypertension

x
IV
Injury Severity Score

JVP
Intravenous

K
Jugular venous pressure/pulse

LA
Potassium

LBP
Local anaesthesia

LMA
Low back pain

LMWH
Laryngeal mask airway

MAOI
Low-molecular-weight heparin

MEP
Monoamine oxidase inhibitor

MH
Motor evoked potentials

MND
Malignant hyperthermia

MODS
Motor neurone disease

MRA
Multiple Organ Dysfunction Score

MST
Magnetic resonance angiography

Na
Morphine sulphate

NG
Sodium

NIBP
Nasogastric

NICE
Non-invasive blood pressure

NIDDM
National Institute for Health and Clinical Excellence

NIV
Non-insulin dependent diabetes mellitus

NMDA
Non-invasive ventilation

NPPE
N-methyl-D-aspartate

NSAID
Negative pressure pulmonary oedema

OD
Non-steroidal anti-inflammatory drug

OSF
Once a day
Organ system failure
Prelims MCQ book_Prelims MCQ book.qxd 22-04-2013 19:17 Page xi

Abbreviations

PCA
PCWP
Patient-controlled analgesia

PE
Pulmonary capillary wedge pressure

PEEP
Pulmonary embolism

PEFR
Positive end expiratory pressure

PHN
Peak expiratory flow rate

POD
Post-herpetic neuralgia

PONV
Postoperative delirium

PPH
Postoperative nausea and vomiting

PT
Postpartum haemorrhage

PTH
Prothrombin time

RBC
Parathyroid hormone

xi
RV
Red blood cell

SAH
Residual volume

SAPS
Subarachnoid haemorrhage

SCD
Simplified Acute Physiology Score

SCS
Sickle cell disease

SIADH
Spinal cord stimulator

SLE
Syndrome of inappropriate anti-diuretic hormone secretion

SOFA
Systemic lupus erythematosus

SSEP
Sequential organ failure assessment

SVC
Somatosensory evoked potentials

SVR
Superior vena cava

TDS
Systemic vascular resistance

TENS
Three times a day

TFPI
Transcutaneous electrical nerve stimulation

THAM
Tissue factor prothrombin inhibitor

TIVA
Tri-hydroxymethyl aminomethane

TLC
Total intravenous anaesthesia

TLCO
Total lung capacity

TRALI
Transfer factor for carbon monoxide

TRH
Transfusion-related acute lung injury

TSH
Thyrotropin releasing hormone

UFH
Thyroid stimulating hormone

VA
Unfractionated heparin

VAE
Alveolar volume

VAS
Venous air embolism
Visual Analogue Score
Prelims MCQ book_Prelims MCQ book.qxd 22-04-2013 19:17 Page xii

Single Best Answer MCQs in Anaesthesia

VF
VIP
Ventricular fibrillation

VTE
Vasoactive intestinal peptide

WBC
Venous thrombo-embolism

WCC
White blood cell
White cell count

xii
set 1_set 1.qxd 22-04-2013 19:18 Page 1

Set 1 questions

Set 1
1 A 60-year-old male patient is admitted to the emergency department.
He was working on his car in a garage and was found unconscious 1
by his wife, with the garage door almost shut and the car engine
running. On assessment, his GCS is 7, oxygen saturation is 99%
and mucous membranes are ‘cherry red’ in colour. Which of the
following actions is most appropriate in the immediate
management?

a. Take a full history from his wife to confirm the facts.


b. Arrange for a CT of the brain to precisely diagnose the cause of
unconsciousness.
c. Arrange for urgent transfer to a neuro-intensive care unit.
d. Intubate and ventilate the patient with 100% oxygen.
e. Oxygenate with a non-rebreathing mask whilst arterial blood gas
results are performed.

2 A 59-year-old male has undergone a left upper lobectomy for a


neoplasm. The peri-operative course was uncomplicated. He has a
history of COPD and 50-pack-per-year history of smoking. His
routine medications include a salbutamol inhaler and uniphyllin
300mg b.d. On the second postoperative day, in the high
dependency unit, the patient suddenly develops atrial fibrillation with
a ventricular rate of 140 to 170 beats/min. His blood pressure falls
set 1_set 1.qxd 22-04-2013 19:18 Page 2

Single Best Answer MCQs in Anaesthesia

to 80/30mmHg from a near normal level for his age. The most
appropriate immediate management is:

a. Check the serum K+ and Mg+ levels.


b. Commence oxygen 6L/min using a non-rebreathing mask.
c. Give amiodarone 5mg/kg boluses over 30 minutes.
d. Synchronised DC shock.
e. Intravenous digoxin.

3 A 21-year-old male has had an extraction of an impacted third molar


tooth under general anaesthetic. Postoperatively he continues to
2 bleed from the surgical site. Despite surgical exploration and
packing, bleeding continues. His coagulation screen reveals both a
prolonged bleeding time and activated partial thromboplastin time
(APTT), but the prothrombin time (PT) and platelet count are within
normal limits. The patient mentions that his father bruises quite
easily. The most likely diagnosis is:

a. Haemophilia A.
b. Haemolytic uraemic syndrome.
c. Haemophilia B.
d. von Willebrand’s disease.
e. Laden V deficiency.

4 A 45-year-old man with a history of gall stones presents to the


emergency department complaining of severe constant epigastric
pain radiating to the back and flanks, and vomiting. Examination
reveals pyrexia, abdominal distension, rebound tenderness and
discolouration of the flanks. Which of the following blood tests
would be most useful in the diagnosis of acute pancreatitis?

a. Serum amylase.
b. Serum trypsinogen.
c. Serum lipase.
d. Serum transaminases.
e. Serum calcium.
set 1_set 1.qxd 22-04-2013 19:18 Page 3

Set 1 questions

5 A 39-year-old male is due to undergo haemorrhoidectomy. He


suffered a complete spinal cord transection at T6 level 2 years ago.
His medications include paracetamol q.d.s., gabapentin 600mg
t.d.s., omeprazole and clonazepam. He intermittently uses a GTN
spray for the management of symptoms related to autonomic hyper-
reflexia. The patient has no particular preference regarding
anaesthetic technique. Which anaesthetic technique would be most
suitable?

a. General anaesthesia with rapid sequence induction.

3
b. Spinal anaesthesia.
c. No need for any anaesthesia.
d. Light general anaethesia.
e. Combined spinal epidural (CSE).

6 An 83-year-old female presents to the pain clinic with a 10-week


history of severe pain in her left eye. The pain is continuous and is
associated with a burning sensation. She also had a skin rash in the
painful area which began after a week of onset of the pain. She has
been treated with an intermittent course of steroid medication for the
management of her poorly controlled COAD. The most likely cause
of her pain is:

a. Trigeminal neuralgia.
b. Atypical facial pain.
c. Atypical presentation of trigeminal neuralgia.
d. Late signs and symptoms of polymyalgia rheumatica.
e. Post-herpetic neuralgia.

7 A 33-year-old female is due to undergo an emergency laparotomy for


a ruptured ectopic pregnancy. She has had a previous general
anaesthesic when she was told she might suffer from a possible
allergy to an anaesthetic agent. She suffers from hay fever and has
a history of allergic reactions to multiple medications including
set 1_set 1.qxd 22-04-2013 19:18 Page 4

Single Best Answer MCQs in Anaesthesia

antibiotics and NSAIDs. Her previous anaesthetic notes are not


available. Which one of the following is most appropriate for
intravenous induction of anaesthesia?

a. Thiopentone.
b. Propofol.
c. Etomidate.
d. Ketamine.
e. Methohexitone.

4
8 A 60-year-old male is undergoing elective posterior fossa surgery in
the sitting position. Forty minutes into the operation, he develops
bronchospasm and his blood pressure drops suddenly from
110/70mmHg to 70/40 mmHg. In the previous 20 minutes the
patient had not received any drugs. What is the most likely cause of
the sudden fall in BP in this patient?

a. Myocardial infarction.
b. High concentration of volatile agents.
c. Profuse bleeding.
d. Air embolism.
e. Anaphylaxis.

9 You are called to assess a 25-year-old female with a history of acute


bilateral symmetrical descending paralysis. She has no changes in
her mental status or sensory deficit, and is afebrile. Which of the
following investigations will best help you to arrive at the diagnosis?

a. Analysis of blood or stool for botulinum toxin.


b. Blood culture for viruses.
c. ‘Tensilon’ test.
d. CSF examination.
e. Nerve conduction studies.
set 1_set 1.qxd 22-04-2013 19:18 Page 5

Set 1 questions

10 A 61-year-old female is ready to be discharged on the fifth


postoperative day following an uneventful total hip replacement. A
nurse has noticed localised necrosis of skin at the sites of
subcutaneous injection of enoxaparin on the abdominal wall. She
also mentions that there has been a fall in platelet count from 216 x
109/L to 64 x 109/L over the last 5 days. What would be your next
action?

a. Reduce the dose of enoxaparin by half and administer two units of


FFP.

5
b. Stop enoxaparin.
c. Stop enoxaparin until the platelet count starts rising.
d. Stop enoxaparin and start an alternative anticoagulant.
e. Stop enoxaparin and transfuse two units of adult platelets.

11 A 27-year-old woman has been diagnosed with untreatable


carcinoma of the cervix and has severe pain. She is taking MST
400mg b.d., Oramorph 20mg four-hourly, gabapentin 600mg t.d.s.
and clonazepam. The doses of opioid medication required to
alleviate the pain have doubled in the last 2 months and the MST
was changed to oxycodone and the dose increased, but with little
effect on the degree of pain. Following palliative surgery she is now
incontinent. Her life expectancy is about 3 months. The next best
possible intervention for the management of her uncontrollable pain
in the perineal area would be:

a. Radiofrequency lesioning of the lumbar sympathetic nerves.


b. Coeliac plexus block using a neurolytic solution.
c. Intrathecal saddle neurolytic block.
d. Intrathecal drug delivery of morphine.
e. Lumbar chemical sympathectomy using a neurolytic solution.

12 A 41-year-old primigravida has been admitted at 38 weeks of


gestation with headache, nausea and a blood pressure of
set 1_set 1.qxd 22-04-2013 19:18 Page 6

Single Best Answer MCQs in Anaesthesia

196/116mmHg. A Caesarean section is planned, and oral labetalol


400mg has been administered. The next BP recorded an hour later
is 176/110 mmHg. The next step in the control of this woman’s pre-
eclampsia should be:

a. Intravenous labetalol infusion.


b. Intravenous magnesium sulphate infusion, following a loading dose.
c. Intravenous magnesium sulphate infusion, without a loading dose.
d. Sublingual nifedipine tablet.
e. Epidural analgesia.

6
13 A 29-year-old woman (gravida 2, para 1) has had a ventouse vaginal
delivery of a baby boy weighing 4.2kg. Intramuscular syntometrine
has been administered by the midwife. Thirty minutes after delivery
of the placenta, she suffers a primary postpartum haemorrhage
(PPH) of about 400ml. The most common cause of PPH in this
scenario is:

a. An atonic uterus.
b. Coagulopathy.
c. Retained placental tissue.
d. A vaginal tear.
e. A perineal tear.

14 A 29-year-old male is admitted to the emergency department


following a road traffic accident. Since admission his GCS has been
gradually deteriorating and is now 11. Which of the following would
warrant an immediate craniotomy in this patient?

a. Status epilepticus.
b. Unilateral pupillary dilatation.
c. Severe headache with neck rigidity.
d. Cerebrospinal fluid rhinorrhoea.
e. Significant hypotension.
set 1_set 1.qxd 02-05-2013 19:16 Page 7

Set 1 questions

15 A woman in preterm labour requires transfer to a hospital with an


available neonatal cot as the baby is at risk. Which of the following
drugs would be most suitable for tocolysis in this situation?

a. Ritodrine.
b. Glyceryl trinitrate (GTN).
c. Atosiban.
d. Magnesium sulphate.
e. Salbutamol nebuliser.

16 A 32-year-old male who is a known heroin addict, is on the trauma


list for open reduction and internal fixation of a fracture of his 7
humerus. He has been abusing heroin for the last 3 years. He is very
anxious about postoperative pain relief and the surgeon is
concerned about compartment syndrome in the postoperative
period. Which of the following is the best choice for postoperative
pain relief?

a. Multimodal analgesia with ‘PRN’ morphine.


b. Multimodal analgesia with an additional infraclavicular brachial
plexus block using 20ml of 0.25% bupivacaine.
c. Multimodal analgesia with a morphine PCA.
d. Multimodal analgesia with an intravenous infusion of morphine.
e. Fentanyl patch with an initial dose of 50µg/hr.

17 A woman without past psychiatric history develops severe postnatal


depression and commits suicide 30 days after giving birth.
According to the Centre for Maternal and Child Enquiries (formerly
CEMACH), this death is best classified as:

a. Late, direct maternal death.


b. Late, indirect maternal death.
c. Indirect maternal death.
d. Direct maternal death.
e. Coincidental maternal death.
set 1_set 1.qxd 02-05-2013 19:16 Page 8

Single Best Answer MCQs in Anaesthesia

18 A 26-year-old male patient is admitted to the intensive care unit


following a severe head injury. On admission the baseline blood
results show: K+ 3.4mmol/L, Na+ 136mmol/L and Cl- 112mmol/L.
He has been intubated, sedated and hyperventilated for the last
hour. His arterial blood gas reveals a PaCO2 of 3.3kPa. His ECG
morphology on the monitor has now changed with ST segment
depression, T-wave flattening and occasional premature ventricular
contractions. The most likely cause for the ECG changes is:

a. Increased intracranial pressure.


b. Hypokalaemia.
8
c. Myocardial ischaemia.
d. Hyperkalaemia.
e. Hyponatraemia.

19 You have inserted a central venous catheter via the right internal
jugular vein in a 40-year-old male patient about to undergo a
laparotomy. The best method to confirm the correct placement of
this central venous catheter would be:

a. Measurement of pH of the blood sample drawn from the catheter.


b. Measurement of PaCO2.
c. Measurement of pressure in the catheter using a pressure
transducer.
d. Chest X-ray.
e. Aspiration of dark red blood from all the lumens of the catheter.

20 A 45-year-old woman complains of gradual increasing numbness


and paraesthesia in the thumb, index and middle fingers, more
severe at night. Clinical examination reveals wasting of the muscles
of the thenar eminence. These clinical features suggest
compression of the following structure in the wrist:

a. Median nerve.
b. Ulnar nerve.
c. Superficial radial nerve.
set 1_set 1.qxd 02-05-2013 19:16 Page 9

Set 1 questions

d. Ulnar artery.
e. Radial artery.

21 A 49-year-old male presents with a history of pain along the lower


jaw on the left side. The pain is paroxysmal, shooting, and very
intense and lasts for a few seconds to minutes. In between the
episodes, he has no residual abnormal sensation. The pain is
brought on by brushing his teeth, shaving or at times, by touch. In
the first instance, what would be the most appropriate action in his
medical management?

9
a. Arrange for an urgent MRI brain scan.
b. Commence treatment with carbamazepine.
c. Refer the patient to a neurosurgeon.
d. Prescribe fentanyl lozenges.
e. Refer the patient to a psychologist.

22 A 72-year-old male patient underwent elective decompression of the


lumbar spine. He had no previous experience of general
anaesthesia. Tracheal intubation was difficult due to a grade 3 view
of the larynx. A gum elastic bougie was successfully placed in the
trachea at the third attempt at direct laryngoscopy and a size 8.5mm
endotracheal tube was railroaded over the bougie with some
difficulty. In the recovery room the patient was noted to have
inspiratory stridor with an oxygen saturation of 92%. The oxygen
saturation rose to 97% following administration of nebulised
adrenaline. Which of the following is the most likely cause of his
symptoms?

a. Airway oedema.
b. Vocal cord paralysis.
c. Arytenoid subluxation.
d. Laryngospasm.
e. Arytenoid dislocation.
set 1_set 1.qxd 02-05-2013 19:16 Page 10

Single Best Answer MCQs in Anaesthesia

23 A 64-year-old male was listed for a lumbar laminectomy in the prone


position. Following pre-oxygenation, general anaesthesia was
induced using propofol and atracurium by a trainee anaesthetist. The
trainee anaesthetist encountered a difficult intubation due to a grade
4 view of the larynx and the airway was secured using a laryngeal
mask airway (LMA). Help was summoned from a consultant
anaesthetist. The most suitable method of performing tracheal
intubation by the second anaesthetist would be:

a. Waking up the patient and performing an awake fibreoptic


intubation.
10
b. Performing a fibreoptic-assisted intubation through the LMA.
c. Removing the LMA and attempting direct laryngoscopy.
d. Removing the LMA, inserting an intubating LMA and attempting
tracheal intubation.
e. Replacing the LMA with a ‘Proseal’ LMA to facilitate positive
pressure ventilation.

24 A 68-year-old male patient is admitted to the intensive care unit


(ICU) with lethargy and shortness of breath. His past medical history
includes hypertension, non-insulin-dependent diabetes, ischaemic
heart disease, and impaired renal function. His current medication
includes metformin and gliclazide. Soon after admission to the ICU,
he is sedated, intubated and ventilated. The subsequent blood gas
analysis reveals a pH of 7.08 and a lactate of 18mmol/L. The most
appropriate measure to correct the acidosis includes:

a. Intravenous sodium bicarbonate.


b. Haemodialysis.
c. Intravenous insulin.
d. Hyperventilation.
e. Tri-hydroxymethyl aminomethane (THAM).

25 A 68-year-old male patient is admitted to the intensive care unit with


lethargy and intermittent disorientation. His past medical history
includes depression and hypertension. On admission the baseline
electrolytes show a low sodium (105mmol/L) and low potassium
set 1_set 1.qxd 02-05-2013 19:16 Page 11

Set 1 questions

(3.1mmol/L). During the first 24 hours, 3L of 0.9% sodium chloride


with 20mmol/L of potassium chloride are administered, and
subsequently, enteral nutrition is started. Five days later the patient
becomes increasingly drowsy despite a normal serum sodium level.
Which of the following investigations would be most useful in
establishing the diagnosis?

a. Serum potassium level.


b. CSF proteins.
c. Electroencephalogram.
d. Magnetic resonance imaging of the brain.
e. Brain stem evoked potentials.

26 A 67-year-old patient has had a total knee replacement. He is on


11

morphine PCA for the management of postoperative pain. He has


received a total of 40mg morphine in the recovery area and you are
worried that he may develop an opioid overdose. Which of the
following is the earliest sign of opioid overdose?

a. Respiratory rate less than 8 per minute.


b. A fall in oxygen saturation.
c. Rapid shallow breathing.
d. Progressive rise in sedation level.
e. Uncontrolled vomiting.

27 A 35-year-old male patient has been admitted to ITU with Guillain


Barré syndrome. You have inserted a fine-bore nasogastric (NG)
tube for enteral feeding. Prior to commencing feeding, which of the
following is the best test to confirm the correct placement of a
nasogastric tube?

a. Injection of 50ml of air with auscultation over the stomach (‘whoosh’


test).
b. Chest X-ray.
c. Aspiration of at least 10ml through the NG tube.
d. Checking the pH of the aspirate.
e. Abdominal X-ray.
set 1_set 1.qxd 02-05-2013 19:16 Page 12

Single Best Answer MCQs in Anaesthesia

28 A 30-year-old, obese, primiparous woman has requested epidural


analgesia for labour pain. After obtaining consent the epidural space
was located at a depth of 7cm with loss of resistance to saline. A
catheter was inserted successfully into the epidural space through a
Tuohy needle despite initial slight resistance. On removal of the
needle it was noted that the catheter had broken at the 6cm mark
and the missing segment was not seen either on the drapes or on
the floor. The most important step in the further management of the
broken epidural catheter is:

12
a. CT scan of the lumbar region.
b. Plain X-ray of the lumbar region.
c. Neurosurgical referral.
d. Surgical exploration of the back for the missing segment of catheter.
e. Perform an epidural at a different space.

29 A 12-year-old girl is undergoing scoliosis correction. Anaesthesia is


maintained with isoflurane in nitrous oxide and oxygen. A total of
10mg morphine has been administered as intermittent boluses.
About 30 minutes into the procedure, the patient develops a
tachycardia which is not responsive to a bolus of intravenous fluids
or intravenous morphine. The EtCO2 is 7.2kPa despite adequate
ventilation and the temperature is recorded as 39°C. The first step
in the immediate treatment should be:

a. Dantrolene sodium 1mg/kg as an initial bolus.


b. Dantrolene sodium 2-3mg/kg as an initial bolus.
c. Send urine sample for myoglobin.
d. Measurement of arterial blood pH.
e. Insertion of central venous line.

30 A 65-year-old male patient is undergoing laser excision of a laryngeal


papilloma. The airway is secured with a ‘Laser-flex’ endotracheal
tube. During the procedure the proximal cuff is burst by a laser beam
set 1_set 1.qxd 02-05-2013 19:16 Page 13

Set 1 questions

and a small flame of fire appears in the surgical field. The most
appropriate immediate measure should be:

a. Increasing the inspired oxygen concentration.


b. Continuing with laser resection to complete the procedure as soon
as possible.
c. Flooding the field with normal saline.
d. Increasing the nitrous oxide concentration in order to reduce the
inspired oxygen concentration.
e. Changing the endotracheal tube immediately.

13
set 1_set 1.qxd 02-05-2013 19:16 Page 14

Single Best Answer MCQs in Anaesthesia

14
set 1 answers_set 1 answers.qxd 22-04-2013 19:17 Page 15

Set 1

Set 1 answers
answers

1 Answer: D. Intubate and ventilate the patient


with 100% oxygen. 15

The history and examination findings indicate that the most likely cause of
unconsciousness in this patient is carbon monoxide (CO) poisoning.
Carboxy-haemoglobin (COHb) has a similar absorption spectrum to
oxyhaemoglobin and therefore oxygen saturation is falsely raised. Carbon
monoxide binds with haemoglobin about 250 times as avidly as oxygen
and this adversely affects the oxygen content of blood. The half-life of
COHb is 4 hours; it is reduced to an hour with 100% oxygen and to 20-
30 minutes with hyperbaric oxygen therapy. This patient is unconscious
which indicates severe CO poisoning. Airway protection and oxygenation
of tissue is an absolute priority and this will be best achieved by tracheal
intubation and ventilation with 100% oxygen.

Further reading
1. Piantadosi CA. Carbon monoxide poisoning. Undersea Hyperb Med
2004; 31: 167-77.

2 Answer: D. Synchronised DC shock.

Atrial fibrillation (AF) is a commonly encountered arrhythmia


postoperatively following cardiothoracic surgery. Lack of co-ordinated
atrial contraction results in impulses from different parts of the atria
reaching the AV node in rapid succession and only some of these are
transmitted. A fast ventricular rate results in inadequate ventricular filling
and reduced cardiac output. The management of AF depends on its
duration, any obvious correctable causes, evidence of haemodynamic
set 1 answers_set 1 answers.qxd 22-04-2013 19:17 Page 16

Single Best Answer MCQs in Anaesthesia

compromise and the coagulation status. Acute onset AF with


haemodynamic compromise is best treated by synchronised DC shock to
restore sinus rhythm. If the patient’s blood pressure is not affected,
correctable causes should be addressed, and amiodarone boluses given
(5mg/kg over 30 minutes), followed by an infusion (15mg/kg over 23
hours). Anticoagulation should be considered if AF has persisted for more
than 2-3 days, especially prior to cardioversion.

Further reading
1. Bajpai A, Rowland E. Atrial fibrillation. British Journal of Anaesthesia
CEACCP 2006; 6: 219-24.

3
16
Answer: D. von Willebrand’s disease.

von Willebrand’s disease is the most commonly inherited (autosomal


dominant) coagulation disorder. von Willebrand Factor is a protein
involved in platelet adhesion and transfer of coagulation of Factor VIII.
Abnormality of this factor leads to abnormal platelet adhesiveness leading
to epistaxis, bruising and haemarthrosis. The coagulation profile reveals a
prolonged bleeding time and APTT with normal PT and platelet count.
Antiplatelet drugs should be avoided and peri-operative use of fresh frozen
plasma, cryoprecipitate and desmopressin (which increases Factor VIII
and von Willebrand Factor) may be required.

Further reading
1. Geil JD. von Willebrand disease. (http://emedicine.medscape.com/
article/959825-overview).
2. von Willebrand’s disease. In: Anaesthesia and intensive care A-Z. An
encyclopaedia of principles and practice, 4th ed, Yentis SM, Hirsch
NP, Smith GB, Eds. Oxford, UK: Butterworth-Heinemann, 2009;
600.

4 Answer: C. Serum lipase.

Serum amylase is a non-specific test and is elevated in bowel perforation,


obstruction and ischaemia, diabetic ketoacidosis, and pneumonia or
set 1 answers_set 1 answers.qxd 22-04-2013 19:17 Page 17

Set 1 answers

neoplasms. Serum lipase levels are more sensitive and specific than
amylase in acute pancreatitis and are elevated for up to 14 days. Urinary
trypsinogen-2 ‘Dipstix’ testing is still being studied as a test for both
confirmation of diagnosis and as an indicator of severity.

Further reading
1. Young SP, Thompson JP. Severe acute pancreatitis. British Journal of
Anaesthesia CEACCP 2008; 8: 125-8.

5 Answer: B. Spinal anaesthesia.

Complete spinal cord transection leads to loss of all sensation below the 17
level of injury. These patients still, however, have very active local spinal
reflexes and this can lead to autonomic hyper-reflexia. The stimuli for
autonomic hyper-reflexia are usually perineal procedures such as urinary
catheterization. Autonomic hyper-reflexia may present as a severe
tachyarrhythmia and hypertension. Autonomic hyper-reflexia can be best
avoided by administration of spinal anaesthesia. Postoperative pain after
haemorrhoidectomy should not be a significant problem in this patient as
the surgery is below the T10 level.

Further reading
1. Teasdale A. Neuromuscular disorders. In: Oxford handbook of
anaesthesia, 1st ed. Allman KG, Wilson IH, Eds. Oxford, UK: Oxford
University Press, 2003; Chapter 9: 180-5.

6 Answer: E. Post-herpetic neuralgia.

Post-herpetic neuralgia (PHN) is pain occurring after Herpes zoster


infection. After the initial infection, the virus remains dormant in the
ganglion of the affected nerve. Reactivation of the virus occurs due to
immunosuppression and therefore is more commonly seen in the elderly,
and in patients with poor nutrition, malignancy, and immunosuppression
due to any cause. The onset of pain is typically followed by the skin rash
in the distribution of the affected nerve. The ophthalmic division of the
set 1 answers_set 1 answers.qxd 22-04-2013 19:17 Page 18

Single Best Answer MCQs in Anaesthesia

trigeminal nerve is the second most commonly affected nerve; the thoracic
dermatome being the commonest. PHN pain is neuropathic in nature and
can be treated by using 5% lidocaine plasters, tricyclic antidepressant
medication (amitriptyline), calcium channel blockers (gabapentin,
pregabalin), sodium channel blockers (phenytoin, carbamazepine) and
non-pharmacological therapies such as TENS, acupuncture and cognitive
behavioural therapy (CBT).

Further reading
1. Dainty P. Prevention and medical management of post-herpetic
neuralgia. British Journal of Hospital Medicine 2008; 69: 275-8.

18
7 Answer: C. Etomidate.

Taking into consideration the history of this patient it would be most


appropriate to use etomidate, which is an intravenous anaesthetic agent
with the lowest incidence of allergic reactions. The incidence of
hypersensitivity reactions amongst induction agents is shown in Table 1.

Table 1. The incidence of hypersensitivity reactions amongst induction


agents.

IV anaesthetic agent Incidence

Propofol 1: 80,000
Thiopentone 1: 20,000 to 40,000
Etomidate 1: 450,000
Ketamine 1: 150,000
Methohexitone 1: 7,000 to 15,000

Further reading
1. Aitkenhead AR. Intravenous anaesthetic agents. In: Textbook of
anaesthesia, 5th ed. Aitkenhead AR, Rowbothom DJ, Smith G, Eds.
London, UK: Churchill Livingstone, 2007; Chapter 3: 34-51.
set 1 answers_set 1 answers.qxd 22-04-2013 19:17 Page 19

Set 1 answers

8 Answer: D. Air embolism.

Air embolism is a well recognised complication of surgery during any


operation in which the operative site is higher than the right atrium. Venous
air embolism (VAE) causes pulmonary microvascular occlusion resulting in
increased physiological dead space. Bronchoconstriction may also
develop. Other signs include hypotension, arrhythmia, increased
pulmonary artery pressure and decreased EtCO2. VAE can be diagnosed
by detection of a sudden reduction in EtCO2, a decrease in blood
pressure, and use of a Doppler, precordial stethoscope (millwheel
murmur), or a transoesophageal stethoscope. Treatment is supportive and
includes informing the surgeon to flood the operative field with saline, 19
discontinuation of nitrous oxide if in use and increasing the FiO2 to 1.0. To
increase the venous pressure, PEEP should be applied and if possible the
position of the operative site should be changed to a level below the heart.
Blood pressure should be supported using intravenous fluids and
vasopressors. If a large volume of air has entered the circulation and the
surgical condition permits, the patient should be turned into the left lateral
position in an attempt to keep the air in the right atrium and aspiration
performed via a central line if present.

Further reading
1. Clayton T, Manara A. Neurosurgery. In: Oxford handbook of
anaesthesia, 1st ed. Allman KG, Wilson IH, Eds. Oxford, UK: Oxford
University Press, 2003; Chapter 19: 418-9.

9 Answer: A. Analysis of blood or stool for


botulinum toxin.

The history is suggestive of botulism. The best diagnostic test is to identify


the toxin in blood or stools. Other conditions which may present similarly
include:

w Poliomyelitis: a febrile illness with asymmetric paralysis. Diagnosis is


by virus culture.
w Guillain Barré syndrome: a febrile illness with loss of sensation.
Diagnosis is by CSF analysis and electrophysiological studies.
set 1 answers_set 1 answers.qxd 22-04-2013 19:17 Page 20

Single Best Answer MCQs in Anaesthesia

w Myasthenia gravis: a fluctuating weakness, which is diagnosed by the


‘Tensilon’ test.

Further reading
1. Wenham T, Cohen A. Botulism. British Journal of Anaesthesia
CEACCP 2008; 8: 21-5.

10 Answer: D. Stop enoxaparin and start an


alternative anticoagulant.

20
This patient’s history is suggestive of heparin-induced thrombocytopaenia
(HIT). HIT is an adverse drug reaction to heparin. Adverse reactions are
either non-immune-mediated (type I) or immune-mediated (type II). The
non-immune mediated reaction typically has an earlier onset and seldom
leads to a drop in the platelet count below 100 x 109/L. The immune-
mediated reaction is clinically more significant as it is associated with
thrombosis. It occurs between 5-14 days post-heparin exposure and this
is known as ‘typical’ HIT. The incidence of HIT is estimated at 1% with
low-molecular-weight heparins (LMWH) and about 5% with unfractionated
heparins (UFH). HIT has been described with every route of heparin
administration. Diagnosis of HIT requires a low threshold of suspicion.
Other features include a systemic response to heparin injection and overt
disseminated intravascular coagulopathy. Where clinical suspicion of HIT
is intermediate to high, it is essential to stop UFH or LMWH. There should
be no delay in commencing anticoagulation with alternative agents while
awaiting confirmatory tests, as the risk of thrombosis remains as high as
50% even after stopping heparin.

Further reading
1. Warkentin TE. Heparin-induced thrombocytopaenia: diagnosis and
management. Circulation 2004; 110: 454-8.

11 Answer: C. Intrathecal saddle neurolytic block.

Cancer pain can be successfully managed in about 90% of patients using


the WHO analgesic ladder. In the remaining 10% of patients,
interventional therapy may be required. The nature of pain can be both
set 1 answers_set 1 answers.qxd 22-04-2013 19:17 Page 21

Set 1 answers

nociceptive and neuropathic. At times, the tolerance to opioid medication


leads to poor pain control and ‘opioid rotation’ may be effective in such
cases. In this patient switching MST to equivalent doses of oxycodone
(‘opioid rotation’) and increasing the opioid medication dose has been
already tried but without satisfactory response. The next appropriate step
would be an interventional therapy. Coeliac plexus block is effective only
for abdominal malignancy and is therefore not a suitable option. Lumbar
sympathectomy is generally effective for lower limb pain. In this particular
patient, intrathecal neurolysis, using hyperbaric phenol to provide a saddle
block, would be most appropriate. It can cause leg numbness but this can
be avoided by carefully restricting it to a saddle block. Alcohol is hypobaric

21
compared to CSF and can also be used for neurolysis in the management
of malignant pain.

Further reading
1. Medicis E, Laon-Casasoal OA. Neurolytic blocks. Clinical pain
management - practical applications and procedures, 1st ed. Breivik
H, Campbell W, Eccleston C, Eds. London, UK: Arnold, 2002;
Chapter 19: 247-54.

12 Answer: B. Intravenous magnesium sulphate


infusion, following a loading dose.
This woman has severe pre-eclampsia, as defined by the level of both
systolic and diastolic hypertension and presence of cerebral symptoms.
She is at risk of eclampsia. The MAGPIE study has demonstrated that
administration of magnesium sulphate (loading dose over 1 hour followed
by infusion) to women with pre-eclampsia reduces the risk of an eclamptic
seizure by over 50%. It should be continued for 24 hours following
delivery or 24 hours after the last seizure, whichever is the later. Sublingual
nifedipine is not recommended. Other antihypertensive agents to consider
are oral nifedipine, an intravenous labetalol infusion and intravenous
hydralazine. In this case, epidural analgesia is not required as the woman
is not in labour and a Caesarean section is planned.

Further reading
1. RCOG Green-Top 10A guideline: The management of severe pre-
eclampsia/eclampsia. (www.rcog.org.uk).
set 1 answers_set 1 answers.qxd 22-04-2013 19:17 Page 22

Single Best Answer MCQs in Anaesthesia

2. Magpie Trial Collaborative Group: Do women with pre-eclampsia, and


their babies, benefit from magnesium sulphate? The Magpie Trial: a
randomised placebo-controlled trial. Lancet 2002; 359: 1877-90.

13 Answer: A. An atonic uterus.

The most common cause of primary (within 24 hours) postpartum


haemorrhage is an atonic uterus, occurring in about 70% of cases. The
other options mentioned above are possible causes which need to be
excluded. In particular, tears may cause significant haemorrhage and can

22
be more difficult to diagnose if high in the genital tract, often requiring an
examination under anaesthesia. Factors predisposing to an atonic uterus
include a large baby (in this case), multiple pregnancy, prolonged labour
(especially if augmented with syntocinon), abnormal placentation,
multiparity, and chorioamnionitis.

14 Answer: B. Unilateral pupillary dilatation.

Traumatic head injury is considered to be severe if the GCS is gradually


deteriorating. The usual clinical presentation of an extradural haematoma
is severe headache, loss of consciousness with or without a lucid interval
and rapid development of a fixed dilated pupil on the side of the injury with
contralateral hemiparesis. Extradural haematoma occurs due to rupture of
the middle meningeal artery. It occurs in approximately 10% of severe
head injuries. Immediate surgery is required to evacuate the haematoma.
Severe head injury can lead to status epilepticus and it may indicate the
need for intubation and ventilation, but not surgery. CSF rhinorrhoea is
associated with a base of skull fracture and surgery may be required if it
does not stop spontaneously.

Further reading
1. Lannoo E, Van Rietvelde F, Colardyn F, et al. Early predictors of
mortality and morbidity after severe closed head injury. J Neurotrauma
2000; 17: 403-14.
2. Triage, assessment, investigation and early management of head injury
in infants, children and adults. NICE guidelines CG 56, September
2007. (http://www.nice.org.uk/CG56).
set 1 answers_set 1 answers.qxd 22-04-2013 19:17 Page 23

Set 1 answers

15 Answer: C. Atosiban.

Preterm birth is defined as that occurring before 37 completed weeks, but


most mortality and morbidity is experienced in babies born before 34
weeks. Ritodrine has predominantly b2-receptor effects, relaxing the
muscles in the uterus, arterioles and bronchi. It is not recommended as it
is associated with a relatively high incidence of pulmonary oedema.
Atosiban (an oxytocin receptor antagonist) and nifedipine appear to have
comparable effectiveness in delaying delivery for a few days, with fewer
maternal adverse effects and less risk of rare serious adverse events.
Nifedipine has the advantage of oral use and it is cheap. However, it is not
licensed in the UK for use as a tocolytic. Though both magnesium sulphate 23
and GTN have a relaxant effect on the uterus, their use in this situation is
not recommended as frequent monitoring of blood pressure and other vital
parameters will be required. A salbutamol nebuliser will only have a
transient effect on the uterus.

Further reading
1. RCOG Green-Top 1B guideline: Tocolytic drugs for women in preterm
labour. (www.rcog.org.uk).

16 Answer: C. Multimodal analgesia with a


morphine PCA.
Acute postoperative pain relief is a challenging issue in patients on long-
term opioids or with opioid addiction. In these patients, non-opioid
analgesics should be used if possible and if opioid analgesics are
indicated, then fast onset, short-acting opioids are preferred to allow dose
adjustment. Partial agonist (e.g. buprenorphine) and agonist-antagonist
(e.g. pentazocine) opioid drugs should be avoided. The use of a morphine
PCA allows dose titration in a safer way and also helps to avoid possible
patient confrontation with health care professionals. In patients with
unmanageable pain after high doses of morphine, a ketamine infusion can
be used in a dose of 0.2 to 0.3mg/kg/hr for 24 to 48 hours and it may, by
its NMDA antagonist action, reverse opioid tolerance in addition to its
analgesic action. Regional analgesia using a brachial plexus block may
mask the clinical signs of compartment syndrome.
set 1 answers_set 1 answers.qxd 22-04-2013 19:17 Page 24

Single Best Answer MCQs in Anaesthesia

Further reading
1. Mehta V, Langford RM. Acute pain management for opioid-dependent
patients. Anaesthesia 2006; 61: 269-76.

17 Answer: C. Indirect maternal death.

This is a maternal death occurring within 42 days of the end of pregnancy.


The suicide was related to the pregnancy, but there is no obstetric cause
and so the death is classified as an indirect maternal death. In the last two
triennial reports, suicide has been the leading cause of maternal death
overall.
24

Table 2. Definition of maternal deaths.

Maternal deaths Deaths of women while pregnant or within 42 days of


the end of the pregnancy, from any cause related to
or aggravated by the pregnancy or its management,
but not from accidental or incidental causes

Direct Deaths resulting from obstetric complications of the


pregnant state (pregnancy, labour and puerperium),
from interventions, omissions, incorrect treatment or
from a chain of events resulting from any of the above

Indirect Deaths resulting from previous existing disease, or


disease that developed during pregnancy and which
was not due to direct obstetric causes, but which
was aggravated by the physiologic effects of
pregnancy

Late Deaths occurring between 42 days and 1 year after


abortion, miscarriage or delivery that are due to direct
or indirect maternal causes

Coincidental Deaths from unrelated causes which happen to occur


(fortuitous) in pregnancy or the puerperium
set 1 answers_set 1 answers.qxd 22-04-2013 19:17 Page 25

Set 1 answers

Further reading
1. Why mothers die 2000-2002; http://www.cmace.org.uk/Publications/
Saving-Mothers-Lives-Report-2000-2002.aspx.

18 Answer: B. Hypokalaemia.

Hypokalaemia is the most likely cause for the ECG abnormality because:

w The baseline plasma potassium level is low at only 3.4mmol/L.


w Hyperventilation leading to respiratory alkalosis can shift the
potassium into the cells, thereby reducing the potassium in the
extracellular fluid and further lowering the plasma potassium. 25
w The ECG abnormalities of hypokalaemia include T-wave inversion, ST
segment depression, a prolonged PR interval and prominent U waves.

Further reading
1. Hypokalaemia. In: ECG diagnosis made easy. Vecht RJ, Ed. Martin
Dunitz, 2001: 185.
2. Abnormal potassium balance. In: Lecture notes on fluid and electrolyte
balance, 2nd ed. Willatts SM, Ed. Oxford, UK: Blackwell Scientific
Publications, 1987; Chapter 8: 167-76.

19 Answer: D. Chest X-ray.

A chest X-ray should confirm the correct placement of a central venous


line. The catheter tip should be just above the pericardial reflection,
outside the cardiac silhouette. The catheter should be within the superior
vena cava (SVC), relatively parallel to the walls of the SVC. The catheter
tip should abut against the wall of the SVC. All other mentioned methods
in the placement of the catheter may be within the SVC or a major vein.

20 Answer: A. Median nerve.

The clinical features described are suggestive of carpal tunnel syndrome,


caused by compression of the median nerve within the carpal tunnel.
Initially symptoms appear typically during the night because flexing of the
set 1 answers_set 1 answers.qxd 22-04-2013 19:17 Page 26

Single Best Answer MCQs in Anaesthesia

wrist during sleep causes further compression of the median nerve. The
contents of the carpal tunnel include the median nerve, the tendon of flexor
pollicis longus, and tendons of flexor digitorum superficialis and
profundus.

Further reading
1. Nerve and muscle. In: Lecture notes on neurology, 7th ed. Ginsberg
L, Ed. Oxford, UK: Blackwell Science, 1999; Chapter 17: 146-7.

21 Answer: B. Commence treatment with


carbamazepine.
26
This patient has a classical history of trigeminal neuralgia pain. Trigeminal
neuralgia usually presents in middle-aged patients with unilateral
neuropathic pain in the distribution of one or more divisions of the
trigeminal nerve. Commonly, the maxillary and/or mandibular divisions are
affected. The pain lasts for a short period and in between the episodes,
there are no symptoms or signs. The pain can be severe enough to induce
suicidal thoughts. The exact pathophysiology is not known but in some
patients vascular compression of the trigeminal ganglion has been
implicated. The pain responds to medications used to treat neuropathic
pain in about 70% of patients and traditionally the first-line drug used is
carbamazepine. About 15-30% may respond to microvascular
decompression of the trigeminal ganglion and this is indicated if the MRI
scan shows vascular compression of the ganglion. If the patient does not
respond to medication therapy, MRI of the brain can be performed to rule
out other pathology.

Further reading
1. Sindrup SH, Jensen TS. Pharmacotherapy of trigeminal neuralgia. The
Clinical Journal of Pain 2002; 18: 22-7.

22 Answer: A. Airway oedema.

Airway oedema as a result of repeated laryngoscopy and intubation


attempts can result in inspiratory stridor along with desaturation. Other
differential diagnoses include laryngospasm, arytenoid subluxation or
set 1 answers_set 1 answers.qxd 22-04-2013 19:17 Page 27

Set 1 answers

dislocation, but these are unlikely to improve with nebulised adrenaline. A


flexible nasendoscopy should confirm the diagnosis. Arytenoid subluxation
can be treated with voice therapy.

Further reading
1. Tan V, Seevanayagam S. Arytenoid subluxation after a difficult
intubation treated successfully with voice therapy. Anesthesia and
Intensive Care 2009; 37: 843-6.

23 Answer: B. Performing a fibreoptic-assisted


intubation through the LMA.
27
The ‘plan B’ of the Difficult Airway Society guidelines includes tracheal
intubation through an LMA or intubating LMA. In this case, as the airway
is secured and the patient is well oxygenated using the LMA, the most
appropriate decision is to perform fibreoptic-assisted tracheal intubation
through the LMA. Blind tracheal intubation through an LMA has a low
success rate and can cause airway trauma. One-stage fibreoptic
intubation performed by directly loading the endotracheal tube over the
fibreoptic scope has certain limitations. It requires a longer tube such as a
microlaryngoscopy tube or a north facing polar nasal tube. It also limits the
size of the endotracheal tube, as the LMA tube only allows an
endotracheal tube of up to 6 to 7mm internal diameter depending on the
size and type of LMA. A two-stage technique using an Aintree intubation
catheter overcomes these limitations. Using an LMA in the prone position
would place the patient’s airway at risk.

Further reading
1. Henderson JJ, et al. Difficult Airway Society guidelines for the
management of the unanticipated difficult intubation. Anaesthesia
2004; 59: 675-94.

24 Answer: B. Haemodialysis.

This patient has severe lactic acidosis with pre-existing impaired renal
function. Metformin is a biguanide oral hypoglycaemic agent. The
set 1 answers_set 1 answers.qxd 22-04-2013 19:17 Page 28

Single Best Answer MCQs in Anaesthesia

mechanism of its action involves reduced intestinal absorption of glucose,


reduced gluconeogenesis and increased peripheral utilisation of glucose
due to increased insulin sensitivity. Biguanides cause type B lactic
acidosis by increasing lactic acid production whilst impairing its removal
by the kidneys and liver. Metformin is contraindicated in the presence of
severe hepatic or renal impairment. Treatment with sodium bicarbonate
alone fails to correct acidosis, and may cause intracellular acidosis and
hypernatraemia. Haemodialysis or continuous venovenous haemofiltration
has been shown to have a better outcome, removing the metformin and
correcting the acidosis. Tri-hydroxymethyl aminomethane (THAM) is an
organic amine proton acceptor, and should be used with caution in the

28
presence of renal impairment as it is associated with hyperkalaemia.

Further reading
1. Pan LTT, MacLaren G. Continuous venovenous haemofiltration for
metformin induced lactic acidosis. Anesthesia and Intensive Care
2009; 37: 830-2.
2. Teale KFH, Devine A, et al. The managemnet of metformin overdose.
Anaesthesia 1998; 53: 698-701.

25 Answer: D. Magnetic resonance imaging of the


brain.
The most likely cause for the deterioration of this patient is central pontine
myelinolysis (CPM). Acute hyponatraemia results in cerebral oedema, due
to the inability of neuronal cells to extrude potassium. In chronic
hyponatraemia, however, neuronal cells may adapt by extrusion of organic
osmolytes from their cytoplasm. Osmotic demyelination is a recognised
complication of rapid correction of hyponatraemia. Patients with severe
malnutrition, alcoholism, and advanced liver disease are more susceptible
to CPM. In order to avoid CPM, it is recommended that during the first 24
hours the total increase in serum sodium should not exceed 20mmol/L.
The diagnosis can be confirmed by an MRI scan (T2 weighed images),
which would show high intensity lesions (bright areas) in the region of the
central pons. Normally there is symmetric, non-inflammatory demyelination
in the central part of the pons. In 10% of patients with central pontine
myelinolysis, however, demyelination also occurs in extrapontine regions,
including the midbrain, thalamus, basal nuclei and cerebellum.
set 1 answers_set 1 answers.qxd 22-04-2013 19:17 Page 29

Set 1 answers

Further reading
1. Luzzio C. Central pontine myelinosis: differential diagnosis and
workup. (http://emedicine.medscape.com/article/1174329-overview).
2. Schuster M, Diekmann S, et al. Central pontine myelinosis despite
slow sodium rise in a case of severe community-acquired
hyponatraemia. Anesthesia and Intensive Care 2009; 37: 117-20.

26 Answer: D. Progressive rise in sedation level.

Opioids are commonly used in the management of intra-operative and

29
postoperative pain. Excessive doses of opioid initially cause rising
sedation levels, confusion, nightmares, hallucinations and at this stage
delaying a further dose or reducing the dose is enough to alleviate the
problem. If not recognized, the patient will develop respiratory depression
followed by a decrease in oxygen saturation. Oxygen saturation changes
may not be evident in the early stages especially if the patient is on
supplementary oxygen.

Further reading
1. Tran ML, Warfield C. Opioid analgesics. In: Clinical pain management
- practical applications and procedures, 1st ed. Breivik H, Campbell
W, Eccleston C, Eds. London, UK: Arnold, 2002; Chapter 6: 59-76.

27 Answer: B. Chest X-ray.

It is essential to ensure the correct position of the NG tube prior to


commencing feeding through it. A chest X-ray is the best method to
confirm the correct placement. Although the other mentioned methods
may indicate the placement of the tube in the stomach, they are not always
reliable. It may be difficult to hear the ‘whoosh’ sound on injecting air in
obese patients.

Further reading
1. Reducing the harm caused by nasogastric tubes - interim advice for
health care staff, 2005. (http://www.baxa.com/resources/docs/
research /NPSAConfPosofNG.pdf).
set 1 answers_set 1 answers.qxd 22-04-2013 19:17 Page 30

Single Best Answer MCQs in Anaesthesia

28 Answer: A. CT scan of the lumbar region.

The epidural catheter is radio-opaque; it may be visible on plain X-ray but


a CT scan is the most helpful in delineating the location of the broken
catheter tip. The catheter fragment is unlikely to cause any further
problems to the patient, but fibrosis around the nerve root may produce
signs of radicular irritation. An immediate management plan for labour
analgesia should be instituted. This may include discussion with the
patient and obstetrician and depends on the progress of labour. Alternate
modes of analgesia and performing another epidural should be
considered.
30
Further reading
1. Fragneto RY. The broken epidural catheter: an anesthesiologist’s
dilemma. Journal of Clinical Anesthesia 2007; 19: 243-4.

29 Answer: B. Dantrolene sodium 2-3mg/kg as an


initial bolus.

The clinical scenario is suggestive of malignant hyperthermia (MH):


unexplained tachycardia, an increase in EtCO2 and hyperthermia. The
correct dose of dantrolene is 2-3mg/kg as an initial bolus followed by
1mg/kg PRN. The other immediate measures include removing the trigger
agent, and maintaining anaesthesia with total intravenous anaesthesia.
Active cooling measures and intravenous infusion of cold I.V. fluids should
be performed. End-tidal CO2, invasive arterial BP, CVP, core and
peripheral temperature, urine output and pH, arterial blood gases,
potassium, haematocrit, platelets, clotting indices, and creatine kinase
should be monitored. The systemic effects of MH include hyperkalaemia,
cardiac arrhythmias, myoglobinaemia and disseminated intravascular
coagulation.

Further reading
1. Guidelines for the management of malignant hyperthermia crisis. The
Association of Anaesthetists of Great Britain and Ireland, 2007.
set 1 answers_set 1 answers.qxd 22-04-2013 19:17 Page 31

Set 1 answers

30 Answer: C. Flooding the field with normal saline.

The main components of the ‘fire triangle’ include fuel, oxygen and energy.
The ‘Laser flex’ endotracheal tube is a metallic tube with two cuffs. Both
cuffs need to be filled with saline. The cuff part of the tube can act as a
fuel. The laser beam acts as a source of energy. Both oxygen and nitrous
oxide supports combustion. The mixture of oxygen and nitrous oxide is
more flammable than an oxygen and air mixture. A lowest possible inspired
oxygen concentration should be used during laser surgery of the airway.

1. Kitching AJ, Edge CJ. Lasers and surgery. British Journal of


Anaesthesia CPD review 2003; 8: 143-6. 31
set 1 answers_set 1 answers.qxd 22-04-2013 19:17 Page 32

Single Best Answer MCQs in Anaesthesia

32
set 2_set 2.qxd 22-04-2013 19:18 Page 33

Set 2 questions

Set 2
1 A 50-year-old female patient with a history of non-insulin-dependent
diabetes has undergone trans-sphenoidal excision of a pituitary 33
adenoma. During the immediate postoperative period she develops
polyuria with a urine output of 600ml over 2 hours. The urine
osmolarity is 320mosmol/L and the specific gravity is 1.001. The
most appropriate treatment for this patient is:

a. Intravenous 0.9% sodium chloride.


b. Intravenous DDAVP.
c. Intravenous glucose.
d. Intravenous insulin.
e. Intravenous glucose and potassium.

2 A 48-year-old female patient involved in a road traffic accident about


36 hours ago is scheduled for an open reduction of bilateral
mandibular fractures. She complains of neck pain and cervical spine
injury is suspected. The best possible investigation to exclude
cervical spine injury is:

a. Cervical spine X-ray lateral view.


b. Cervical spine X-ray anteroposterior view.
c. MRI scan of the cervical spine.
d. Helical CT with sagittal reformat.
e. Dynamic fluoroscopy.
set 2_set 2.qxd 22-04-2013 19:18 Page 34

Single Best Answer MCQs in Anaesthesia

3 A 53-year-old female patient is anaesthetised for an emergency


laparotomy. She is obese with a BMI of 39. After induction of
anaesthesia a central venous catheter is placed via the right
subclavian vein following two failed attempts via the right internal
jugular vein. About 30 minutes after starting the procedure, the
airway pressure and heart rate increase and the oxygen saturation
decreases to 88%. The most likely cause is:

a. A displaced endotracheal tube.


b. Severe bronchospasm.
c. Kinking of the endotracheal tube.
34 d. Anaphylaxis.
e. Tension pneumothorax.

4 A 69-year-old male patient is admitted to the intensive care unit


following emergency repair of a leaking abdominal aortic aneurysm.
On admission to the unit, he is hypotensive, and requires inotropic
support. The urine output over the last 12 hours is only 30mL. Blood
and urine analysis reveal: creatinine – 545mmol/L, urine osmolality –
165mmol/L and a urine: plasma creatinine ratio of 25. The most
likely cause of the low urine output is:

a. Intrinsic renal failure.


b. Severe dehydration.
c. Pre-renal type of renal failure.
d. Post-renal type of failure.
e. Adrenal failure with hypotension.

5 A 50-year-old male patient has been admitted to the high


dependency unit following surgery for bilateral fractures of the lower
limbs. His medical history includes bipolar disorder and mild
hypertension, and he is treated with lithium 400mg per day and
bendrofluazide 2.5mg per day. During the intra-operative period,
gentamicin 240mg and flucloxacillin 1g is administered as antibiotic
prophylaxis. Since admission to the unit he has developed significant
polyuria with a urine output of 200-400ml per hour for the past 6
set 2_set 2.qxd 02-05-2013 19:24 Page 35

Set 2

hours. There is no myoglobin in the urine. The most likely cause for
his polyuria is:

a. Rhabdomyolysis.
b. Central diabetes insipidus.
c. Nephrogenic diabetes insipidus.
d. Bendrofluazide.
e. Antibiotic therapy.

6 You have been asked to review a 67-year-old patient on the fourth

35
postoperative day, who is complaining of severe back pain and
increasing numbness in both legs developing over the previous few
hours. He has had an epidural in situ since his operation; the
epidural infusion was switched off 8 hours ago as his blood pressure
had been low. For the last few hours his temperature has been
38°C. Your first step in the management of this patient should be:

a. Give an epidural top-up using 0.5% bupivacaine.


b. Remove the epidural catheter and start morphine PCA.
c. Arrange for an MRI scan of the spine.
d. Arrange for surgical review as soon as possible.
e. Start multimodal analgesia and broad spectrum antibiotics.

7 A 75-year-old male patient had a transurethral resection of the


prostate under spinal anaesthesia 2 weeks ago. He now presents
with weakness of the left foot. On clinical examination he has loss of
sensation over the dorsum of the left foot and motor power is grade
3/5 for dorsiflexion of the left foot. The most useful investigation in
establishing the diagnosis would be:

a. MRI scan of the lumbar spine.


b. Fasting blood glucose.
c. A complete neurological examination.
d. CT scan of the lumbar spine.
e. Electromyography.
set 2_set 2.qxd 02-05-2013 19:24 Page 36

Single Best Answer MCQs in Anaesthesia

8 A 65-year-old female patient with a history of ischaemic heart


disease is scheduled for a laparoscopic cholecystectomy. A 12-lead
ECG has been performed as part of a routine pre-operative
assessment. Which of the following abnormal findings is most likely
to be present on the ECG?

a. Tall R waves in leads V5 and V6.


b. ST-T wave abnormalities.
c. Pathological Q waves.
d. Right bundle branch block.

36
e. Deep S waves in lead V1.

9 A 45-year-old man presents to the emergency department with


severe constant epigastric pain radiating to the back and flanks, and
vomiting. Examination reveals pyrexia, abdominal distension,
rebound tenderness and discolouration of the flanks. Blood tests
reveal elevated lipase and amylase levels. Which one of the
following is the recommended investigation for an initial
assessment?

a. Biliary tract ultrasound.


b. X-ray abdomen.
c. Angiography.
d. CT scan.
e. MRI scan.

10 A 30-year-old male is undergoing a laparotomy for a ruptured


spleen, liver laceration and bowel injury following a road traffic
accident. The estimated blood loss so far is 4L. During the
procedure 8 units of packed red cells and 3L of Hartmann’s solution
have been infused. There is increased bleeding from the wound
edge and from the site of venous access. The following laboratory
test supports the diagnosis of DIC rather than dilutional
coagulopathy:
set 2_set 2.qxd 02-05-2013 19:24 Page 37

Set 2

a. Haemoglobin.
b. D-Dimer.
c. Platelet count.
d. Bleeding time.
e. INR.

11 A 48-year-old male lorry driver presents with severe right-sided


sciatica which he has suffered from for the last 9 months. He has
already tried treatment with analgesics, physiotherapy and
acupuncture. His MRI scan shows moderate disc prolapse at the

37
L5-S1 level. He does not wish to undergo any surgical intervention.
The most suitable treatment is:

a. Stronger opioid medication.


b. Facet joint injection.
c. Epidural steroid injection.
d. Traction therapy to the lumbar spine.
e. Six weeks’ bed rest.

12 A 50-year-old female patient has had a parathyroidectomy for


hyperparathyroidism secondary to chronic renal failure. She has a
history of arthritis, backache and neck pain. In the recovery room,
she complains of weakness in all four limbs. The clinical examination
reveals reduced power in all four limbs and reduced sensation
below the C6 dermatome. The most useful investigation in
establishing the cause of weakness would be:

a. CT scan of the head.


b. MRI scan of the cervical spine.
c. Serum calcium level.
d. MRI scan of the head.
e. Electromyography.
set 2_set 2.qxd 02-05-2013 19:24 Page 38

Single Best Answer MCQs in Anaesthesia

13 A 4-week baby with a history of projectile vomiting for the last few
days has been diagnosed with pyloric stenosis. Which one of the
following parameters is most likely to suggest severe volume
depletion?

a. Urine output of 0.5ml/hour.


b. Serum sodium 129mmol/L.
c. Metabolic alkalosis with alkaline urine.
d. Metabolic alkalosis with acidic urine.
e. Serum chloride 100mmol/L.

14 A
38
60-year-old male patient with a history of hypertension and
ischaemic heart disease is scheduled for a carotid endarterectomy
under general anaesthesia. Which of the following would be the
most appropriate monitor of peri-operative cerebral ischaemia?

a. Electro-encephalography.
b. Somatosensory evoked potentials.
c. Transcranial Doppler.
d. Motor evoked potentials.
e. Auditory evoked potentials.

15 A 68-year-old male with known hypertension and a long-term smoker


has been diagnosed with bronchogenic carcinoma and is being
assessed for his suitability for a right-sided pneumonectomy. His
post-bronchodilator FEV1 is 1.6L. Which one of the following tests
should be performed next to assess suitability for the
pneumonectomy?

a. Best distance on two shuttle walk test.


b. Arterial blood gas analysis.
c. Cardiopulmonary exercise test.
d. Pulmonary function tests to estimate postoperative FEV1.
e. CT scan of the chest.
set 2_set 2.qxd 02-05-2013 19:24 Page 39

Set 2

16 A 7-year-old boy is in severe pain and distress in the recovery room


following urgent open reduction and internal fixation of a fractured
radius. He is awake and is tachycardic. Intra-operative analgesia
included fentanyl 3µg/kg I.V., paracetamol 500mg rectally and
diclofenac 25mg rectally. What would be the most appropriate
analgesic option for him now?

a. Intravenous morphine infusion at 10µg/kg/hour.


b. Intramuscular codeine phosphate 1mg/kg.
c. Axillary brachial plexus block.
d. Morphine 100µg/kg as an intravenous bolus.

39
e. Administer entonox until he calms down.

17 A 57-year-old female is due to undergo urgent surgery for internal


fixation of a fractured humerus. She is known to have primary
hyperparathyroidism. Pre-operative blood results and the ECG are
unremarkable except for a serum calcium level of 3.3mmol/L. Which
initial therapy should be used to treat hypercalcaemia in this patient?

a. Intravenous corticosteroids.
b. Intravenous saline and furosemide.
c. Intravenous calcitonin.
d. Haemodialysis.
e. Intravenous biphosphonates.

18 A 53-year-old female patient is undergoing a total hip replacement


under spinal anaesthesia. She has a history of bipolar disorder and
has been on lithium for the last 2 years. Half-way through the
procedure she complains of discomfort in the chest and the ECG
shows irregular, broad complexes (Torsade de pointes). Her blood
pressure is 100/70mmHg. Which of the following anti-arrhythmic
treatment should be administered to this patient?

a. Isoprenaline infusion.
b. Intravenous lidocaine 2mg/kg.
c. Intravenous phenytoin 15mg/kg.
d. Intravenous magnesium 2g.
e. Intravenous potassium chloride.
set 2_set 2.qxd 02-05-2013 19:24 Page 40

Single Best Answer MCQs in Anaesthesia

19 A 73-year-old female was found collapsed at home. On admission to


the emergency department she is confused and her core
temperature is 33.2°C. Her other vital parameters are within the
normal range. It is suspected that she has suffered a minor
cerebrovascular event. Which of the following would be the best
treatment of hypothermia in this patient?

a. Oxygen supplementation with rapid re-warming using gastric and


bladder warm fluid lavage.
b. Oxygen supplementation with re-warming using warm intravenous
fluid and warming blankets.
40
c. Intubation and ventilation and rewarming using dialysis or
cardiopulmonary bypass.
d. Intubation and ventilation with rapid rewarming using gastric and
bladder warm fluid lavage.
e. Intubation and ventilation with warmed intravenous fluid/warming
blankets.

20 A 28-year-old female patient is scheduled for correction of


kyphoscoliosis and insertion of Harrington rods. Which of the
following intra-operative monitoring is most useful in detecting
neurological injury during instrumentation of the spine?

a. Wake-up test.
b. Bispectral index.
c. Somatosensory evoked potentials.
d. Invasive blood pressure monitoring.
e. Peripheral nerve stimulation.

21 A 36-year-old male presents to the pain clinic with a history of pain


in the left forearm associated with burning sensations, muscle
spasm, swelling and discolouration. Three months before he
suffered a fracture of the scaphoid bone of his left hand, which was
treated surgically under general anaesthesia. The most likely
diagnosis is:
set 2_set 2.qxd 02-05-2013 19:24 Page 41

Set 2

a. Complex regional pain syndrome type I.


b. Post-surgical pain due to nerve damage.
c. Complex regional pain syndrome type II.
d. Peripheral vascular disease involving the upper limb.
e. Peripheral nerve injury occurring during general anaesthesia.

22 A 24-year-old man presents for extraction of two upper molar teeth


under general anaesthesia. He gives a history of episodes of
haemarthrosis. Blood investigations have revealed an elevated
activated partial thromboplastin time (aPTT), a normal platelet count
and a normal prothrombin time (PT). Which of the following
haematological disorders is most likely to be present in this patient? 41

a. von Willebrand disease.


b. Haemophilia A.
c. Factor V deficiency.
d. Afibrinogenaemia.
e. Factor XIII deficiency.

23 A 50-year-old female patient presents for decompression of a


thoracic epidural abscess. Her weight is 96kg and height is 158cm.
She has developed increasing weakness of both lower limbs over
the past 2 days. General anaesthesia is induced with propofol,
atracurium and remifentanil. Soon after tracheal intubation her
oxygen saturation decreases to 89%, which does not respond to
increasing the inspired oxygen concentration to 100%. On
auscultation bilateral equal air entry is confirmed with no added
sounds. Her EtCO2 decreases to 2.5kPa whilst her minute
ventilation is maintained at 8L/minute. Which of these is the most
likely cause of the hypoxia?

a. Pulmonary embolism.
b. Endobronchial intubation.
c. Severe bronchospasm.
d. Pulmonary oedema.
e. Air embolism.
set 2_set 2.qxd 02-05-2013 19:24 Page 42

Single Best Answer MCQs in Anaesthesia

24 A 26-year-old woman, on her first postnatal day, suffers a fit on the


postnatal ward. The first step in the immediate management should
be:

a. Intravenous magnesium sulphate, with a loading dose of 4g over 10


minutes.
b. Intravenous lorazepam 4mg.
c. Rapid sequence induction with thiopentone and suxamethonium.
d. Oxygen via a face mask with a reservoir bag at 15L/min.
e. Phenytoin, with a loading dose of 18mg/kg over 1 hour.

25 A 65-year-old male patient undergoing an arthroscopic procedure on


42

the right shoulder under general anaesthesia and an interscalene


block in the sitting position develops bradycardia and hypotension
during the intra-operative period. The interscalene block was
performed using 20ml of 1% lidocaine with epinephrine. The most
appropriate treatment is:

a. Ephedrine.
b. Glycopyrrolate.
c. Atropine.
d. Metaraminol.
e. Phenylephrine.

26 A 78-year-old male presents with pain in the lower back which he


has suffered for the last 2 years. The pain radiates bilaterally to the
back of the thigh and groin and is increased by extension and
twisting movements at the lumbar spine. The most likely cause of his
pain is:

a. Discogenic lower back pain.


b. Referred pain from sacro-iliac joint arthropathy.
c. Pain due to facet joint arthropathy.
d. Pain due to spinal stenosis.
e. Pain due to muscle spasm.
set 2_set 2.qxd 02-05-2013 19:24 Page 43

Set 2

27 An 84-year-old woman with rheumatoid arthritis presents for an


elective cholecystectomy. At the pre-anaesthetic assessment, she
describes severe neck pain with numbness and tingling in both
arms. Which of the following investigations is most useful in the
further management of this case?

a. Proceed with general anaesthesia with manual in-line stabilisation.


b. Perform lateral cervical spine X-rays.
c. Arrange nerve conduction studies.
d. Perform a flexion view of the cervical spine.

43
e. Perform an MRI scan of the cervical spine.

28 A 35-year-old female patient with Crohn’s disease presents for


elective laparotomy and resection of a small bowel stricture. She has
been taking 12.5mg prednisolone once a day for the last 6 months
to control her disease. Which one of the following would be the
most appropriate peri-operative management of her steroid
treatment?

a. Continuation only of 12.5mg prednisolone once a day.


b. 50mg intravenous hydrocortisone at induction and her usual steroid
dose after surgery.
c. Delay the surgery for 3 months.
d. Usual steroid dose on the morning of surgery and hydrocortisone
50mg intravenously at induction, followed by 50mg three times a day
by intravenous injection for 24 hours.
e. Usual steroid dose on the morning of surgery and hydrocortisone
50mg intravenously at induction, followed by 50mg three times a day
by intravenous injection for 48-72 hours.

29 A woman delivers a baby by normal vaginal delivery. A needle-


through-needle combined spinal and epidural (CSE) was inserted 2
hours prior to delivery to provide labour analgesia. An initial
intrathecal dose of 5µg fentanyl and 2.5mg bupivacaine was given
set 2_set 2.qxd 02-05-2013 19:24 Page 44

Single Best Answer MCQs in Anaesthesia

and nothing via the epidural component was required. Twenty-four


hours after delivery, she has unilateral foot drop (no other
neurology). The best course of action is:

a. Reassurance that this is not due to the epidural.


b. An MRI scan.
c. A further review in 24 hours time.
d. Immediate nerve conduction studies.
e. A CT scan.

44
30 A 63-year-old male develops a brochopleural fistula following a
pneumonectomy. He is ventilated on the ICU, but achieving
adequate tidal volume is proving to be difficult due to an air leak of
2.5L/min through the fistula. Which one of the following would be
most effective in achieving adequate ventilation in this patient?

a. Adding PEEP of 7.5cm of H2O.


b. Decreasing the inflation pressure.
c. Increasing the flow rate by 2.5L/min.
d. High frequency jet ventilation.
e. Decreasing the respiratory rate.
set 2 answers_set 2 answers.qxd 22-04-2013 19:18 Page 45

Set 2

Set 2 answers
answers

1 Answer: B. Intravenous DDAVP.


45
Diabetes insipidus is the common endocrine dysfunction following trans-
sphenoidal hypophysectomy. The diagnosis of diabetes insipidus is
supported by a high urine output, low serum osmolarity and low specific
gravity of the urine. Other causes of high urine output include diuresis
secondary to hyperglycaemia, mannitol or crystalloid administration.
Neurogenic diabetes insipidus should be treated with 5 units of aqueous
vasopressin subcutaneously or 2-4mg of DDAVP (1-desamino 8-D
arginine vasopressin) I.V. or S.C. In neurogenic diabetes insipidus, urine
osmolarity increases following administration of vasopressin or DDAVP.

Further reading
1. Cronin AJ. Acute postcraniotomy agitation. In: Near misses in
neuroanaesthesia. Russell GB, Cronin AJ, Longo S, Blackburn TW,
Eds. Butterworth Heinemann, 2002; Case 43: 155-7.
2. Osborn IP. Trans-sphenoidal hypophysectomy. In: Clinical cases in
anaesthesia, 3rd ed. Reed AP, Yudkowitz FS, Eds. Philadelphia, USA:
Elsevier Churchill Livingstone, 2005; Case 22: 113-6.

2 Answer: D. Helical CT with sagittal reformat.

Although lateral and anteroposterior cervical spine X-rays with an open


mouth view have been considered ‘gold standards’ for the exclusion of c-
spine injury, they can miss a substantial amount of bony and soft tissue
injury. An MRI scan can detect soft tissue injury but may miss bony lesions
on the posterior wall of the cervical spine. Helical CT with sagittal
set 2 answers_set 2 answers.qxd 22-04-2013 19:18 Page 46

Single Best Answer MCQs in Anaesthesia

reconstruction is the best way to exclude cervical spine injury. Dynamic


fluoroscopy involves passive manipulation of the cervical spine under real-
time fluoroscopic imaging. It has a low sensitivity and a false negative rate
of 0.33%. There is no conclusive evidence for the use of dynamic
fluoroscopy as a screening test for cervical spine injury.

Further reading
1. Bonhomme V, Hans P. Management of the unstable cervical spine:
elective versus emergency cases. Current Opinions in Anesthesiology
2009; 22: 579-85.
2. Richards PJ. Cervical spine clearance: a review. Injury 2005; 36: 248-
69.

3
46

Answer: E. Tension pneumothorax.

Pneumothorax is one of the complications which can follow central venous


cannulation. Severe bronchospasm is an alternative diagnosis. The clinical
history, however, with a difficult central line insertion, is suggestive of a
tension pneumothorax. Anaphylaxis may present with similar clinical
features of tachycardia, hypotension and increased airway pressure, but is
most likely to occur at induction or following a particular drug
administration.

Further reading
1. Giacomini M, Iapichino M, Armani S, et al. How to avoid and manage
pneumothorax. Vascular Access 2006; 7: 7-14.

4 Answer: A. Intrinsic renal failure.

The history and blood/urine results indicate that this patient is in renal
failure. In intrinsic renal failure, the tubules are dysfunctional and therefore
electrolytes and water are not absorbed efficiently. This leads to dilute
urine with an osmolarity of less than 300mosmol/L and urine Na+ loss of
more than 20mmol/L.

In pre-renal failure, renal tubules still work efficiently and therefore all the
unwanted elements are excreted in the minimum possible volume in order
set 2 answers_set 2 answers.qxd 22-04-2013 19:18 Page 47

Set 2 answers

to conserve water. This leads to very concentrated urine (high specific


gravity >1.020, urine osmolarity >500). The tubules also absorb most of
the filtered Na+ and urine Na+ is <20mmol/L. Both urea and creatinine are
excreted in a small volume of urine and therefore the urine: plasma
creatinine ratio is >40 and for urea >20.

Further reading
1. Anaesthesia and intensive care A-Z. An encyclopaedia of principles
and practice, 4th ed. Yentis SM, Hirsch NP, Smith GB. Oxford, UK:
Butterworth-Heinemann, 2009: 473-4.

5 Answer: C. Nephrogenic diabetes insipidus. 47

Diabetes insipidus (DI) can be classified as central or nephrogenic.


Central DI is caused by reduced or absent synthesis or release of
antidiuretic hormone (ADH, vasopressin), whereas nephrogenic DI is
caused by the reduced responsiveness of distal tubules and collecting
ducts to ADH.

A common side effect of lithium is nephrogenic DI; about one third of


patients on long-term lithium develop diabetes insipidus. Lithium may
reduce the number of ADH-regulated water channels (aquaporin-2) in the
collecting ducts, thereby reducing water absorption in the distal tubules
with resulting polyuria and polydypsia.

Rhabdomyolysis due to associated crush injury may result in acute renal


failure and low urine output. Urine testing is positive for blood and
myoglobin. Bendrofluazide can cause a diuresis but is unlikely to cause
such a marked polyuria as this. Gentamicin, an aminoglycoside antibiotic,
can cause impaired renal function.

Further reading
1. Paw H, Slingo ME, Tinker M. Late onset of nephrogenic diabetes
inspidus following cessation of lithium therapy. Anesthesia Intensive
Care 2007; 35: 278-80.
set 2 answers_set 2 answers.qxd 22-04-2013 19:18 Page 48

Single Best Answer MCQs in Anaesthesia

6 Answer: C. Arrange for an MRI scan of the spine.

The clinical triad of fever, back pain, and neurologic deficit is suggestive of
an epidural abscess. A sequential evolution of symptoms and signs has
been described, with localised spinal pain, radicular pain and
paresthesiae, muscular weakness, sensory loss, sphincter dysfunction,
and, finally, paralysis. The incidence is extremely rare but is partly affected
by the time the epidural catheter has been in situ, and the general health
of the patient. In order to prevent permanent neurological sequelae, an
early definitive diagnosis by MRI scan and surgical decompression of the
spinal cord and drainage of the abscess is essential. Consultation with a
48 neurosurgeon or spinal surgeon should be requested when a spinal
epidural abscess is detected or strongly suspected. Increasing
neurological deficit, persistent severe pain, or persistent fever and
leukocytosis are all indications for surgery.

Further reading
1. Grewal S, Hocking G, Wildsmith JA. Epidural abscesses. British
Journal of Anaesthesia 2006; 96: 292-302.

7 Answer: E. Electromyography.

Nerve conduction studies include tests for both sensory and motor
components. The likely cause for weakness of the foot in this patient is
compression of the common peroneal nerve by the lithotomy leg holder. A
detailed history and complete neurological examination is essential prior to
any investigations.

Injury to the common peroneal nerve causes foot drop and loss of
sensation over the dorsum of the foot. Electromyography involves
recording the electrical activity in the muscle. It is useful in distinguishing
peripheral neuropathy from nerve root compression from a more central
cause and establishing the site of the lesion. The other differential
diagnosis includes a lumbar disc prolapse causing nerve root
compression for which CT and MRI scans are useful investigations.
Spontaneous foot drop in a previously healthy patient may be due to a
metabolic cause such as diabetes mellitus.
set 2 answers_set 2 answers.qxd 22-04-2013 19:18 Page 49

Set 2 answers

Further reading
1. Yigit NA, Bagbanc B, Celebi H. Drop foot after pediatric urological
surgery under general and epidural anesthesia. Anesth Analg 2006;
103: 1616.
2. Horlocker TT, Cabanela ME, Wedel DJ. Does postoperative epidural
analgesia increase the risk of peroneal nerve palsy after total knee
arthroplasty? Anesth Analg 1994; 79: 495-500.
3. Hubbert CH. Peroneal palsy after epidural analgesia. Anesth Analg
1993; 77: 405-6.
4. Hogan QH. Pathophysiology of peripheral nerve injury during regional
anesthesia. Reg Anesth Pain Med 2008; 33: 435-41.

8
49
Answer: B. ST-T wave abnormalities.

In about 50% of patients with ischaemic heart disease, the ECG may be
normal. ST-T wave abnormalities are the most commonly observed ECG
findings (65-90%). Tall R waves in V5 and V6, and deep S waves in lead
V1 indicate left ventricular hypertrophy which may be seen in 10-20% of
abnormal ECGs. Pathological Q waves account for 0.5 to 8% of ECG
abnormalities.

Further reading
1. Mittnacht A, Reich DL. Recent myocardial infarction. In: Clinical cases
in anaesthesia, 3rd ed. Reed AP, Yudkowitz FS, Eds. Philadelphia,
USA: Elsevier Churchill Livingstone, 2005; Case 3: 15-20.

9 Answer: A. Biliary tract ultrasound.

This patient is displaying the signs and symptoms of acute pancreatitis,


supported by the blood results. Biliary tract ultrasound is recommended in
the initial assessment of all cases of acute pancreatitis. It is non-invasive,
relatively inexpensive and may be performed at the bedside. The sensitivity
of this study in detecting pancreatitis is 62 to 95%. A CT scan is indicated
if the diagnosis is equivocal, to rule out alternative intra-abdominal
catastrophes, and to detect and stage regional complications such as
pancreatic necrosis. If the diagnosis is clear, it may be appropriate to delay
CT imaging for at least 48-72 hours after the onset of symptoms because
set 2 answers_set 2 answers.qxd 22-04-2013 19:18 Page 50

Single Best Answer MCQs in Anaesthesia

the full extent of pancreatic necrosis cannot be determined until this time.
A contrast-enhanced CT scan is useful to assess the severity by detecting
pancreatic necrosis and the degree of peri-pancreatic collection. A plain
X-ray of the abdomen may show a gas-filled duodenum secondary to
obstruction, but this is not a specific diagnostic test.

Further reading
1. Young SP, Thompson JP. Severe acute pancreatitis. British Journal of
Anaesthesia CEACCP 2008; 8: 125-8.

10 Answer: B. D-Dimer.
50
Fibrinolysis is an important component of disseminated intravascular
coagulation (DIC). Breakdown products of fibrin, fibrin degradation
products (FDPs) and D-Dimers are therefore elevated. The specificity of
these tests is, however, limited because other conditions such as venous
thrombo-embolism, trauma and recent surgery can lead to elevated FDPs
and D-Dimers. The ongoing consumption of coagulation factors leads to
elevated global clotting times (aPTT and PT). The diagnosis of DIC should
be based on both clinical history and laboratory tests. The laboratory tests
include FDPs, D-Dimers, fibrinogen level, PT and aPTT. D-Dimers would
not be elevated in dilutional coagulopathy.

Further reading
1. Becker JU, Wira CR. Disseminated intravascular coagulation:
differential diagnosis and workup. (http://emedicine.medscape.com/
article/779097-diagnosis).

11 Answer: C. Epidural steroid injection.

Sciatica is defined as the pain caused by compression or irritation of the


sciatic nerve or one of the five nerve roots from which it originates. The
pain is felt in the lower back, buttock, posterior aspect of the thigh, or
posterolaterally in the leg and foot. The patient may experience numbness,
muscular weakness, pins and needles or tingling. Sciatica can be caused
by various factors. If caused by lumbar disc prolapse or herniation, it will
resolve in about 90% of patients over a few weeks with no specific
set 2 answers_set 2 answers.qxd 22-04-2013 19:18 Page 51

Set 2 answers

intervention. Persistent sciatica pain can be treated with different


modalities. These include simple analgesics such as paracetamol and
NSAIDs, sodium and calcium channel blockers, tricyclic antidepressants,
opioids, physiotherapy, epidural steroids, and surgical intervention.
Evidence of effectiveness for these measures is, however, limited. Surgery
speeds up the resolution of pain; however, 2 years post-surgery, outcomes
are equivalent, so patient preference is an important factor when deciding
treatment. Long-term bed rest as a treatment in sciatica is rarely practised
conservative treatment nowadays and, commonly, patients are advised
bed rest for a few days only. In this patient, as he is a lorry driver, it would
be wise to avoid strong opioids if possible as these could affect his driving.

51
Epidural steroids are used in the treatment of sciatica with limited
effectiveness.

Further reading
1. Gregory DS, Seto CK, Wortley GC, Shugart CM. Acute lumbar disk
pain: navigating evaluation and treatment choices. Am Fam Physician
2008; 78: 835-42.

12 Answer: B. MRI scan of the cervical spine.

This patient has had a parathyroidectomy. During surgical positioning, the


neck is usually hyper-extended to improve surgical access. In a patient
with pre-existing cervical spine disease, such as disc prolapse and spinal
canal stenosis, extension manoeuvres during laryngoscopy and
positioning may damage the spinal cord leading to tetraparesis. Patients
with chronic renal failure are prone to degenerative disease of the spinal
cord. In addition, extradural deposition of amyloid is seen in patients on
long-term dialysis which can result in spinal canal stenosis.

The neurological symptoms of hypocalcaemia include numbness, a


tingling sensation in the peri-oral area and fingers and toes, muscle
cramps and carpopedal spasm (tetany). Hypocalcaemia following
parathyroidectomy usually occurs 12-24 hours after surgery. A CT scan of
the head would be useful in assessing intracranial lesions, but would not
identify lesions of the cervical spine, which would be best identified with
an MRI of the spine itself.
set 2 answers_set 2 answers.qxd 22-04-2013 19:18 Page 52

Single Best Answer MCQs in Anaesthesia

Further reading
1. Mercieri M, Paolini S, et al. Tetraplegia following parathyroidectomy in
two long-term haemodialysis patients. Anaesthesia 2009; 64: 1010-3.
2. Whiteson JH, Panaro N, et al. Tetraparesis following dental extraction:
case report and discussion of preventive measures for cervical spinal
hyperextension injury. The Journal of Spinal Cord Medicine 1997; 20:
422-5.
3. Mihai R, Farndon JR. Parathyroid disease and calcium metabolism.
British Journal of Anaesthesia 2000; 85: 29-43.

13 Answer: D. Metabolic alkalosis with acidic urine.


52
Infantile pyloric stenosis is a medical emergency requiring surgical
intervention for definitive treatment. It usually presents between the 3rd
and 5th week of life and is more common in males (M:F 4:1). Projectile
vomiting causes loss of K+, H+, Cl-, Na+ and water, leading to metabolic
alkalosis. Initially, alkaline urine is produced as excess plasma bicarbonate
is excreted. If severe volume depletion occurs with continuing vomiting,
the kidney conserves Na+ (and water) in exchange for H+ and K+ ions. This
causes ‘paradoxical’ acid urine in the presence of metabolic alkalosis.

Further reading
1. Fell D, Chelliah S. Infantile pyloric stenosis. British Journal of
Anaesthesia CEPD review 2001; 1: 85-8.

14 Answer: C. Transcranial Doppler.

Stroke is the most common major complication of carotid endarterectomy


(CEA). Embolisation of thrombus or air during manipulation is the most
likely cause of cerebral ischaemia. Transcranial Doppler measures the
blood flow velocity in the middle cerebral artery on the operative side. It is
an extremely sensitive detector of cerebral embolisation.

Electro-encephalography (EEG) is the most sensitive method for the


detection of cerebral ischaemia in the unconscious patient, demonstrated
by changes such as reduced amplitude, decreased frequency and burst
suppression. Intra-operative EEG monitoring is, however, too complex to
set 2 answers_set 2 answers.qxd 22-04-2013 19:18 Page 53

Set 2 answers

use in a theatre environment and requires specialist experience to interpret


the findings. A unilateral decrease in amplitude and an increase in latency
of somatosensory evoked potentials (SSEP) indicate ischaemia, but SSEP
are less sensitive than EEG for detecting ischaemia.

Further reading
1. Schwartz AE. Carotid endarterectomy. In: Clinical cases in
anaesthesia, 3rd ed. Reed AP, Yudkowitz FS, Eds. Philadelphia, USA:
Elsevier Churchill Livingstone, 2005; Case 19: 101-3.
2. Babikian VL, Cantelmo NL. Cerebrovascular monitoring during carotid
endarterectomy. Stroke 2000; 31: 1799-1801.

15
53
Answer: D. Pulmonary function tests to estimate
postoperative FEV1.

A joint initiative by the British Thoracic Society and the Society of


Cardiothoracic Surgeons of Great Britain and Ireland (SCTS) has produced
detailed recommendations for the selection and management of patients with
potentially operable lung cancer. According to these, the pulmonary function
of the patient undergoing pneumonectomy should be assessed as follows:

w No further respiratory function tests are required in patients


undergoing pneumonectomy if the post-bronchodilator FEV1 is >2.0L,
provided that there is no evidence of interstitial lung disease or
unexpected disability due to shortness of breath.
w
• full pulmonary function tests including estimation of the transfer
All patients with a post-bronchodilator FEV1 <2.0L should have:

• measurement of oxygen saturation on air at rest;


factor for carbon monoxide (TLCO);

• a quantitative isotope perfusion scan.


w Estimated postoperative FEV1 >40% of predicted and estimated
postoperative TLCO >40% of predicted and oxygen saturation
(SaO2) >90% on air indicates average risk.
w Estimated postoperative FEV1 <40% of predicted and estimated
postoperative TLCO <40% of predicted indicates high risk.

Further reading
1. https://www.brit-thoracic.org.uk/Portals/0/Clinical%20Information/
Lung%20Cancer/Guidelines/lungcancersurgery.pdf.
set 2 answers_set 2 answers.qxd 22-04-2013 19:18 Page 54

Single Best Answer MCQs in Anaesthesia

16 Answer: D. Morphine 100mg/kg as an intra-


venous bolus.

This child needs treatment of his pain as soon as possible. A morphine


bolus is the most appropriate as the onset time of pain relief will be shorter
than most of the other mentioned options. Intramuscular codeine is painful
and its slower onset may make it inadequate. Performing an axillary plexus
block in a distressed child is not an ideal option; it has a delayed onset and
potential for failure. Although entonox has a rapid onset of action it may be
difficult to administer and only lasts for a short time. It may also cause
dizziness, nausea and vomiting. Pain after open reduction of a fracture in
54 children can usually be adequately managed by regular paracetamol and
ibuprofen with PRN Oramorph. A morphine infusion is rarely needed and
would require a bolus prior to its administration.

Further reading
1. Webber SJ, Barker I. Paediatric anaesthetic pharmacology. British
Journal of Anaesthesia CEPD review 2003; 3: 50-3.

17 Answer: B. Intravenous saline and furosemide.

Hypercalcaemia most commonly occurs due to either malignancy or


primary hyperparathyroidism. The normal range of serum calcium levels is
2.1 to 2.5mmol/L. Approximately 40% of calcium is bound to albumin,
50% is ionised and is in the physiologically active form, and the remaining
10% forms complexes with anions. Plasma calcium is regulated by
parathyroid hormone (PTH), vitamin D and calcitonin. Calcium enters the
body through the small intestine and is eventually excreted via the kidney.
Bone can act as a storage depot. This entire system is controlled through
a feedback loop; individual hormones respond as needed to increase or
decrease the serum calcium concentration.

The initial step in the treatment of hypercalcaemia is hydration with saline;


this helps to decrease the calcium level by dilution. The expansion of
extracellular volume also increases renal calcium clearance. Loop diuretics
may be used in conjunction with intravenous hydration to increase calcium
excretion. This may also prevent volume overload during the therapy. The
rate of fluid therapy depends on the degree of hypercalcaemia, severity of
set 2 answers_set 2 answers.qxd 22-04-2013 19:18 Page 55

Set 2 answers

dehydration, and the ability of the patient to tolerate rehydration.


Haemodialysis may be necessary to correct hypercalcaemia in patients
with renal failure.

Intravenous bisphosphates and calcitonin should be considered for the


management of symptomatic hypercalcaemia. Corticosteroids tend to be
more effective in patients with hypercalcaemia due to myeloma,
sarcoidosis and vitamin D excess.

Further reading
1. Ariyan CE, Sosa JA. Assessment and management of patients with
abnormal calcium. Crit Care Med 2004; 32: S146-54.
55
2. Mihai R, Farndon JR. Parathyroid disease and calcium metabolism.
British Journal of Anaesthesia 2000; 85: 29-43.

18 Answer: D. Intravenous magnesium 2g.

Torsade de pointes is a type of ventricular tachycardia in which the


morphology of the QRS complexes varies from beat to beat. The
ventricular rate can vary between 150 to 250 beats per minute. The
definition also requires that the QT interval be increased markedly (usually
to 600msec or greater). The underlying basis for rhythm disturbance is a
delay in phase III of the action potential.

Prolongation of the QT interval may be congenital (e.g. Romano Ward


syndrome). The acquired conditions that predispose to Torsade de pointes
are hypokalaemia, hypomagnesaemia, cirrhosis and hypothyroidism. Some
drugs can prolong the QT interval; these include phenothiazines, tricyclic
antidepressants, lithium carbonate, cisapride, highly active antiretrovirals,
some fluoroquinolones, and any other drugs using the CYP3A metabolic
pathway. Risk factors for this condition are female gender, history of
syncope, congenital deafness and a family history of sudden death.

Magnesium is the treatment of choice for Torsade de pointes and is


effective even in a patient with a normal magnesium level. Other therapies
include overdrive pacing and isoprenaline infusion. A patient with unstable
blood pressure should be treated with electrical cardioversion or
defibrillation. Anecdotal evidence exists for successful conversion with
phenytoin and lidocaine.
set 2 answers_set 2 answers.qxd 22-04-2013 19:18 Page 56

Single Best Answer MCQs in Anaesthesia

Further reading
1. Roden DM. A practical approach to Torsade de pointes. Clin Cardiol
1997; 20: 285-90.
2. Schenck JB, Rizvi AA, Lin T. Severe primary hypothyroidism
manifesting with Torsades de pointes. Am J Med Sci 2006; 331: 154-
6.

19 Answer: B. Oxygen supplementation with re-


warming using warm intravenous fluid and
warming blankets.

56 The body’s core temperature is closely regulated between 37.2°C +/-


0.4°C. The body maintains a stable core temperature by balancing heat
production and heat loss. The hypothalamus controls thermoregulation via
increased heat conservation (peripheral vasoconstriction, and behavioural
responses) and heat production (shivering, and increasing levels of
thyroxine and epinephrine). Depending on body temperature, hypothermia
can be graded as mild hypothermia (32-35°C), moderate hypothermia (28-
32°C), and severe hypothermia (<28°C).

Treatment depends on the degree of hypothermia. Patients with mild


hypothermia may be rewarmed using commonly available measures (warm
I.V. fluids, warm blankets, removal of cold, wet clothing). Since the risk of
cardiac dysrhythmia is low, surface rewarming is adequate. In severe
hypothermia, the risk of cardiac arrhythmias is high and the treatment is
aimed at maintaining or restoring cardiac perfusion and maximising
oxygenation until the core temperature is at least 30°C.

Further reading
1. Polderman KH. Mechanisms of action, physiological effects, and
complications of hypothermia. Crit Care Med 2009; 37: S186-202.
2. Management of inadvertent hypothermia, NICE clinical guideline.
(http://www.nice.org.uk/CG65).
3. Kirkbride DA, Buggy DJ. Thermoregulation and mild peri-operative
hypothermia. British Journal of Anaesthesia CEPD review 2003; 3: 24-
8.
4. Kirkpatrick AW, Chun R, Brown R, et al. Hypothermia and the trauma
patient. Canadian Journal of Surgery 1999; 42: 333-43.
set 2 answers_set 2 answers.qxd 22-04-2013 19:18 Page 57

Set 2 answers

20 Answer: C. Somatosensory evoked potentials.

The ‘wake-up’ test involves reducing the depth of anaesthesia to a plane


where the patient can respond to simple commands in order to assess
motor function. The anaesthetic is then deepened to allow completion of
the surgical procedure. Disadvantages include coughing, bucking and the
risk of accidental extubation during this wake-up period.

The bispectral index monitors the depth of anaesthesia. It is not specific to


this surgical procedure but is useful in situations where awareness is a
significant risk. Invasive blood pressure monitoring and peripheral nerve
57
stimulation are the essential components of monitoring during the above
procedure but are not specific to the detection of nerve injury.

Continuous intra-operative neurophysiological monitoring includes


somatosensory evoked potentials (SSEP) and motor evoked potentials
(MEP). SSEP involves stimulating peripheral nerves and detecting the
response with epidural or scalp electrodes. Decreased amplitude and
increased latency indicate nerve injury. SSEPs are useful to avoid spinal
cord damage during insertion of Harrington rods, a specific complication
related to this operation. MEP involves using a transcranial impulse to
stimulate the motor cortex; the resulting signal is detected as compound
muscle action potentials. MEPs are more challenging to apply and often
require the use of TIVA with minimal or no muscle relaxants. Inhalational
and intravenous anaesthetic agents can cause a significant decrease in
the amplitude or latency of MEPs. A combination of an opioid, such as a
remifentanil infusion, and a low dose of propofol is a particularly
successful technique during the monitoring of MEPs.

Further reading
1. Osborn IP, Spine surgery. In: Clinical cases in anaesthesia, 3rd ed.
Reed AP, Yudkowitz FS, Eds. Philadelphia, USA: Elsevier Churchill
Livingstone, 2005; Case 21: 109-22.
2. Entwistle MA, Patel D. Scoliosis surgery in children. British Journal of
Anaesthesia CEACCP 2006; 6: 13-6.
set 2 answers_set 2 answers.qxd 22-04-2013 19:18 Page 58

Single Best Answer MCQs in Anaesthesia

21 Answer: A. Complex regional pain syndrome


type I.

Complex regional pain syndrome (CRPS) type I consists of continuous


pain in the extremity following trauma, including fractures. The pain does
not, however, correspond to a particular dermatome. The pain is
increased with movement and patients often complain of cool, clammy
skin which later becomes pale, cold, stiff and atrophied. Many patients
with CRPS will exhibit some type of movement disorder ranging from
reduced strength to tremor, myoclonus and dystonia.

58 CRPS II consists of pain and the symptoms described above in the


distribution of a partially damaged peripheral nerve. The clinical features
and management of CRPS I and CRPS II are similar. The only difference
between the two types is the presence of a definable nerve injury in
CRPS II.

The International Association for the Study of Pain (IASP) lists the following
diagnostic criteria for complex regional pain syndrome I (CRPS I):

w The presence of an initiating noxious event or a cause of


immobilization.
w Continuing pain, allodynia (perception of pain from a non-painful
stimulus), or hyperalgesia disproportionate to the initiating event.
w Evidence of changes in skin blood flow, or abnormal sudomotor
activity in the area of pain.
w The diagnosis is excluded by the existence of any condition that
would otherwise account for the degree of pain and dysfunction.

Further reading
1. Wilson JG, Serpell MG. Complex regional pain syndrome. British
Journal of Anaesthesia CEACCP 2007; 7: 51-4.

22 Answer: B. Haemophilia A.

Haemophilia A is the most common hereditary coagulation disorder. It is


due to deficiency of Factor VIII and primarily affects males. Haemophilia A
set 2 answers_set 2 answers.qxd 22-04-2013 19:18 Page 59

Set 2 answers

may present with haemarthrosis, bruising occurring easily, and potentially


fatal haemorrhage following trauma and surgery. PT and platelet counts
are normal but the aPTT is prolonged. Patients with severe haemophilia
have Factor VIII levels less than 1% of normal.

von Willebrand’s disease occurs due to the deficiency of von Willebrand


Factor, which is necessary for the adherence of platelets to the exposed
endothelium. The bleeding time is prolonged but the platelet count is
normal. The clinical presentation includes bruising readily and bleeding
from the mucosal surfaces (epistaxis).

59
In Factor V deficiency, PT, PTT and bleeding times are all prolonged. The
bleeding is most often from the mucosal membranes.

Afibrinogenaemia is due to congenital absence of fibrinogen. Coagulation


tests such as bleeding time, PT, PTT and thrombin time are usually
prolonged.

Further reading
1. Coagulopathies. In: Anesthesia and co-existing disease, 4th ed.
Stoelting RK, Dierdorf SF. Philadelphia, USA: Churchill Livingstone,
2002; 489-504.

23 Answer: A. Pulmonary embolism.

This patient is obese and suffers with immobility due to paraparesis; both
are risk factors for pulmonary embolism. The causes of desaturation
should be systematically considered starting from the anaesthetic machine
all the way to the lungs. Once the oxygen delivery, integrity of the breathing
system and airway patency are confirmed, any cause arising in the cardio-
respiratory system, such as bronchospasm, pneumothorax, and
endobronchial intubation should be diagnosed and corrected. In this
scenario desaturation is associated with a low EtCO2. Other causes of
low EtCO2, such as breathing system disconnection, a leak in the gas
sampling line and hyperventilation, should be ruled out.
set 2 answers_set 2 answers.qxd 22-04-2013 19:18 Page 60

Single Best Answer MCQs in Anaesthesia

Clinical examination has ruled out the two common causes,


bronchospasm and endobronchial intubation, and also confirmed the
correct placement of the endotracheal tube. The most likely cause is
ventilation and perfusion mismatch, which would be a pulmonary
embolism.

Further reading
1. Riedel M. Diagnosing pulmonary embolism. Postgraduate Medical
Journal 2004; 80: 309-19.
2. van Beek EJR, Elliot CA, Kiely DG. Diagnosis and initial treatment of
patients with suspected pulmonary thromboembolism. British Journal

60
of Anaesthesia CEACCP 2009; 9(4): 119-24.

24 Answer: D. Oxygen via a face mask with a


reservoir bag at 15L/min.

Although magnesium sulphate is the treatment of choice for eclampsia, it


is most important to administer oxygen immediately before implementing
other therapy. Do not leave the woman alone but call for help, including a
senior obstetrician. Ensure that it is safe to approach the woman and aim
to prevent maternal injury during the convulsion. Place the woman in the
left lateral position and administer oxygen. Assess the airway and
breathing and check the pulse and blood pressure. Benzodiazepines and
phenytoin should no longer be used as first-line drugs. A loading dose of
4g of magnesium should be administered by an infusion pump over 5-10
minutes, followed by a further infusion of 1g/hour, maintained for 24 hours
following the most recent seizure. Recurrent seizures should be treated
with a further bolus of 2g of magnesium sulphate.

Further reading
1. RCOG Green-Top 10A guideline: The management of severe pre-
eclampsia/eclampsia (www.rcog.org.uk).
2. The Eclampsia Trial Collaborative Group: Which anticonvulsant for
women with eclampsia? Evidence from the Collaborative Eclampsia
Trial. Lancet 1995; 345: 1455-63.
set 2 answers_set 2 answers.qxd 22-04-2013 19:18 Page 61

Set 2 answers

25 Answer: A. Ephedrine.

This is likely to be the Bezold-Jarisch reflex (BJR), a paradoxical activation


of the left ventricular mechanoreceptors caused by reduced venous
return, due to pooling of blood in the lower extremities in the sitting
position. Ephedrine is the drug of choice. It is a sympathomimetic amine
with both a and b effects. Via its b1 effects it causes positive chronotropy
and inotropy which increases cardiac output, and through its a1 effects it
causes peripheral vasoconstriction. Phenylephrine has predominantly a1
effects causing peripheral vasoconstriction. Metaraminol is also a potent

61
vasoconstrictor. Both phenylephrine and metaraminol will increase blood
pressure and can cause reflex bradycardia. Atropine alone is not effective
in treating this condition.

BJR, with bradycardia and hypotension, may progress to cardiovascular


collapse, and is most commonly seen during surgery on the shoulder
performed under an interscalene block in the sitting position. Peripheral
vasodilatation, increased contractility from absorbed epinephrine and
vigorous contractions of a relatively empty ventricle are three components
that may contribute to BJR.

Further reading
1. Campagna JA, Carter C. Clinical relevance of the Bezold-Jarisch
reflex. Anesthesiology 2003; 98: 1250-60.
2. Reiss LWJ. Brachial plexus anesthesia. In: Clinical cases in
anaesthesia, 3rd ed. Reed AP, Yudkowitz FS, Eds. Philadelphia, USA:
Elsevier Churchill Livingstone, 2005; Case 57: 337.

26 Answer: C. Pain due to facet joint arthropathy.

Low back pain (LBP) is a common musculoskeletal complaint, with a


reported lifetime incidence of 60-90%. Various structures have been
suggested as possible sources of chronic LBP and these include the
posterior longitudinal ligament, dorsal root ganglia, dura, annular fibres,
muscles of the lumbar spine, and the facet joints. Two medial branches of
the dorsal rami innervate the facet joints. The presence of nociceptive
set 2 answers_set 2 answers.qxd 22-04-2013 19:18 Page 62

Single Best Answer MCQs in Anaesthesia

nerve fibres in the various tissue structures of facet joints suggests that
these structures may cause pain when placed under increased or
abnormal loads. Biomechanically, facet joints assume a prominent role in
resisting stress. During the rotation of the spine, the facet capsular
ligaments protect the intervertebral discs by preventing excessive
movement. There are no unique signs or symptoms identified which can
help in diagnosing the pain originating from the facet joint. However,
biomechanical studies of the facet joint during extension and rotation
support the belief that facet joint pain is worse with extension and rotation.
Lumbar facet joint pain is lateralised and can radiate to the groin and thigh.

Further reading
62 1. Manchikanti L, Boswell MV, Singh V, et al. Prevalence of facet joint
pain in chronic spinal pain of cervical, thoracic, and lumbar regions.
BMC Musculoskelet Disord 2004; 5: 15.

27 Answer: E. Perform an MRI scan of the cervical


spine.

These symptoms in a chronic rheumatoid patient with possible erosive


disease are suggestive of atlanto-axial subluxation with spinal cord
compromise. Associated symptoms include occipital headaches and
motor weakness. Direct laryngoscopy and tracheal intubation may cause
irreversible spinal cord damage if there is ligamentous instability. Elective
surgery should be delayed and further investigations organised to confirm
the diagnosis. Flexion and extension cervical spine X-rays demonstrate the
subluxation and are the most appropriate initial investigation. However, in
a patient with significant symptoms, an MRI scan provides more details,
such as nerve root compression, spinal cord compression and spinal
canal involvement. It may be necessary to arrange spinal stabilisation
surgery prior to any other elective surgery. If the patient presents for
emergency surgery, these investigations may not be possible, and the
cervical spine should be treated as unstable.

Further reading
1. Fombon F, Thompson J. Anaesthesia for the adult patient with
rheumatoid arthritis. British Journal of Anaesthesia CEACCP 2006; 6:
235-39.
set 2 answers_set 2 answers.qxd 22-04-2013 19:18 Page 63

Set 2 answers

28 Answer: E. Usual steroid dose on the morning of


surgery and hydrocortisone 50mg intravenously
at induction, followed by 50mg three times a
day by intravenous injection for 48-72 hours.

This patient is scheduled to have major elective surgery. During prolonged


therapy with corticosteroids, adrenal atrophy develops. Abrupt withdrawal
can lead to acute adrenal insufficiency. To compensate for diminished
adrenocortical response caused by prolonged corticosteroid treatment,
any significant intercurrent illness, trauma or surgical procedures, there
should be a temporary increase in the dose. The British National Formulary
recommends the following regimen for corticosteroid replacement in 63
patients who have taken more than 10mg prednisolone daily (or
equivalent) within 3 months of surgery:

w Minor surgery: usual oral corticosteroid dose on the morning of


surgery or hydrocortisone 25-50mg intravenously at induction; the
usual oral corticosteroid dose is recommenced after surgery.
w Moderate or major surgery: usual oral corticosteroid dose on the
morning of surgery and hydrocortisone 25-50mg intravenously at
induction, followed by hydrocortisone 25-50mg three times a day by
intravenous injection for 24 hours after moderate surgery or for 48-
72 hours after major surgery; the usual pre-operative oral
corticosteroid dose is recommenced on stopping hydrocortisone
injections.

Further reading
1. Joint Formulary Committee. British National Formulary, 58th ed.
London: British Medical Association and Royal Pharmaceutical
Society of Great Britain; 2009.
2. Davies M, Hardman J. Anaesthesia and adrenocortical disease. British
Journal of Anaesthesia CEACCP 2005; 5: 122-6.
3. Nicholson G, Burrin JM, Hall GM. Peri-operative steroid
supplementation. Anaesthesia 1998; 53: 1091-104.
set 2 answers_set 2 answers.qxd 22-04-2013 19:18 Page 64

Single Best Answer MCQs in Anaesthesia

29 Answer: B. An MRI scan.

Unilateral foot drop may be the only presenting symptom of conus injury
and a needle-through-needle technique for CSE technique carries a risk
of this problem occurring. A CT scan is not the best imaging technique to
look for this problem, so an MRI scan is indicated. Other pathologies
which can cause foot drop in this situation include disc prolapse and
space-occupying lesions (though other neurological signs and symptoms
are more likely to be present with the latter). If the MRI scan is normal
further management should include referral to a neurologist and possible
nerve conduction studies.
64
Further reading
1. Reynolds F. Damage to the conus medullaris following spinal
anaesthesia. Anaesthesia 2001; 56: 235-47.
2. Complications of obstetric regional anaesthesia. In: Complications of
regional anaesthesia, 2nd ed. Finucane BT, Ed. Springer, 2007:
Chapter 14.

30 Answer: D. High frequency jet ventilation.

Bronchopleural fistula can occur after a pneumonectomy due to the failure


of the bronchial stump to heal. As the bronchus is in direct connection with
the pleural cavity, some of the tidal volume is lost into the pleural cavity
affecting achievable lung ventilation. Management of a ventilated patient
with a bronchopleural fistula is particularly challenging and it is often
difficult to wean a patient from the ventilator. High frequency ventilation
with small tidal volumes, low airway pressure and a high respiratory rate
provides the best chance of ventilating the lungs in these patients if
conventional ventilation fails.

Further reading
1. Lois N, Noppen M. Bronchopleural fistulas: an overview of the
problem with special focus on endoscopic management. Chest 2005;
128: 3955-65.
set 3_set 3.qxd 22-04-2013 19:19 Page 65

Set 3 questions

Set 3
1 A 50-year-old man with Parkinson’s disease is suffering from severe
postoperative nausea and vomiting following an inguinal hernia 65
repair under general anaesthesia. The most suitable choice of anti-
emetic from this list for this patient would be:

a. Intravenous metoclopramide 10mg.


b. Intravenous dexamethasone 8mg.
c. Intramuscular prochlorperazine 12.5mg.
d. Intravenous droperidol 12.5mg.
e. Intravenous ondansetron 4mg.

2 A 64-year-old female is brought to the emergency department after


being stabbed in the groin. She is comatose, her BP is 68/34, pulse
120 bpm and she is very pale. Her haemoglobin is 4.5g/dL. There
is ST depression on a 12-lead ECG. She is wearing a bracelet
which states ‘Jehovah’s Witness’. There is no other information
available. The most appropriate management to treat her
hypovolaemia would be:

a. Start immediate cell salvage via a ‘closed circuit’.


b. Administer intravenous iron.
c. Transfuse 4 units of O-negative allogenic blood.
d. Try to contact the next-of-kin.
e. Give 500ml of Gelofusin.
set 3_set 3.qxd 22-04-2013 19:19 Page 66

Single Best Answer MCQs in Anaesthesia

3 An anaesthetist administers 10ml of 0.5% bupivacaine as part of an


epidural top-up for a Caesarean section. One minute after
completing the injection, the patient complains of dizziness, difficulty
in breathing and then starts to convulse. She then suffers a VF
cardiac arrest. The most appropriate management in the first 4
minutes would be:

a. Defibrillation, CPR, adrenaline, amiodarone.


b. Defibrillation, CPR, adrenaline, Caesarean section.
c. CPR, adrenaline, 20% Intralipid, Caesarean section.

66
d. CPR, defibrillation, 20% Intralipid, Caesarean section.
e. CPR, defibrillation, adrenaline, 20% Intralipid.

4 A 54-year-old man has suffered a flu-like illness which has lasted 3


days. Ten days later he presented with worsening bilateral leg
weakness which progressed over the next few days to involve his
upper limbs. He was admitted to the intensive care unit with a
diagnosis of Guillain-Barré syndrome. Which of the following
interventions is likely to have the best disease-modifying effect in his
case?

a. Non-invasive ventilation.
b. Corticosteroid therapy.
c. Intravenous immunoglobulin given over 5 days.
d. CSF filtration.
e. Physiotherapy.

5 A woman with HELLP syndrome requires a Caesarean section


urgently. Her platelet count 12 hours ago was 96 x 109/L and has
now dropped to 65 x 109/L. The other blood investigation results are
Hb 8.2g/dL, WBC 15 x 109/L, bilirubin 45µmol/L, ALT 150IU/L, INR
1.7, and APTT 1.9. Her BP is 140/100mmHg following treatment
with labetalol and magnesium sulphate. Which one of the following
is the best anaesthetic option?
set 3_set 3.qxd 22-04-2013 19:19 Page 67

Set 3 questions

a. Single-shot spinal anaesthesia.


b. Platelet transfusion, followed by single-shot spinal anaesthesia.
c. Epidural anaesthesia.
d. Combined spinal-epidural anaesthesia.
e. General anaesthesia with attenuation of the response to
laryngoscopy and intubation.

6 A 62-year-old female with widespread metastatic carcinoma of the


rectum is on morphine sulphate (MST) 200mg b.d., gabapentin
600mg t.d.s., paracetamol 1g q.d.s. and Oramorph 40mg 4-hourly.

67
More recently her pain control has been poor, needing an increase
in the dose of MST. Although this has relieved her pain it has caused
unacceptable side effects. Her life expectancy is about 2 years.
Which of the following would be most likely to improve her pain
control with minimal risk of side effects?

a. Epidural infusion of opioids via a tunnelled catheter.


b. Intrathecal opioid delivery via an implanted pump.
c. Transdermal fentanyl patches.
d. Sublingual buprenorphine 0.4mg added to the existing analgesia.
e. Increasing the frequency of administration of MST.

7 A 35-year-old female underwent bilateral endoscopic thoracic


sympathectomy for palmar hyperhydrosis. Which of the following
complications is most likely to occur in the postoperative period?

a. Compensatory sweating.
b. Horner’s syndrome.
c. Persisting pneumothorax.
d. Subcutaneous emphysema.
e. Haemothorax requiring drainage.

8 During a laparoscopic appendicectomy, you notice a sudden


reduction in oxygen saturation associated with a low EtCO2. Which
set 3_set 3.qxd 22-04-2013 19:19 Page 68

Single Best Answer MCQs in Anaesthesia

one of the following would provide the earliest warning of gas


embolism in this situation?

a. Use of a precordial Doppler.


b. Measurement of change in lung compliance.
c. Measurement of EtCO2.
d. Use of pulse oximetry.
e. Use of an oesophageal stethoscope.

9 A 51-year-old male is admitted to the intensive care unit with severe

68
sepsis secondary to community-acquired pneumonia. The patient is
intubated, ventilated and commenced on inotropic support. The
haematology results, 24 hours after admission show:

Hb 9.2g/dL, WCC 23.4 x 109/L, platelets 51 x 109/L, PT 23


seconds, fibrinogen 0.7g/L and raised D-dimers.

Clinical examination reveals a petechial rash all over his skin. From
the list below, the most appropriate treatment of this patient’s
haematological condition would be:

a. Warfarin.
b. Antithrombin concentrate.
c. Transexamic acid.
d. Activated protein C.
e. Tissue factor prothrombin inhibitor (TFPI).

10 A 50-year-old man is scheduled for a laparotomy and biopsy of a


mass originating from the appendix. A recent CT scan showed
evidence of liver metastases. At anaesthetic pre-assessment, he
describes a recent onset of diarrhoea and facial flushing. His GP
has also recently diagnosed asthma and has prescribed inhalers.
During the intra-operative period he suddenly becomes hypotensive
as the mass is being manipulated by the surgeon. The most
appropriate pharmacological management of this is:
set 3_set 3.qxd 22-04-2013 19:19 Page 69

Set 3 questions

a. Intravenous ketanserin.
b. Intravenous ephedrine 6mg.
c. Intravenous metaraminol 1mg.
d. Intravenous octreotide 10µg.
e. Intravenous atracurium 10mg.

11 A 52-year-male has been suffering from pain in the right groin for the
last 6 months. The pain started after hernia repair surgery and any
identifiable reversible cause for this has been ruled out. Which of the
following would be the most appropriate scale with which to assess
69
his pain?

a. Brief Pain Inventory.


b. Visual Analogue Scale.
c. Numeric Rating Scale.
d. McGill Pain Questionnaire.
e. Neuropathic Pain Scale.

12 A young male patient is admitted to a neurosurgical high


dependency unit following a fall from a ladder. He is fully conscious
and complains of pain over the posterior aspect of his neck. A CT
scan of his cervical spine has revealed a non-displaced fracture at
the level of C6-C7. Clinical examination reveals bilateral absent
biceps reflexes and reduced sensation over the shoulders. About 8
hours later his SaO2 decreases to 90% and blood gas analysis
reveals a PaO2 of 9kPa and PaCO2 of 7.5kPa. The most likely cause
of these abnormal blood gas results is:

a. Diaphragmatic paralysis.
b. Intercostal muscle paralysis.
c. Respiratory centre depression.
d. Pulmonary aspiration.
e. Morphine overdose.
set 3_set 3.qxd 22-04-2013 19:19 Page 70

Single Best Answer MCQs in Anaesthesia

13 A 59-year-old male patient has had a hemi-arthroplasty of his right


hip for a fractured neck of the femur. His past medical history
includes heavy alcohol abuse and cirrhosis of the liver. During the
intra-operative period the estimated blood loss was 1100ml and
required two units of blood transfusion. During the immediate
postoperative period his prothrombin time (PT) is noted to be 10
seconds above the control value, and the platelet count is 146 x
109/L. His haemoglobin is 9g/dL. From the list below the most
appropriate therapeutic measure to correct the abnormal clotting is
administration of:

70 a. Packed red blood cells.


b. Vitamin K.
c. Fresh frozen plasma.
d. Cryoprecipitate.
e. Platelets.

14 A 25-year-old male is involved in a high speed road traffic accident.


He has multiple injuries and is admitted to the intensive care unit
after initial surgical stabilisation. On day two he deteriorates and
requires intubation and ventilation for evolving ARDS. Later he
requires haemofiltration due to acute renal failure and also requires
inotropic support. Which of the following scoring systems will most
accurately reflect the severity of his current clinical state and
probability of mortality?

a. Multiple Organ Dysfunction Score (MODS).


b. Acute Physiology and Chronic Health Evaluation (APACHE) II
score.
c. Acute Physiology and Chronic Health Evaluation (APACHE) III
score.
d. Simplified Acute Physiology Score (SAPS).
e. Injury Severity Score (ISS).
set 3_set 3.qxd 22-04-2013 19:19 Page 71

Set 3 questions

15 A 57-year-old male patient is scheduled for a nasal septoplasty.


During the pre-operative assessment he is noted to be hypertensive.
Further questioning reveals a history of headache, palpitations and
sweating. Which of the following biochemical tests would be most
sensitive in supporting the clinical diagnosis of pheochromocytoma?

a. Serum free metanephrine.


b. Urinary metanephrine.
c. Plasma catecholamines.
d. Urinary catecholamines.

71
e. Urinary vanillylmandelic acid.

16 A 37-year-old, ASA1 female patient with a mid-humeral fracture is


scheduled for open reduction and internal fixation. She is very
concerned about postoperative pain and has agreed to have a nerve
block. Which one of the following nerve blocks is most likely to be
the most successful in relieving her pain?

a. Supraclavicular brachial plexus block.


b. Interscalene brachial plexus block.
c. Cervical plexus block.
d. Axillary brachial plexus block.
e. Infraclavicular brachial plexus block.

17 A 75-year-old male patient with cardiovascular disease is being risk


assessed prior to surgery. Which of the following independent
factors carries the greatest risk?

a. Presence of a third heart sound.


b. Age >70 years.
c. Emergency procedure.
d. ECG showing atrial fibrillation.
e. Aortic surgery planned.
set 3_set 3.qxd 22-04-2013 19:19 Page 72

Single Best Answer MCQs in Anaesthesia

18 A 35-year-old female presents with weakness of the legs. An urgent


MRI scan is organised which shows a disc prolapse at the L5/S1
vertebral level causing compression of the cauda equina. On
neurological examination, which of the following clinical signs is
likely to be present?

a. Extensor plantar response.


b. Brisk ankle jerks.
c. Weakness of hip flexion.
d. Peri-anal numbness.
e. Reduced knee jerks.
72

19 A 34-year-old woman presents to the emergency department


following a road accident. She is conscious with a clear airway and
adequate breathing. A CT scan shows a small splenic laceration
with blood around the spleen. There is no blood in the peritoneum.
No other injuries are identified. Following initial resuscitation with
crystalloids her blood pressure stabilises at 110/70mm Hg and her
pulse is 84/min. Her Hb is 12g/dL. What should be the next step in
her management?

a. Blood transfusion.
b. Exploratory laparotomy.
c. Splenectomy.
d. Observation and monitoring.
e. Arterial embolisation.

20 A 4-year-old girl weighing 20kg was admitted to hospital for an


elective tonsillectomy. During the procedure which was performed
uneventfully, intravenous fluids were commenced using sodium
chloride 0.18% with glucose 4% at a rate of 100ml/hour. She was
discharged to the ward in the afternoon and intravenous fluids were
continued at the same rate. The next day in the early morning she
develops a generalised tonic-clonic seizure, which is treated with
intravenous lorazepam. What should be done as a first step in her
subsequent management?
set 3_set 3.qxd 22-04-2013 19:19 Page 73

Set 3 questions

a. Measure urinary sodium level.


b. Measure plasma electrolytes.
c. Perform an urgent CT brain scan.
d. Commence a phenytoin infusion.
e. Check the plasma glucose.

21 A 14-year-old girl with sickle cell disease presents with severe chest
and abdominal pain. For the last 2 years she has suffered from
intermittent exacerbations every few weeks. During the acute
exacerbations her pain always has been severe and is affecting her
73
sleep and ability to attend school. Which one of the following would
be the most suitable analgesic to manage her pain?

a. Psychological counselling.
b. Regular paracetamol.
c. Regular diclofenac sodium.
d. Regular morphine.
e. Tunnelled thoracic epidural catheter.

22 A 33-year-old woman is admitted to the emergency department. She


is known to have brittle asthma. She is admitted with an acute
exacerbation; her peak flow is 40% of predicted and her pulse is
112 bpm. She has difficulty talking in sentences. Arterial blood gas
shows a PaCO2 of 3.9kPa. She is given oxygen, nebulised
salbutamol and ipratropium bromide, and intravenous
hydrocortisone. There is, however, no improvement. The next step in
the subsequent management should be:

a. Intravenous magnesium sulphate.


b. Intravenous co-amoxiclav.
c. Nebulised adrenaline.
d. Arrange for non-invasive ventilation on intensive care.
e. Intravenous aminophylline.
set 3_set 3.qxd 22-04-2013 19:19 Page 74

Single Best Answer MCQs in Anaesthesia

23 The results of pulmonary function tests performed on a patient


complaining of exertional breathlessness are as follows: FEV1 2.4L
(57% of predicted), FEV1/FVC ratio 0.8, DLCO reduced, DLCO/VA
reduced. [DLCO = Diffusion lung capacity for carbon monoxide; VA
= alveolar volume]. Which is the most likely diagnosis?

a. Pulmonary haemorrhage.
b. Emphysema.
c. Fibrosing alveolitis.
d. Asthma.
e. Pneumonectomy.

24 A 67-year-old gentleman without any previous medical problem was


74

admitted with community-acquired pneumonia. His condition rapidly


worsened over a few hours and he required mechanical ventilation.
The most common causative agent likely to produce his community-
acquired pneumonia is:

a. Streptococcus pneumoniae.
b. Staphylococcus aureus.
c. Haemophilus influenzae.
d. Legionella pneumophilia.
e. Mycoplasma pneumoniae.

25 A 3-year-old child has had an adenotonsillectomy. The operation


itself was uneventful with minimal blood loss. Four hours after the
procedure the child is found to be very irritable, pale, tachycardic
and is spitting blood. Which of the following would be the next step
in the management of this child?

a. Administration of I.V. morphine.


b. Suction of the mouth and pharynx.
c. Encourage the child to drink cold oral fluids.
d. The child should be returned to theatre and intubated immediately.
e. Administration of I.V. fluids and/or blood prior to induction of
anaesthesia.
set 3_set 3.qxd 22-04-2013 19:19 Page 75

Set 3 questions

26 A 62-year-old female patient is scheduled for right hip hemi-


arthroplasty. She has a history of angina, hypertension, and chronic
obstructive airway disease. She has been on home oxygen
2L/minute, 4-6 hours per day, for the last 6 months. Which one of
the following conditions would be an absolute contraindication to
spinal anaesthesia in this patient?

a. Presence of urinary tract infection.


b. History of spinal bifida occulta.
c. Previous spinal decompression at the L5-S1 level.
d. History of multiple sclerosis.
75
e. Patient refusal.

27 A 35-year-old fit young woman is undergoing an internal fixation of her


tibia under a general anaesthetic. While the surgeon is inserting the
implant she develops sudden hypotension and her EtCO2 shows a
sudden drop from 5kPa to 2kPa. What is the most likely diagnosis?

a. Anaesthetic-induced myocardial depression.


b. Hypothermia.
c. Pulmonary embolism.
d. Massive haemorrhage.
e. Pneumothorax.

28 A 23-year-old primigravida was given a spinal anaesthetic for a


Caesarean section after a prolonged labour and failure to progress.
She had not used any labour analgesia. Prior to the block she had
developed bilateral carpo-pedal spasm which resolved
spontaneously soon after the subarachnoid block was established.
The most likely diagnosis is:

a. Bupivacaine toxicity.
b. Hypocalcaemia.
c. Hypokalaemia.
d. Hypermagnesaemia.
e. Hypercapnia.
set 3_set 3.qxd 22-04-2013 19:19 Page 76

Single Best Answer MCQs in Anaesthesia

29 A young male had a splenectomy for blunt abdominal trauma 10


days ago. He now complains of upper abdominal pain worsened by
deep breathing. His temperature is 38.4°C and he has decreased
breath sounds over the left lung base. His respiratory rate is
20/minute and oxygen saturation is 99% on room air. His total white
cell count is 15 x 109/L and a chest X-ray shows atelectasis in his
lower lung field with a raised left hemi-diaphragm. Abdominal X-ray
shows a non-specific gas pattern in the bowel and an air fluid level
in the left upper quadrant. The most likely diagnosis is:

a. Diaphragmatic palsy.
76 b. Acute pancreatitis.
c. Sub-phrenic abscess.
d. Abdominal wound infection.
e. Hospital-acquired pneumonia.

30 A 2-year-old child is brought to the emergency department with


acute onset of respiratory distress, cough and stridor. The chest
appears hyperinflated on the right side with reduced movements and
breath sounds. The child is irritable with an oxygen saturation of
90% on air and a heart rate of 120/minute. What is the most likely
diagnosis?

a. Acute severe asthma.


b. Acute epiglottitis.
c. Aspiration pneumonia.
d. Foreign body aspiration.
e. Anaphylaxis.
set 3 answers_set 3 answers.qxd 22-04-2013 19:19 Page 77

Set 3

Set 3 answers
answers

1 Answer: E. Intravenous ondansetron 4mg.


77
Metoclopramide, prochlorperazine and droperidol can cause
extrapyramidal side effects and worsen the symptoms of Parkinson’s
disease. Dexamethasone given to an awake patient can cause distressing
perineal dysaesthesia. 5-HT3 receptor antagonists such as ondansetron
are the most suitable for this patient. Cyclizine and domperidone (which
do not cross the blood-brain barrier to any significant degree) are also safe
to use in Parkinson’s disease.

Further reading
1. Errington D, Severn A, Meara J. Parkinson’s disease. British Journal
of Anaesthesia CEPD reviews 2002; 2: 69-73.

2 Answer: C. Transfuse 4 units of O-negative


allogenic blood.

This patient is in haemorrhagic shock and the ST depression suggests


myocardial hypoperfusion. She requires urgent blood transfusion.
Although there is evidence that she is a Jehovah’s Witness, there is no
formal advance directive available and she is unable to express her wishes
regarding blood transfusion due to coma. She must therefore be treated
in her best interests with blood as this may be life-saving. Efforts can be
made subsequently to establish her wishes. Some Jehovah's Witnesses
may electively agree to a blood transfusion if it is clear that without this
their life is in grave danger.
set 3 answers_set 3 answers.qxd 02-05-2013 19:41 Page 78

Single Best Answer MCQs in Anaesthesia

Cell salvage is acceptable to some Jehovah’s Witnesses if the circuit is set


up such that their blood is kept in continuity with their body. Intravenous
iron can be useful in pre-operative optimisation or to treat anaemia but will
not be of use in the emergency situation.

Further reading
1. Management of Anaesthesia for Jehovah’s Witnesses, 2nd ed.
Association of Anaesthetists of Great Britain and Ireland, 2005.

3 Answer: E. CPR, defibrillation, adrenaline, 20%


Intralipid.
78
The cause of this woman’s symptoms and cardiac arrest is likely to be due
to intravascular injection of local anaesthetic (LA) leading to LA toxicity.
The principles of managing cardiac arrest associated with LA toxicity are:

w To start cardiopulmonary resuscitation (CPR) using standard


protocols.
w To manage arrhythmias using the same protocols, recognising that
they may be very refractory to treatment.
w Prolonged resuscitation may be necessary; it may be appropriate to
consider other options (cardiopulmonary bypass and continuing
treatment with 20% Intralipid).

CPR should be continued throughout the treatment with the 20%


Intralipid. After 4 minutes, preparations to perform an immediate
Caesarean section should commence, aiming to deliver the foetus 5
minutes after cardiac arrest has occurred. A gravid uterus causes
aortocaval compression which will only hamper resuscitation efforts;
emptying the uterus affords the mother the best chance of survival.

Further reading
1. Guidelines for the management of severe local anaesthetic toxicity.
Association of Anaesthetists of Great Britain and Ireland, 2007.
2. Cardiopulmoary resuscitation in the non-pregnant and pregnant
woman. In: Managing obstetric emergencies and trauma course
manual, 2nd ed. Grady K, Howell C, Cox C, Eds. Advanced Life
Support Group 2007; Chapter 4; 21-9.
set 3 answers_set 3 answers.qxd 02-05-2013 19:41 Page 79

Set 3 answers

4 Answer: C. Intravenous immunoglobulin given


over 5 days.

The two currently recognised disease-modifying modalities are


intravenous immunoglobulin therapy and plasma exchange. Non-invasive
ventilation is unlikely to be of benefit in a situation where clearance of
secretions is a problem. Despite the anti-inflammatory actions of
corticosteroids, there is no convincing evidence for the use of steroids.
Pain killers, physiotherapy and mechanical ventilation will form a part of
supportive management. Intravenous immunoglobulin therapy is easier to
administer and has a similar efficacy to plasma exchange.
79
Further reading
1. Richards KJC, Cohen AT. Guillain-Barré syndrome. British Journal of
Anaesthesia CEPD Reviews 2003; 3: 46-9.
2. Pritchard J. What’s new in Guillain-Barré syndrome? Postgrad Med J
2008; 84: 532-8.

5 Answer: E. General anaesthesia with


attenuation of the response to laryngoscopy
and intubation.

The platelet count has decreased by one third in the last 12 hours, which
is quite a precipitous fall. There is no test for platelet function, however,
and clotting results are abnormally prolonged (INR and APTT are >1.5).
The systolic blood pressure is adequately controlled. Given the clotting
and platelet abnormalities, and the fact that the platelet count often drops
in the 48 hours following delivery in haemolysis, elevated liver enzymes
and low platelets (HELLP) syndrome, the best choice in this situation
would be general anaesthesia, with attenuation of the response to
laryngoscopy and intubation. This may be done by administering any of the
following drugs prior to laryngoscopy:

w Alfentanil 20-30mg/kg.
w Magnesium sulphate 40mg/kg.
w Labetalol 0.25mg/kg.
w Esmolol 0.5mg/kg.
w Remifentanil 0.5mg/kg.
set 3 answers_set 3 answers.qxd 02-05-2013 19:41 Page 80

Single Best Answer MCQs in Anaesthesia

Further reading
1. HELLP syndrome. In: Analgesia, anaesthesia and pregnancy – a
practical guide, 2nd ed, Yentis S, May A, Malhotra S, Eds.
Cambridge, UK: Cambridge University Press, 2007; 80: 187-9.
2. The use of neuraxial anesthesia in parturients with thrombocytopenia:
what is an adequate platelet count? In: Evidence-based obstetric
anesthesia. Halpern SJ, Douglas M, Eds. London: BMJ Books, 2005.

6 Answer: B. Intrathecal opioid delivery via an


implanted pump.

80 Drug toxicity and fear of drug toxicity are the leading causes of failure of
cancer pain therapy. About 10% of cancer patients have refractory pain
and require advanced techniques such as adjunct medications, nerve
blocks, or an intrathecal implantable drug delivery system (IDDS).
Systemic drugs relieve pain but often have serious side effects including
sedation, confusion, constipation, or fatigue. These symptoms can be
severe enough to limit the increment in the dose to an adequate level.

IDDSs deliver small doses of morphine directly into the cerebrospinal fluid
(CSF), achieving pain relief with much smaller doses (1/300th of the oral
dose). As the dose required is only a fraction of the oral or parenteral
dose, side effects are significantly less. The IDDS consists of a small,
battery-powered, programmable pump that is implanted under the skin of
the abdomen and connected to a small intrathecal catheter.

In this patient a limiting factor in achieving satisfactory pain relief is the side
effects of opioids, and this can be best addressed by an implantable
IDDS. An epidural infusion would have a higher risk of infection and the
dose required would be ten times higher than the intrathecal dose. All
other mentioned options would reduce the dose and therefore not reduce
the side effects.

Further reading
1. Smith TJ, et al. Randomized clinical trial of an implantable drug delivery
system compared with comprehensive medical management for
refractory cancer pain: impact on pain, drug-related toxicity, and
survival. Journal of Clinical Oncology 2002; 20: 4040-9.
set 3 answers_set 3 answers.qxd 02-05-2013 19:41 Page 81

Set 3 answers

7 Answer: A. Compensatory sweating.

The indications for endoscopic thoracic sympathectomy include


craniofacial hyperhydrosis, facial blushing, chronic regional pain
syndromes, angina pectoris and congenital long QT syndrome. To treat
palmar hydrosis the sympathetic chain is divided at the T2-4 level. The
most common anaesthesia-related intra-operative complication is hypoxia
and the most serious intra-operative surgical complication is injury to the
subclavian vessels. The approximate incidence of postoperative
complications is as follows:

w Compensatory sweating (back, trunk and thigh): 50-67%. 81


w Gustatory sweating: 28-47%.
w Persistent pneumothorax: 2-4%.
w Complete Horner’s syndrome: 0.1.3%.
w Subcutaneous emphysema: 0-2%.
w Haemothorax requiring drainage: 0-2%.

Further reading
1. Martin A, Telford R. Anaesthesia for endoscopic thoracic
sympathectomy. British Journal of Anaesthesia CEACCP 2009; 9: 52-
5.

8 Answer A. Use of a precordial Doppler.

Venous gas embolism is a serious complication of laparoscopic surgery


and is more common if excessive inflation pressures are used. Precordial
Doppler ultrasound is the most sensitive method of detecting venous gas
embolism. Unfortunately, it is not quantitative and does not differentiate
between massive and physiologically insignificant embolism.
Transoesophageal echocardiography allows determination of the volume
of air, but is invasive and difficult to place. Practically, EtCO2 is the most
useful monitor as it is routinely available and sensitive. An oesophageal
stethoscope may indicate the presence of a large embolus if a mill-wheel
murmur is heard. There should be no change in pulmonary compliance
when pulmonary embolism occurs.
set 3 answers_set 3 answers.qxd 02-05-2013 19:41 Page 82

Single Best Answer MCQs in Anaesthesia

Further reading
1. Webber S, Andrzejowski J, Francis G. Gas embolism in anaesthesia.
British Journal of Anaesthesia CEPD Reviews 2002; 2: 53-7.

9 Answer: D. Activated protein C.

This patient meets the criteria for overt disseminated intravascular


coagulation (DIC). The cornerstone of management is treatment of the
underlying condition. Treatment with blood products (platelets, FFP,
cryoprecipitate) should not be based solely on laboratory results and is
better reserved for use in patients with evidence of bleeding.
82
Activated protein C should be considered in patients with severe sepsis
and DIC. Manufacturer guidance advises against its use if the platelet
count is <30 x 109/L or if there is a high risk of bleeding. Warfarin would
promote bleeding and is not appropriate. In general, transexamic acid and
antithrombin concentrate are not recommended. There is no evidence that
TFPI increases survival rate.

Further reading
1. Levi M, Toh C, Thachil J, Watson H. Guidelines for the diagnosis and
management of disseminated intravascular coagulation. British Journal
of Haematology 2009; 145: 24-33.

10 Answer: D. Intravenous octreotide 10mg.

This patient has features indicative of carcinoid syndrome from a carcinoid


tumour of the appendix. Flushing, diarrhoea and bronchospasm result
from the systemic release of vasoactive amines. Mediators are
metabolised in the liver via portal drainage; therefore, only when there are
liver metastases will carcinoid syndrome result. Large swings in blood
pressure can be seen when the tumour is manipulated. There is an
exaggerated response to exogenous catecholamines. Histamine-releasing
drugs should be avoided. Octreotide, a somatostatin analogue, prevents
the release of vasoactive mediators from the tumour and is the best
treatment of sudden severe hypotension. It suppresses the secretion of
pituitary growth hormone (GH) and thyrotropin, and decreases the release
set 3 answers_set 3 answers.qxd 02-05-2013 19:41 Page 83

Set 3 answers

of a variety of pancreatic islet cell hormones including insulin, glucagon,


and vasoactive intestinal peptide (VIP).

Ketanserin is a 5-hydroxy-tryptamine (5-HT2) type 2 receptor antagonist,


which also has some a-adrenoreceptor blocking effect. It has been used
for the treatment of intra-operative hypertensive episodes in patients with
carcinoid syndrome.

Further reading
1. Chinniah S, French J, Levy D. Serotonin and anaesthesia. British
Journal of Anaesthesia CEACCP 2008; 8: 43-5.

83
2. Houghton K, Carter JA. Peri-operative management of carcinoid
syndrome. Anaesthesia 1986; 41: 596-9.

11 Answer: D. McGill Pain Questionnaire.

This patient has post-surgical chronic pain. In the majority of cases,


patients and relatives are fearful of the cause, prognosis, and treatment,
and there may be an impact on the patient’s work, family life and social life.
Psychological factors can therefore play a larger part in the presentation.
For these reasons, multidimensional assessment tools are more commonly
used in the chronic than in the acute setting. Common multidimensional
pain scales are the McGill Pain Questionnaire and the Brief Pain Inventory.
These scales assess pain intensity as well as mood, behaviours, thoughts
and beliefs, physiological effects and their interaction with each other. The
McGill Pain Questionnaire is more comprehensive than the Brief Pain
Inventory. The McGill Pain Questionnaire has been validated for the
assessment of sensory, affective and evaluative aspects of chronic pain. It
has also been used in research studies related to chronic pain. Visual
analogue and numeric rating scales are designed to assess acute pain.
The Neuropathic Pain Scale is designed to diagnose neuropathic pain and
is not a multidimensional pain scale.

Further reading
1. Bruce J, Poobalan AS, Smith WC, Chambers WA. Quantitative
assessment of chronic postsurgical pain using the McGill Pain
Questionnaire. Clin J Pain 2004; 20: 70-5.
set 3 answers_set 3 answers.qxd 02-05-2013 19:41 Page 84

Single Best Answer MCQs in Anaesthesia

12 Answer: A. Diaphragmatic paralysis.

The blood gas result shows hypoxia and hypercapnia suggestive of


ventilatory failure. This patient has a cervical spine injury below the level of
the phrenic nerve and above the level of the cardiac sympathetic fibres.
Although injury at this level can cause paralysis of intercostal and
abdominal muscles, the patient is able to maintain adequate minute
ventilation via the diaphragm and accessory muscles. Several hours later
oedema of the spinal cord may result in ascending injury causing
diaphragmatic paralysis.

Further reading
84
1. Kang L, Tao-Chen L, Cheng-Loong L, et al. Delayed apnea in patients
with mid- to lower cervical spinal cord injury. Spine 2000; 25: 1332-8.

13 Answer: C. Fresh frozen plasma.

As this patient has cirrhosis of the liver, he is likely to have deranged


coagulation secondary to decreased synthesis of clotting factors in the
liver, leading to an increased PT. An increase in the PT by only 2 seconds
above the control values is likely to cause delayed coagulation. The
haematological investigations reveal adequate levels of haemoglobin and
platelets. Vitamin K administration helps in the synthesis of vitamin K
dependent clotting factors, such as Factors II, VII, IX and X, but the effect
is not immediate, taking at least 12-24 hours to work; it is also less
effective in the presence of hepatic insufficiency.

The platelet count is within normal limits; therefore, platelet transfusion is


not required for this patient. The low fibrinogen level can be corrected by
transfusing cryoprecipitate. One adult dose of cryoprecipitate contains
3.2-4g of fibrinogen in a volume of 150-200ml. Cryoprecipitate also
contains Factors VIII, XIII and von Willebrand Factor.

Further reading
1. Feierman DE, Gaberielson GV. Liver disease. In: Clinical cases in
anaesthesia, 3rd ed. Reed AP, Yudkowitz FS, Eds. Philadelphia, USA:
Elsevier Churchill Livingstone, 2005; Case 35: 181-93.
set 3 answers_set 3 answers.qxd 02-05-2013 19:41 Page 85

Set 3 answers

14 Answer: A. Multiple Organ Dysfunction Score


(MODS).

APACHE and SAPS are scores calculated on the first day of ICU
admission only. In this case, these scores would have been relatively low,
since on day 1 the patient was relatively well. Other scoring systems are
repetitive and collect data sequentially throughout the duration of the ICU
stay and, hence, would give worsening scores should the patient
deteriorate, e.g. MODS, Sequential Organ Failure Assessment (SOFA)
and Organ System Failure (OSF). ISS is an anatomical scoring system
based on the location and types of injury following trauma.
85
Further reading
1. Bouch D, Thompson J. Severity scoring systems in the critically ill.
British Journal of Anaesthesia CEACCP 2008; 8: 181-5.

15 Answer: A. Serum free metanephrine.

Plasma free metanephrines provide the best test for the diagnosis of
pheochromocytoma. Sensitivities of plasma free metanephrines is 99%
and urinary fractionated metanephrines (97%) are higher than those for
plasma catecholamines (84%) and urinary catecholamines (86%).
Specificity is highest for urinary vanillylmandelic acid (95%) and urinary
total metanephrines (93%).

Further reading
1. Lenders JW, Pacak K, et al. Biochemical diagnosis of
phaeochromocytoma: which test is best? JAMA 2002; 287: 1427-34.

16 Answer: A. Supraclavicular brachial plexus block.

The brachial plexus is formed by C5, C6, C7, C8, and T1 nerve roots. In
the supraclavicular region, the brachial plexus is most compact as at this
level roots join to form the trunks. Blockade at this level has the greatest
likelihood of blocking all the branches of the plexus and has a high
success rate. The apex of the lung is just medial and posterior to the
set 3 answers_set 3 answers.qxd 02-05-2013 19:41 Page 86

Single Best Answer MCQs in Anaesthesia

brachial plexus at this level and there is a relatively high risk of


pneumothorax occurring. To minimize this risk, the needle should not be
directed too medially.

The interscalene approach is also indicated for shoulder surgery. It is the


most proximal approach to the brachial plexus, a paravertebral approach
at the level of the cervical roots in the neck. The areas supplied by C8
and T1 nerve roots may prove difficult to block and this approach is
therefore less suitable for surgery involving the area supplied by the
ulnar nerve.

Further reading
86 1. Neal JM, et al. Brachial plexus anaesthesia: essentials of our current
understanding. Reg Anesth Pain Med 2002; 27: 402-28.
2. Pinnock CA, Fischer HBJ, Jones RP. Peripheral nerve blockade.
Edinburgh: Churchill Livingstone, 1996.

17 Answer: A. Presence of a third heart sound.

The Goldman Risk Index uses nine independent risk factors which are
evaluated on a point scale:

w Third heart sound (S3): 11 (as this is a sign of left ventricular


dysfunction).
w Elevated jugular venous pressure: 11.
w Myocardial infarction in the past 6 months: 10.
w ECG: premature atrial contractions or any rhythm other than sinus: 7.
w ECG shows >5 premature ventricular contractions per minute: 7.
w Age >70 years: 5.
w Emergency procedure: 4.
w Intra-thoracic, intra-abdominal or aortic surgery: 3.
w Poor general status, metabolic or bedridden: 3.

Patients with scores >25 have a 56% incidence of death, and a 22%
incidence of severe cardiovascular complications.

Patients with scores <25 but >6 have a 4% incidence of death, and a
17% incidence of severe cardiovascular complications.
set 3 answers_set 3 answers.qxd 02-05-2013 19:41 Page 87

Set 3 answers

Patients with scores <6 have a 0.2% incidence of death, and a 0.7%
incidence of severe cardiovascular complications.

Further reading
1. Goldman L, Caldera DL, Nussbaum SR, et al. Multifactorial index of
cardiac risk in noncardiac surgical procedures. N Engl J Med 1977;
297: 845-50.

18 Answer: D. Peri-anal numbness.

87
Cauda equina syndrome causes lower motor neurone signs in the lower
limbs, the precise features of which depend upon on the level of
compression. Lower motor neurone signs include: muscle wasting,
fasciculations, flaccid paralysis, absent/reduced reflexes and a flexor
plantar response. Compression of L5, S1 nerve roots can cause peri-anal
numbness (saddle anaesthesia), sensory loss along the lateral aspect of
the foot, an absent ankle jerk and loss of bowel and bladder function. Hip
flexion is a function of L2 and L3 nerve roots and the knee jerk involves L3
and L4 nerve roots.

Further reading
1. Small SA, Perron AD, Brady WJ. Orthopedic pitfalls: cauda equina
syndrome. American Journal of Emergency Medicine 2005; 23: 159-
63.

19 Answer: D. Observation and monitoring.

Isolated solid organ injury in a haemodynamically normal patient can often


be managed conservatively. Such patients must be admitted to hospital for
careful observation. Evaluation by a surgeon is essential. Concomitant
hollow viscus injury occurs in less than 5% of patients initially thought to
have isolated solid organ injuries.

Further reading
1. Abdominal trauma. In: ATLS manual, 7th ed., 2004: 141.
set 3 answers_set 3 answers.qxd 02-05-2013 19:41 Page 88

Single Best Answer MCQs in Anaesthesia

20 Answer: B. Measure plasma electrolytes.

This child has been given hypotonic fluid as a maintenance regime and this
has been administered at a high rate for a long period of time. The child
weighs 20kg, and using the ‘4-2-1’ rule she should have received fluid at
a rate of 60ml/hr. Severe hyponatraemia is the most likely explanation for
the seizure and plasma levels need to be urgently measured.
Hypoglycaemia is unlikely given the glucose-containing fluid being infused.

Hyponatraemia (serum Na <135mmol/l) may occur following surgery with


any fluid regime, particularly if hypotonic maintenance fluids are given. The
88 early signs are non-specific and often the first presenting feature is a
seizure, or even respiratory arrest. Children with hyponatraemic
encephalopathy should be managed as a medical emergency and
transferred to a paediatric intensive care unit. Hyponatraemic seizures
respond poorly to anticonvulsants and initial management is to give an
infusion of 3% sodium chloride.

Further reading
1. Association of Paediatric Anaesthetists of Great Britain and Ireland.
APA consensus guideline on perioperative fluid management in
children. London: Association of Paediatric Anaesthetists of Great
Britain and Ireland, 2007.

21 Answer: D. Regular morphine.

Pain is the most frequent problem experienced by patients with sickle cell
disease. The frequency and severity of painful episodes are highly variable
among patients. Painful episodes may start in the first year of life and
continue thereafter. The episodes last from hours to weeks followed by a
return to baseline. Dehydration, infection, stress, fatigue, menstruation and
cold can precipitate painful episodes. Medication is one of the mainstays
of treatment during the acute episode. Medication includes paracetamol,
non-steroidal anti-inflammatory drugs (NSAIDs), opioids, adjuvants such
as tricyclic antidepressants, and invasive approaches such as epidural
analgesia. Painful crises are treated symptomatically with analgesics. The
set 3 answers_set 3 answers.qxd 02-05-2013 19:41 Page 89

Set 3 answers

milder crises can be managed using NSAIDs and paracetamol, but most
patients with severe crises such as this will require opioids.

NSAIDs are often considered benign and preferable to opioids. However,


particular risks such as blood loss from occult gastritis and analgesic
nephropathy are possible from long-term use. In this patient the
exacerbations are frequent and severe, and use of regular morphine would
be indicated in order to manage the pain.

Further reading
1. Ballas SK. Current issues in sickle cell pain and its management.
Hematology American Society of Education program book, 2007: 97-
89
105.

22 Answer: A. Intravenous magnesium sulphate.

The severity of exacerbations of asthma can be assessed using the criteria


of the British Thoracic Society. A severe exacerbation can be defined as
having any one of the following features:

w PEFR 33-50% best or predicted.


w Heart rate >110 bpm or respiratory rate >25.
w Inability to complete sentences in one breath.
Pulse oximetry, arterial blood gases and chest X-ray are useful
investigations. Oxygen, nebulised b2-agonists and ipratropium bromide,
and steroids should be administered.

If there is a poor initial response to inhaled bronchodilators, a single dose


of magnesium sulphate should be given (1.2-2g intravenous infusion over
20 minutes). Routine prescription of antibiotics for acute asthma is not
indicated.

The role of intravenous aminophylline in acute asthma is now less clear.


Patients with a poor response to initial therapy may gain benefit from
treatment. Such patients are probably rare and could not be identified in a
meta-analysis of trials. Side effects of aminophylline include arrhythmias
and vomiting.
set 3 answers_set 3 answers.qxd 02-05-2013 19:41 Page 90

Single Best Answer MCQs in Anaesthesia

Hypercapnic respiratory failure developing during an acute asthmatic


episode is an indication for urgent ICU admission. It is unlikely that non-
invasive ventilation (NIV) would replace intubation in these very unstable
patients, but it has been suggested that treatment is safe and effective.

Further reading
1. Scottish Intercollegiate Guidelines Network: British guideline on the
management of asthma (SIGN guideline 101). Edinburgh: SIGN,
2009. (http://www.sign.ac.uk/pdf/sign101.pdf).

90
23 Answer: C. Fibrosing alveolitis.

Spirometry is useful in distinguishing obstructive lung disease from


restrictive lung disease. A low FEV1 with a normal/raised FEV1/FVC is
suggestive of restrictive lung disease. Lung volume measurements
showing reduced total lung capacity (TLC) are the hallmark of restrictive
disease.

In obstructive lung disease (e.g. COPD, asthma), both FEV1 and the
FEV1/FVC ratio are reduced. There may be an increase in residual volume
(RV) and TLC. This is particularly marked in emphysema.

DLCO, also known as the transfer factor of the lung for carbon monoxide,
is a measure of the diffusing capacity from alveolar gas to the red blood
cells in the pulmonary circulation. It is helpful in evaluating the presence of
possible parenchymal lung disease when spirometry and/or lung volume
determinations suggest a reduced vital capacity, RV, and/or TLC. Because
the DLCO is directly proportional to VA, non-pulmonary processes that
reduce the TLC cause reductions in the DLCO. If VA can be assessed
accurately, these reductions produce a normal or elevated DL/VA ratio,
e.g. lung resection, thoracic cage abnormalities. A reduced DLCO and a
reduced DL/VA ratio suggest a true interstitial disease such as pulmonary
fibrosis or pulmonary vascular disease.

Further reading
1. Plummer AL. The carbon monoxide diffusing capacity. Chest 2008;
134: 663-7.
set 3 answers_set 3 answers.qxd 02-05-2013 19:41 Page 91

Set 3 answers

24 Answer: A. Streptococcus pneumoniae.

Although all of the above mentioned agents can produce community-


acquired pneumonia, the most common agent causing it is
Streptococcus pneumoniae. Patients suffering from severe community-
acquired pneumonia are more likely to suffer secondary infections by
other organisms, for example, S. aureus, P. aeruginosa and other gram-
negative bacilli.

Further reading
1. Lim WS, et al. BTS guidelines for the management of community-
acquired pneumonia in adults: update 2009. Thorax 2009; 64 (Suppl 91
III): iii1-55

25 Answer: E. Administration of I.V. fluids and/or


blood prior to induction of anaesthesia.

This child is showing signs of significant post-tonsillectomy


haemorrhage. The usual cause of blood loss is venous or capillary ooze
from the tonsillar bed and it is difficult to measure, as it occurs over
several hours and is partly swallowed. Excessive blood loss may lead to
the child spitting blood. In these cases, the child is likely to be seriously
hypovolaemic, anaemic and potentially difficult to intubate because of
poor visualization of the larynx. Pre-operative resuscitation (guided by
trends in monitoring) is essential, as induction of anaesthesia in a
hypovolaemic child can precipitate cardiovascular collapse. Anaesthesia
is induced once the child is haemodynamically stable. Preoxygenation
and rapid sequence induction with a slight head-down positioning of the
patient ensures rapid control of the airway and protection from
pulmonary aspiration.

Further reading
1. Ravi R, Howell T. Anaesthesia for paediatric ear, nose, and throat
surgery. British Journal of Anaesthesia CEACCP 2007; 7: 33-7.
set 3 answers_set 3 answers.qxd 02-05-2013 19:41 Page 92

Single Best Answer MCQs in Anaesthesia

26 Answer: E. Patient refusal.

If patients have the capacity to make decisions for themselves, patient


refusal is an absolute contraindication to any procedure. The basic
principles of consent in a patient with capacity are:

w The doctor uses specialist knowledge and experience to make a


clinical judgement, and taking the patient’s views and understanding
of their condition, identifies which investigations or treatments are
likely to result in overall benefit for the patient. The doctor explains the
options to the patient, setting out the potential benefits, risks, burdens
92 and side effects of each option, including the option of having no
treatment. The doctor may recommend a particular option which they
believe to be best for the patient, but they must not put undue
pressure on the patient to accept their advice.
w The patient weighs up the potential benefits and risks of the various
options as well as any non-clinical issues that are relevant to them,
then decides whether to accept any of the options and, if so, which
one. They also have the right to accept or refuse an option for a reason
that may seem irrational to the doctor or for no reason at all.

In all of the other mentioned conditions, spinal anaesthesia can be


performed with care. Severe coagulopathy and severe sepsis are also
absolute contraindications to neuraxial anaesthesia.

Further reading
1. General Medical Council. Consent guidance: patients and doctors
making decisions together, 2008. (http://www.gmc-uk.org/guidance
/ethical_guidance/consent_guidance_index.asp).
2. O’Rourke N, Khan K, Hepner DL. Contraindications to neuraxial
anaesthesia. In: Spinal and epidural anesthesia. Wong CA, Ed. USA:
McGraw Hill Medical, 2007; Chapter 5: 127-50.

27 Answer: C. Pulmonary embolism.

The increased intra-medullary pressure during nailing of long bones can


lead to fat embolism. Apart from haemodynamic changes there may be
set 3 answers_set 3 answers.qxd 02-05-2013 19:41 Page 93

Set 3 answers

hypoxia, pulmonary oedema, development of a coagulation disorder and


CNS changes leading to convulsions or coma if the patient is awake. The
sudden drop in EtCO2 in this case is due to an increase in the alveolar
dead space.

Further reading
1. Wiersema UF. Chest Injuries. In: Oh’s intensive care manual, 6th ed.
Bersten A, Soni N, Eds. Elsevier, 2009: 800.

28 Answer: B. Hypocalcaemia.

Without analgesia, a labouring woman can demonstrate a very high 93


hyperventilatory response, with marked hypocapnia (PaCO2 as low as
2kPa). This can lead to an acute reduction in ionised calcium leading to
these symptoms.

Further reading
1. Ray N, Camann W. Hyperventilation-induced tetany associated with
epidural analgesia for labour. International Journal of Obstetric
Anaesthesia 2005; 14: 74-6.

29 Answer: C. Sub-phrenic abscess.

This patient shows signs of sepsis with symptoms mainly localizing to the
left upper abdomen. A sub-phrenic abscess usually produces elevation of
the left hemi-diaphragm, pleural effusion and basal atelectasis. The
abscess can be confirmed by a CT scan or ultrasound. Unilateral
diaphragmatic palsy can produce a raised hemi-diaphragm, seen on the
chest X-ray. Acute pancreatitis can present with epigastric pain, pyrexia
and shock. Usually there is a history of alcohol abuse, dyspepsia or biliary
colic. In this patient, as there is a history of recent trauma and abdominal
surgery, a sub-phrenic abscess is the most likely diagnosis. The clinical
features are also suggestive of a chest infection, but the abdominal X-ray
showing air fluid level suggests intra-abdominal pathology.
set 3 answers_set 3 answers.qxd 02-05-2013 19:41 Page 94

Single Best Answer MCQs in Anaesthesia

Further reading
1. Padley S. Imaging the chest. In: Oh’s intensive care manual, 6th ed.
Bersten A, Soni N, Eds. Elsevier, 2009: 466.

30 Answer: D. Foreign body aspiration.

Aspiration of a foreign body, commonly food, has a peak incidence at 1-2


years of age. A sudden onset of coughing, gagging, and choking suggest
foreign body aspiration and may necessitate basic life support manoeuvres
for the choking child. Partial obstruction of a lower airway may cause air

94
trapping behind the foreign body (ball and valve effect) with pneumothorax,
surgical emphysema, and pneumo-mediastinum a possibility. In this
situation, the usual inspiratory chest X-ray can appear normal; an
expiratory film, however, if it can be obtained, may reveal air trapping.

Further reading
1. Maloney E, Meakin GH. Acute stridor in children. British Journal of
Anaesthesia CEACCP 2007; 183-6.
2. Warshawsky ME. Foreign body aspiration. (http://emedicine.
medscape.com/article/298940-overview).
set 4_set 4.qxd 22-04-2013 19:20 Page 95

Set 4 questions

Set 4
1 A 60-year-old female patient is admitted to the neurosurgical unit
with a subarachnoid haemorrhage secondary to a ruptured 95
intracranial aneurysm. Which of the following is the most serious
complication that could occur during the subsequent 3 days?

a. Re-bleeding.
b. Cerebral vasospasm.
c. Hypertension.
d. Hydrocephalus.
e. Pulmonary oedema.

2 A 70-year-old male patient is undergoing an oesophagectomy via a


thoraco-abdominal approach requiring one-lung ventilation. Which
of the following pathophysiological changes is least likely to occur
during one-lung ventilation?

a. Hypercarbia.
b. Hypoxia.
c. Intrapulmonary shunt.
d. Hypoxic pulmonary vasoconstriction.
e. Ventilation perfusion mismatch.
set 4_set 4.qxd 22-04-2013 19:20 Page 96

Single Best Answer MCQs in Anaesthesia

3 A 67-year-old male patient has been anaesthetised with total


intravenous anaesthesia using propofol and remifentanil for an
inguinal hernia repair. About 10 minutes after induction his blood
pressure falls to 80/50mmHg and heart rate to 42 per minute. The
immediate treatment should be:

a. Intravenous ephedrine.
b. Intravenous metaraminol.
c. Intravenous phenylephrine.
d. Reducing the dose of propofol.
e. I.V. infusion of 1L of normal saline.
96

4 A 72-year-old man underwent abdominopelvic resection of a


carcinoma of the rectum. On the third postoperative day he
developed right-sided chest pain with no new ST-T changes on the
ECG. Clinical examination revealed a temperature of 37.6°C, SpO2
of 88% (on 15L/min of oxygen via a non-rebreathing mask),
tachypnoea, tachycardia with a pulse rate of 130 per minute and a
BP of 128/66mm Hg. Auscultation of the chest was unremarkable
and the chest X-ray was normal. An arterial blood gas while on
15L/min oxygen revealed the following: pH 7.46kPa, PaCO2
4.8kPa, PaO2 7.1kPa, lactate 1.2mmol/L and HCO3- 20mmol/L.
The most likely diagnosis is:

a. Postoperative pneumonia.
b. Bibasal atelectasis.
c. Severe sepsis.
d. Acute coronary syndrome.
e. Pulmonary embolism.

5 A 36-year-old female patient presents for emergency


appendicectomy. Two weeks ago she underwent scleral buckle
repair of a spontaneous retinal detachment of the right eye. The
most important precaution to be taken during the anaesthetic for her
appendicectomy to avoid damage to her eye is:
set 4_set 4.qxd 22-04-2013 19:20 Page 97

Set 4 questions

a. Avoiding rapid sequence induction.


b. Avoiding nitrous oxide.
c. Avoiding hypercarbia.
d. Maintaining normocarbia.
e. Avoiding hypotension.

6 A patient is undergoing a percutaneous, X-ray-guided coeliac plexus


block for the management of pain due to chronic pancreatitis. What
position should the needle tip be in to provide the most effective
block?

a. Anterior to the pancreas. 97


b. Anterior to the aorta.
c. Lateral to the inferior vena cava.
d. Posterior to the aorta.
e. Antero-lateral to the L1 vertebral body.

7 A 50-year-old patient is undergoing a posterior fossa craniotomy in


the sitting position. Anaesthesia and surgery proceed uneventfully
until approximately 2 hours into the procedure, when a sudden
decrease in EtCO2, a slight increase in heart rate and a reduction in
blood pressure are noticed. Venous air embolism is suspected. The
most sensitive clinical monitor in this situation would be:

a. Transoesophageal Doppler.
b. Precordial stethoscope.
c. EtCO2 monitor.
d. Pulse oximeter.
e. PA catheter.

8 A young ASA1 patient underwent an elective abdominal hysterectomy


under general anaesthesia with endotracheal intubation. She had no
history of acid reflux. Soon after extubation, the patient developed
severe laryngospasm which responded to intravenous propofol and
CPAP via a face mask. Despite having a clear upper airway, she
set 4_set 4.qxd 22-04-2013 19:20 Page 98

Single Best Answer MCQs in Anaesthesia

remained breathless with an oxygen saturation of 90% whilst


breathing 100% oxygen via a face mask. The oxygenation improved
over a period of 2 hours following the use of CPAP and diuretic
treatment. The most likely diagnosis in this patient is:

a. Aspiration pneumonia.
b. Negative pressure pulmonary oedema.
c. Bronchial asthma.
d. Fluid overload.
e. Congestive cardiac failure.

98 9 A 33-year-old woman is brought to the emergency department with


a suspected drug overdose. She is drowsy, with tachycardia,
hypotension and dilated pupils. Blood gases reveal a metabolic
acidosis. The ECG shows sinus tachycardia with a QRS duration of
140ms. From the following which drug is she most likely to have
taken as an overdose?

a. Phenytoin.
b. Fluoxetine.
c. Ethylene glycol.
d. Amitryptiline.
e. Amphetamine.

10 A 60-year-old male patient has undergone repair of a thoraco-


abdominal aortic aneurysm. On the third postoperative day, 8 hours
following discontinuation of epidural analgesia, he complains of an
inability to move the lower limbs. Clinical examination reveals
paraplegia with absence of cold and pain sensation with preserved
sensation of deep touch, vibration and proprioception. The most
likely diagnosis is:

a. Transverse myelitis.
b. Epidural haematoma.
c. Epidural abscess.
d. Anterior spinal artery syndrome.
e. Posterior spinal artery syndrome.
set 4_set 4.qxd 22-04-2013 19:20 Page 99

Set 4 questions

11 A 50-year-old male patient is scheduled for amputation of his great


toe under regional anaesthesia. Which one of the following groups
of nerves needs to be blocked to provide effective anaesthesia?

a. Deep peroneal and superficial nerves.


b. Deep peroneal nerve.
c. Deep peroneal and saphenous nerves.
d. Deep peroneal and superficial peroneal, and posterior tibial nerves.
e. Superficial peroneal and posterior tibial nerves.

12 A 35-year-old lady with a history of two previous Caesarean sections 99


is undergoing a third Caesarean section under spinal anaesthesia.
After the baby is delivered she develops an ongoing postpartum
haemorrhage due to uterine atony. She has already received five
units of oxytocin I.V. and 500µg ergometrine I.M. An oxytocin
infusion (40 units in 500ml) has been commenced at a rate of
125ml/hour. What would be the next choice of drug for
pharmacological management of her atonic PPH?

a. I.M. carboprost 250µg repeated at 15-minute intervals if


necessary.
b. Intra-myometrial injection of carboprost 250µg.
c. A further 10 units of I.V. oxytocin as a bolus.
d. A repeat dose of I.V. ergometrine 500µg.
e. Rectal misoprostol 400µg.

13 A young female patient with isolated moderate mitral regurgitation is


scheduled for a laparotomy and removal of an ovarian tumour.
Which of the following statements would describe the best peri-
operative haemodynamic goals for this patient?

a. Use of spinal anaesthesia to reduce the SVR.


b. Maintenance of normal SVR with a slow heart rate in order to
maintain a favourable myocardial oxygen demand-supply ratio.
set 4_set 4.qxd 22-04-2013 19:20 Page 100

Single Best Answer MCQs in Anaesthesia

c. Use of an inhalational anaesthetic in preference to intravenous


agents in order to achieve pulmonary vasodilation.
d. Maintenance of normal SVR using an I.V. infusion of metaraminol if
required.
e. Maintenance of a normal to high heart rate avoiding sudden
bradycardia, and maintenance of a normal to low SVR.

14 A 65-year-old female patient with rheumatoid arthritis presents to the


emergency department with chest pain, shortness of breath and a
low-grade fever of 3 days’ duration. Her medications include

100
prednisolone and methotrexate. Auscultation of the chest reveals a
pericardial rub. The 12-lead ECG shows diffuse ST elevation with a
heart rate of 110 per minute. The most likely diagnosis is:

a. Congestive cardiac failure.


b. Pericardial effusion.
c. Cardiac tamponade.
d. Myocardial infarction.
e. Pericarditis.

15 A 52-year-old woman presents for a right hemi-arthroplasty following


a fall and sustaining a fractured neck of the femur. She has a history
of alcoholic liver disease. On clinical examination she is fully
conscious and orientated in time and place. She is jaundiced, but
has no signs of hepatic encephalopathy, and there is no ascites.
Biochemistry investigations reveal an increased prothrombin time (5
seconds above the control), the serum bilirubin is 3mg/dL and the
serum albumin is 32g/L. Which of the following statements describe
most accurately her likely peri-operative mortality?

a. She has a low operative mortality.


b. Her operative mortality is about 25%.
c. Her operative mortality is less than 5%.
d. According to modified Child’s criteria she has a total score of 5.
e. According to modified Child’s criteria she has a total score of 10.
set 4_set 4.qxd 22-04-2013 19:20 Page 101

Set 4 questions

16 A 32-year-old male is admitted to the high dependency unit for


observation following a road traffic accident. He has sustained
fractures of the 3rd, 4th, and 5th ribs on the right side with
associated pain. You want to perform an intercostal nerve block for
the management of the pain. At which site should the block be
performed?

a. Anterior axillary line.


b. Mid-axillary line.
c. Posterior axillary line.
d. Posterior angle of the rib.
101
e. At the fracture site.

17 A 23-year-old previously fit male presents with increasing dyspnoea.


He has undergone cardiac catheterisation and the results of this
investigation are shown in Table 1.

Table 1. Results of cardiac catheterisation.

Cardiac chamber RA RV PA PA wedge LA LV Aorta

Pressure in mmHg 4 12 45 9 5 110/8 110/70

The most likely diagnosis in this patient is:

a. Mitral stenosis.
b. Mitral incompetence.
c. Aortic stenosis.
d. Idiopathic pulmonary hypertension.
e. Aortic incompetence.
set 4_set 4.qxd 22-04-2013 19:20 Page 102

Single Best Answer MCQs in Anaesthesia

18 A 27-year-old primigravida who is 37 weeks’ pregnant is admitted to


the labour ward with symptoms of headache, blurring of vision and
confusion. On examination she has a blood pressure of 190/110mm
Hg with increased reflexes, clonus and 3+ of protein in the urine.
Which one of the following is the most appropriate treatment?

a. Oral labetalol.
b. Immediate delivery.
c. Hydralazine and magnesium.
d. Continue close monitoring and initiate treatment if she deteriorates.
e. Oral methyldopa.

19 A middle-aged female patient has been admitted to the intensive


102

care unit with an acute exacerbation of asthma. On admission she


was hypoxic, hypercarbic and tachycardic with a reduced level of
consciousness. She was therefore sedated and ventilated manually
initially in order to improve her oxygenation. Over a period of a few
minutes she became hypotensive and more tachycardic and
hypoxic. The most likely cause for her deterioration following
ventilation was:

a. Pneumonia.
b. Tension pneumothorax.
c. Sepsis.
d. Hypovolaemia.
e. Myocardial infarction.

20 A 35-year-old female patient has had a laparoscopic


cholecystectomy performed under general anaesthesia. The airway
was secured using an orotracheal tube. A nasogastric tube was
inserted using Magill forceps under direct vision with the aid of a
laryngoscope to decompress the stomach. At the end of the
procedure the trachea was extubated following blind suction using a
Yankauer sucker. On transfer to the recovery room she developed
breathing difficulties, and her oxygen saturation decreased to 94%
despite 40% supplemental oxygen. No improvement was noted
set 4_set 4.qxd 22-04-2013 19:20 Page 103

Set 4 questions

despite administration of nebulised epinephrine. Examination of the


oropharynx revealed a large oedematous uvula. Which of the
following would be the most effective next step in her management?

a. Intravenous dexamethasone.
b. Intravenous hydrocortisone.
c. Saline nebulisation.
d. CPAP by face mask.
e. Sedation and tracheal intubation.

21 In which of the following conditions is chronic post-surgical pain


most commonly seen? 103

a. Cholecystectomy.
b. Mastectomy.
c. Amputation of limb.
d. Thoracotomy.
e. Hernia repair.

22 A 76-year-old female underwent a hemi-arthroplasty under general


anaesthesia. A lumbar plexus block was performed at induction to
provide postoperative analgesia. During the intra-operative period
she required a transfusion of 2 units of blood. In the recovery room
she is confused, disorientated and agitated. Her pain appears to be
adequately controlled. Her vital parameters in recovery are shown in
Table 2. 40% oxygen has been administered via a face mask.

Table 2. Vital parameters in recovery.

Pulse rate BP Respiratory rate SpO2 Temperature

86 bpm 164/88 mmHg 14 bpm 97% 36.9°C


set 4_set 4.qxd 22-04-2013 19:20 Page 104

Single Best Answer MCQs in Anaesthesia

Despite all measures it is difficult to control her agitation. The


following is the most appropriate pharmacological treatment in this
patient:

a. Midazolam 2mg I.V.


b. Haloperidol 0.5-1mg I.V.
c. Titrated doses of diazepam.
d. Ketamine 0.25mg/kg I.V.
e. Propofol 50-100mg I.V.

23 A 24-year-old woman who has been in prolonged labour is


104 undergoing an emergency Caesarean section for failure to
progress. During the procedure she loses about 3L of blood. The
most likely cause for this massive haemorrhage is:

a. Trauma to the cervix and birth canal.


b. Primary coagulopathy.
c. Disseminated intravascular coagulopathy.
d. Uterine atony.
e. Retained products of conception.

24 A 68-year-old man is admitted to the intensive care unit with severe


sepsis and acute renal failure. He is commenced on continuous
veno-venous haemodiafiltration, using unfractionated heparin as an
anticoagulant. Sometime later it is noted that his platelet count has
fallen. His platelet count on admission to the unit was 386 x 109/L.
Which of the following laboratory results makes a diagnosis of type
2 heparin-induced thrombocytopaenia (HIT) more likely?

a. Platelet count of 102 x 109/L 6 days after starting heparin.


b. Lowest platelet count of 12 x 109/L during the treatment period.
c. Detection of PF4-heparin IgA antibodies.
d. Platelet count of 297 x 109/L 2 days after stopping heparin
administration.
e. Fibrinogen level of 0.6g/L.
set 4_set 4.qxd 22-04-2013 19:20 Page 105

Set 4 questions

25 A middle-aged male patient with a history of alcohol abuse presents


to the accident and emergency department with a bout of prolonged
vomiting followed by a massive haematemesis. Clinical examination
reveals tachycardia, tachypnoea and hypotension. His body
temperature is normal. Stool examination is negative for occult
blood. The most likely diagnosis is:

a. Peptic ulcer.
b. Oesophageal stricture.
c. Reflux oesophagitis.
d. Hiatus hernia.
105
e. Mallory-Weiss syndrome.

26 A 52-year-old patient is known to have ischaemic heart disease, and has


a permanent pacemaker. He is on various cardiac medications and had
a CABG a year ago. He continues to suffer from chest pain and has
been diagnosed with refractory angina. He also suffers from obstructive
sleep apnoea and uses CPAP during the night. Which of the following
would be the most appropriate treatment to manage his chest pain?

a. TENS therapy.
b. Stellate ganglion block.
c. Regular NSAIDs.
d. Regular diamorphine.
e. Fentanyl patch.

27 A 30-year-old male patient underwent open reduction and internal


fixation of a fracture of the neck of the right humerus. Three weeks later
he presents with an inability to lift the right arm. On examination there
is reduced sensation over the lower part of the right deltoid region.
Which of the following nerves are likely to have been injured thus
causing his symptoms?

a. The axillary nerve.


b. The median nerve.
c. The suprascapular nerve.
d. The musculocutaneous nerve.
e. The radial nerve.
set 4_set 4.qxd 22-04-2013 19:20 Page 106

Single Best Answer MCQs in Anaesthesia

28 A 70-year-old man presents in the emergency department with a


history of sudden onset of severe back ache. His past medical
history includes hypertension, hypercholesterolaemia and a 5cm
abdominal aortic aneurysm diagnosed 3 years before. On clinical
examination his radial pulse is weak with a rate of 115 per minute;
his blood pressure is 84/60mm Hg. There is a pulsatile mass in the
epigastrium. The most likely diagnosis is:

a. Acute cholecystitis.
b. Acute gastritis.
c. Acute pancreatitis.
106
d. Ruptured abdominal aortic aneurysm.
e. Acute lumbar disc prolapse.

29 A 60-year-old male patient developed laryngospasm on the first


postoperative day following a total thyroidectomy for carcinoma of
the thyroid. Which of the following investigations is likely to be the
most useful in the further management of this patient?

a. Serum magnesium.
b. Nasendoscopy to assess the vocal cord function.
c. Serum calcium.
d. Serum TSH.
e. Electrocardiogram.

30 A 40-year-old male patient underwent neck dissection and excision


of a tumour on the floor of the mouth. Two weeks later he complains
of a loss of taste sensation. Which one of the following nerves is
likely to be injured during this surgery?

a. Inferior alveolar nerve.


b. Lingual nerve.
c. Glossopharyngeal nerve.
d. Buccal nerve.
e. Hypoglossal nerve.
set 4 answers_set 4 answers.qxd 22-04-2013 19:20 Page 107

Set 4

Set 4 answers
answers

1 Answer: A. Re-bleeding.
107
Although cerebral vasospasm is the most common problem, the most
serious complication of subarachnoid haemorrhage (SAH) is re-bleeding;
the occurrence is about 15% during the first week. The management of
cerebral vasospasm consists of nimodipine and triple H therapy (induced
hypertension, hypervolaemia and haemodilution). SAH is also frequently
associated with systemic and pulmonary hypertension, cardiac
arrhythmias and neurogenic pulmonary oedema. Other complications
include hydrocephalus and electrolyte disturbances. Hyponatraemia
develops as a result of either cerebral salt wasting syndrome or the
syndrome of inappropriate ADH secretion.

Further reading
1. Solenski NJ, Haley E, et al. Medical complications of aneurysmal
subarachnoid hemorrhage: a report of the multicenter, cooperative
aneurysm study. Critical Care Medicine 1995; 23: 1007-17.
2. Priebe H-J. Aneurysmal subarachnoid haemorrhage and the
anaesthetist. British Journal of Anaesthesia 2007; 99: 102-18.

2 Answer: A. Hypercarbia.

Patients undergoing oesophagectomy are positioned in the lateral


decubitus position. During surgery, once the non-dependent lung is
collapsed, ventilation to that lung is eliminated. The blood passing through
the collapsed non-dependent lung does not take part in gas exchange,
possibly resulting in hypoxia. CO2 exchange is not affected to the same
set 4 answers_set 4 answers.qxd 22-04-2013 19:20 Page 108

Single Best Answer MCQs in Anaesthesia

extent as oxygenation. The blood passing through the collapsed, non-


ventilated lung contributes to the ventilation perfusion mismatch and shunt.

Further reading
1. Neustin SM, Eisenkraft JB. One-lung anesthesia. In: Clinical cases in
anaesthesia, 3rd ed. Reed AP, Yudkowitz FS, Eds. Philadelphia, USA:
Elsevier Churchill Livingstone, 2005; Case 15: 73-84.

3 Answer: A. Intravenous ephedrine.

This patient has bradycardia with hypotension, most likely caused by


108 remifentanil. This should best respond to intravenous ephedrine and
reducing the dose of remifentanil. Speed of injection is an important factor
for bradycardia with remifentanil, commonly seen with administration of a
bolus dose. The incidence of bradycardia may be minimized by slow
administration. It is generally accepted that the bradycardia is vagally
mediated and bilateral vagotomy has been shown to abolish this effect.

Ephedrine acts both directly by causing release of noradrenaline from the


sympathetic nerve terminals, and indirectly by stimulation of a and ß
adrenoreceptors. It has positive chronotropic and inotropic effects on the
heart.

Metaraminol also has both direct and indirect sympathomimetic effects. It


stimulates both a and b adrenoreceptors, but a effects predominate
causing an increase in systemic vascular resistance. It increases both
systolic and diastolic blood pressure, and a reflex bradycardia occurs
which may further decrease the heart rate in this patient.

Phenylephrine is a selective a1 agonist, which has no chronotropic effect.


It can be used in situations where peripheral vasoconstriction is needed
and cardiac output is adequate, as in hypotension that may accompany
spinal anaesthesia.

Further reading
1. DeSouza G, Lewis MC, TerRiet MF. Severe bradycardia after
remifentanil. Anesthesiology 1997; 87: 1019-20.
set 4 answers_set 4 answers.qxd 22-04-2013 19:20 Page 109

Set 4 answers

4 Answer: E. Pulmonary embolism.

This patient has type 1 respiratory failure with severe hypoxia indicating a
significant ventilation perfusion mismatch. Pneumonia and atelectasis are
unlikely due to the absence of clinical and radiological signs. Similarly,
severe sepsis is less likely because of the normal pH and lactate. Acute
coronary syndrome complicated with left ventricular failure and pulmonary
oedema can result in hypoxia. Pulmonary embolism is the most likely
diagnosis given the background of major abdominopelvic cancer surgery
with the associated immobilisation, chest pain and unexplained severe
hypoxia.
109
Further reading
1. Tapson VF. Acute pulmonary embolism. N Engl J Med 2008; 358:
1037-52.

5 Answer: B. Avoiding nitrous oxide.

During the repair of retinal detachment, the surgeon usually uses intra-ocular
injection of air and sulfurhexafluoride (SF6). The coincident use of nitrous
oxide expands the volume of air within the eye and increases the intra-ocular
pressure. During the procedure nitrous oxide should be discontinued at least
10-15 minutes prior to intra-ocular injection of SF6. Nitrous oxide should be
avoided at least 30 days following repair of retinal detachment to prevent a
rise in intra-ocular pressure. Rapid sequence induction using succinylcholine
can cause a transient increase in intra-ocular pressure but rocuronium can
be used as an alternative to succinylcholine.

Further reading
1. Herlich A. Retinal detachment. In: Clinical cases in anaesthesia, 3rd
ed. Reed AP, Yudkowitz FS, Eds. Philadelphia, USA: Elsevier
Churchill Livingstone, 2005; Case 44: 239-41.

6 Answer: B. Anterior to the aorta.

The coeliac plexus consists of the coeliac ganglia with a network of


interconnecting nerve fibres supplying the upper abdominal organs
set 4 answers_set 4 answers.qxd 22-04-2013 19:20 Page 110

Single Best Answer MCQs in Anaesthesia

(pancreas, liver, gall bladder, stomach, spleen, kidneys, small bowel, and
2/3 of the large bowel). Coeliac ganglia lie on each side of the L1 vertebral
body with the aorta lying posteriorly, the pancreas anteriorly and the
inferior vena cava laterally. The plexus receives sympathetic fibres from the
greater splanchnic nerve (T6 to T10), lesser splanchnic nerve (T10, T11),
and least splanchnic nerve (T11, T12).

The block is performed with the patient in the prone position under X-ray
guidance. Normally, two needles are inserted, one on each side to block
both of the coeliac ganglia; good spread to both sides can sometimes be
achieved just using one needle. The needle entry point is just below the tip

110
of the 12th rib. Using X-ray screening in two planes, the needle is
advanced until the tip of the needle is in front of the aorta (one needle
technique) or just lateral to the aorta (two needle technique) at the level of
the L1 vertebra.

Coeliac plexus block relieves pain from non-pelvic intra-abdominal organs.


The common indications are to relieve pain related to chronic pancreatitis
or carcinoma of the pancreas.

Further reading
1. Garcia-Eroles X, Mayoral V, Montero A, et al. Celiac plexus block: a
new technique using the left lateral approach. The Clinical Journal of
Pain 2007; 23: 635-7.

7 Answer: A. Transoesophageal Doppler.

A sudden decrease or complete loss of EtCO2 can be due to


oesophageal intubation, complete disconnection of the airway, total
airway obstruction, cardiac arrest, severe bronchospasm and massive
pulmonary embolism or venous air embolism. Venous air embolism (VAE)
is a recognised complication of posterior fossa craniotomy in the sitting
position. Transoesophageal Doppler and precordial Doppler are
sensitive detectors of air in the right atrium, transoesophageal Doppler
slightly more so than the precordial Doppler. A mill-wheel murmur can be
detected using a precordial stethoscope, but it is a very late sign of VAE.
The changes in pulmonary artery wedge pressure also occur at a later
set 4 answers_set 4 answers.qxd 22-04-2013 19:20 Page 111

Set 4 answers

stage of VAE. Pulse oximetry may show low oxygen saturation which is
not specific for VAE.

Further reading
1. Omlor J. Loss of CO2 trace. In: Near misses in neuroanaesthesia.
Russell GB, Cronin AJ, Longo S, Blackburn TW, Eds. Butterworth
Heinemann, 2002; Case 6: 19-21.
2. Palmon SC, Moore IE, et al. Venous air embolism: a review. Journal of
Clinical Anaesthesia 1997; 9: 251-7.

8 Answer: B. Negative pressure pulmonary


oedema. 111

This patient remained in respiratory distress despite successful


management of the laryngospasm. The most likely diagnosis is either
aspiration of gastric contents or negative pressure pulmonary oedema
(NPPE). Since the condition improved rapidly after application of CPAP
and diuretics it is unlikely to be due to aspiration. There is no history of
asthma (ASA1 patient) and there is no mention of bronchodilators in the
treatment.

NPPE occurs rarely in patients, requiring active intervention for acute


upper airway obstruction during anaesthesia. The risk factors include
obesity with obstructive sleep apnoea, the presence of airway lesions,
upper airway surgery and young male athletes.

High negative intrathoracic pressures initiate a cascade of events in the


development of NPPE. Forceful inspiration against a closed glottis, such
as in laryngospasm, can result in markedly negative intrathoracic
pressures, up to -140cm H2O from a baseline average of -4cm H2O. With
such negative intrathoracic pressure, venous return is increased to the
right heart, subsequently raising pulmonary hydrostatic pressure. The
increased pulmonary hydrostatic pressure leads to the transudation of fluid
from the pulmonary capillaries into the pulmonary interstitial space,
resulting in pulmonary oedema. Hypoxic vasoconstriction of both
pulmonary and systemic arterioles raises systemic blood pressure and
increases the afterload of the left and right ventricles.
set 4 answers_set 4 answers.qxd 22-04-2013 19:20 Page 112

Single Best Answer MCQs in Anaesthesia

Expiration against a closed system generates high intraluminal airway


pressures. This serves as auto-PEEP (positive end-expiratory pressure)
that prevents fluid from leaving the pulmonary capillaries and from entering
the alveoli. When laryngospasm is relieved, either spontaneously or by
reintubation, the resultant drop in airway pressure causes a transudation
of fluid into the alveoli and severe acute pulmonary oedema.

Treatment of NPPE is supportive and includes maintenance of a patent


airway and adequate oxygenation. In severe cases, endotracheal
intubation and mechanical ventilation with PEEP may be necessary.

Further reading
112 1. Thiagarajan RR, Laussen PC. Negative pressure pulmonary oedema
in children - pathogenesis and clinical management. Pediatric
Anesthesia 2007; 17: 307-10.
2. Herrick IA, Mahendran B, Penny FJ. Postoperative pulmonary oedema
following anesthesia. J Clin Anesth 1990; 2: 116-20.
3. Dicpinigaitis PV, Mehta DC. Postobstructive pulmonary oedema
induced by endotracheal tube occlusion. Intensive Care Med 1995;
21: 1048-50.

9 Answer: D. Amitryptiline.

Amitryptiline is a tricyclic antidepressant. Overdose can present with


tachycardia, mydriasis, hyper-reflexia, seizures, coma, ECG changes,
hypotension and arrhythmias.

It acts by sodium channel blockade (class 1a anti-arrhythmic drug). ECG


changes are a useful measure of toxicity with a QRS duration >120ms
indicating a risk of seizures and arrhythmias. A QRS duration >160ms
suggests a particularly high risk, but a normal ECG does not exclude
significant toxicity. There is no specific antidote. Treatment options include
activated charcoal to reduce absorption, sodium bicarbonate to alkalinise
blood, which increases protein-binding therefore reducing the free drug
concentration, and benzodiazepines for seizures. Phenytoin is the anti-
arrhythmic of choice where bicarbonate fails. Supportive measures
including sedation and ventilation may be required until the drug is
metabolised, usually within 24 hours.
set 4 answers_set 4 answers.qxd 22-04-2013 19:20 Page 113

Set 4 answers

Fluoxetine is a selective serotonin reuptake inhibitor. The most common


clinical features after an overdose include agitation, tremor, mild
hypertension, arrhythmias and nystagmus. In combination with other drugs
such as cocaine, tricyclics and MAOIs, it may produce serotonin
syndrome, characterised by a triad of neuromuscular irritability, altered
mental status and autonomic instability.

Ethyl glycol poisoning results in metabolic acidosis with an increased


anion gap. Amphetamine overdose results in hyperthermia due to its effect
on the thermoregulatory centre. It also causes tachycardia, hypertension
and arrhythmias due to its sympathomimetic effect.

Further reading
113
1. Drug overdose. In: Key topics in critical care, 2nd ed. Craft TM, Nolan
JP, Parr MJA, Eds. Taylor and Francis, 2004.
2. Ward C, Sair M. Oral poisoning: an update. British Journal of
Anaesthesia CEACCP 2010; 10: 6-10.

10 Answer: D. Anterior spinal artery syndrome.

Anterior spinal artery syndrome (ASAS) occurs due to inadequate blood


supply to the anterior 2/3 of the spinal cord. It is characterized by a lower
motor neurone type of lesion with reduced or absent pain and temperature
sensation below the level of the lesion. The vibration and proprioception
(joint position sensation) are, however, preserved because, they are
carried by the posterior columns of the spinal cord (supplied by the
posterior spinal artery). The blood supply to the anterior 2/3 of the spinal
cord is reinforced by the anterior radicular arteries. The largest radicular
artery, the artery of Adamkiewicz, usually arises from the aorta at the level
of T8-T11 (may vary from T8-L4). There is an increased risk of damage to
this radicular artery during major thoraco-abdominal surgical procedures,
such as aneurysm repair and oesophagectomy. Other causes include
aortic disease (atherosclerosis of the aorta and dissecting aortic
aneurysm), hyper-coagulability, acute transverse myelitis, arteriovenous
malformation, sickle cell disease and acute transverse myelitis.

Further radiological investigation is essential to rule out other causes of


paraplegia such as epidural abscess and haematoma, which are less likely
set 4 answers_set 4 answers.qxd 22-04-2013 19:20 Page 114

Single Best Answer MCQs in Anaesthesia

on the third postoperative day. Further clinical signs such as pyrexia and a
raised white cell count would support the diagnosis of an epidural
abscess.

The posterior spinal artery suplies the posterior 1/3 of the spinal cord,
mainly the dorsal columns. Ischaemia of the posterior 1/3 of the spinal
cord is less common than the anterior 2/3.

Further reading
1. Djurberg H. Haddad M. Anterior spinal artery syndrome. Paraplegia
following segmental ischaemic injury to the spinal cord after
oesophagectomy. Anaesthesia 1995; 50: 345-8.
114 2. Sinha AC, Cheung AT. Spinal cord protection and thoracic aortic
surgery. Current Opinion in Anaesthesiology 2010; 23: 95-102.

11 Answer: D. Deep peroneal and superficial


peroneal, and posterior tibial nerves.

Ankle block is a safe and effective method for providing analgesia


following surgical procedures on bones and soft tissues of the foot and
can also be used alone as a regional technique. Five nerve branches carry
sensation from the different areas of the foot. All are branches of the
sciatic nerve, except the saphenous nerve, which is the terminal branch of
the femoral nerve. The sciatic nerve divides into the tibial nerve and the
common peroneal nerve at a variable point between the buttock and the
popliteal fossa. The tibial nerve then divides into the posterior tibial and
sural nerves, and the common peroneal nerve into the deep and superficial
peroneal nerves. The posterior tibial nerve finally divides into the medial
and lateral plantar nerves. The medial plantar nerve provides sensory
innervation to the plantar aspect of the great toe, the deep peroneal nerve
innervates the first web space and the superficial peroneal nerve
innervates the medial half of the dorsal aspect of the great toe. All three
nerves therefore need to be blocked for great toe amputation.

Further reading
1. Reilley TE, Gerhardt MA. Anesthesia for foot and ankle surgery. Clin
Podiatr Med Surg 2002; 19: 125-47.
set 4 answers_set 4 answers.qxd 22-04-2013 19:20 Page 115

Set 4 answers

12 Answer: A. I.M. carboprost 250mg repeated at


15-minute intervals if necessary.

Carboprost has been successfully used following failure of conventional


treatment. It is not licensed for intra-myometrial use. The uterus after
delivery is extremely vascular and an intra-myometrial injection may cause
systemic adverse affects. The BNF recommended dose of oxytocin after a
Caesarean section is 5 units I.V. followed by an infusion if necessary.
Rectal administration of drugs can be unreliable in a shocked patient. In
the non-shocked patient, misoprostol is often used at a dose of 800mg per
rectally at the end of the operation to maintain uterine tone.
115
Further reading
1. Howell C, Irani S. Massive obstetric haemorrhage. In: MOET course
manual, 2nd ed. Royal College of Obstetrics & Gynaecology (RCOA)
Press, 2007; Chapter 17: 179.

13 Answer: E. Maintenance of a normal to high


heart rate avoiding sudden bradycardia, and
maintenance of a normal to low SVR.

General anaesthesia is usually the technique of choice in patients with


mitral regurgitation. Although a decrease in systemic vascular resistance
(SVR) is beneficial, an uncontrolled or sudden response can be
detrimental. The main aim is to avoid events that may further decrease
cardiac output. Maintenance of a normal to slightly increased heart rate is
recommended, as forward left ventricular stroke volume is likely to be
heart rate-dependent. Likewise, any sudden increase in systemic vascular
resistance will increase the regurgitant fraction and thereby reduce the
forward flow.

Further reading
1. Valvular heart disease. In: Anesthesia and co-existing diseases, 4th
ed. Stoelting RK, Dierdorf S, Eds. Philadelphia, USA: Churchill
Livingstone, 2002; Chapter 2: 33-5.
set 4 answers_set 4 answers.qxd 22-04-2013 19:20 Page 116

Single Best Answer MCQs in Anaesthesia

14 Answer: E. Pericarditis.

The presenting features of pericariditis include retrosternal stabbing chest


pain, a low-grade intermittent fever, shortness of breath, a cough and
dyspnoea. The most common clinical finding is a pericardial rub. Serous
pericarditis is commonly seen in rheumatoid arthritis and systemic lupus
erythematosus (SLE). The other causes of acute pericarditis include viral
infection, tuberculosis and scleroderma. Acute pericarditis with a
pericardial effusion may occur following myocardial infarction and trauma
such as cardiac surgery and pacemaker insertion.

116 The classical signs of cardiac tamponade include an elevated JVP,


hypotension and muffled heart sounds. Signs of right and left ventricular
failure can be seen in patients with cardiac tamponade. The clinical
features of congestive cardiac failure include peripheral oedema, ascites
and raised JVP. In pericarditis, superficial inflammation of the myocardium
results in diffuse ST segment elevation on the ECG. Absence of reciprocal
ST segment depression on the ECG distinguishes acute pericarditis from
acute myocardial infarction.

Further reading
1. Pericardial Diseases. In: Anesthesia and co-existing disease, 4th ed.
Stoelting RK, Dierdorf SF, Eds. Philadelphia, USA: Churchill
Livingstone, 2002; 135-42.

15 Answer: B. Her operative mortality is about


25%.

In patients with liver disease the mortality risk can be estimated using
Pugh’s modification of Child’s scoring system. There are five variables in
the scoring system (Table 1). Each variable has a lowest score of 1 and a
highest score of 3. From the available clinical and laboratory data, this
patient has a total score of 8. This patient therefore carries a moderate
peri-operative risk with a mortality rate of about 25%.
set 4 answers_set 4 answers.qxd 22-04-2013 19:20 Page 117

Set 4 answers

Table 1. Pugh’s modification of Child’s score.

Variable Score
1 2 3
Encephalopathy None Minimal Advanced
Ascites None Minimal Moderate
Bilirubin, mg/dL <2 2-3 >3
Albumin, g/L >35 28-35 <28
PT (s above control) <4 4-6 >6

(Encephalopathy: minimal = grade 1 and 2; advanced = grade 3 and 4)

Total score versus risk 117


Total score Child’s class Operative mortality
5-6 A Low, <5%
7-9 B Moderate, 25%
10-15 C High, >50%

Further reading
1. Diseases of the liver and biliary tract. In: Anesthesia and co-existing
diseases, 4th ed, Stoelting RK, Dierdorf S, Eds. Philadelphia, USA:
Churchill Livingstone, 2002; Chapter 18: 299-324.
2. Vaja R, McNicol R, Sisley I. Anaesthesia for patients with liver disease.
British Journal of Anaesthesia 2010; 10: 15-9.

16 Answer: D. Posterior angle of the rib.

The intercostal nerve enters the subcostal groove and continues to run
parallel to the rib. Its course within the thorax is sandwiched between
internal intercostal and innermost intercostal muscles. A typical intercostal
nerve gives off the lateral cutaneous branch and terminates as the anterior
cutaneous nerve.

The lateral cutaneous branch of an intercostal nerve is given off at variable


points distal to the posterior axillary line. So, for reliable blockade of the
lateral cutaneous branch, the intercostal nerve should be blocked at the
set 4 answers_set 4 answers.qxd 22-04-2013 19:20 Page 118

Single Best Answer MCQs in Anaesthesia

posterior angle of the rib. 3ml of local anesthetic solution injected through
a needle spreads some 4-6cm easily along that single subcostal groove
distally and proximally. If a catheter is inserted at the angle of the rib and
directed medially 2-3cm, a larger volume (about 20ml) of solution could be
injected which can spread to the paravertebral space to block 3-5
intercostal nerves.

Further reading
1. Karmakar MK, Ho AMH. Acute pain management of patients with
multiple fractured ribs. J Trauma 2003; 54: 612-5.

118 17 Answer: D. Idiopathic pulmonary hypertension.

Idiopathic pulmonary hypertension (IPH) is defined as a mean pulmonary


artery pressure greater than 25mm Hg at rest with a normal pulmonary
capillary wedge pressure and the absence of identifiable causes of
pulmonary hypertension. The exact pathogenesis of IPH is unclear. The
mechanism that appears to be most widely accepted is that of pulmonary
vasoconstriction with an imbalance of vasoactive mediators. Factors such
as thromboxane, arachidonic acid metabolites, and prostacyclin, as well as
other endothelial factors, have been invoked. Trigger factors such as high
altitude, hypoxaemia, drugs, toxins, sympathetic tone, and autoimmune
disorders, can cause pulmonary vasoconstriction in susceptible
individuals.

Morbidity and mortality rates vary and depend on the age, the degree of
pulmonary hypertension, and the response to vasodilator therapy. Death
as a result of both acute and chronic right heart failure and its associated
arrhythmias may occur. The treatment of IPH includes general medical
measures such as annual influenza vaccination, treating fever and
respiratory illnesses aggressively, and supplemental oxygen. Patients with
severe pulmonary hypertension resulting in recurrent syncope or right-to-
left intracardiac shunting may benefit from palliation with blade atrial
septostomy or balloon dilation of the atrial septum.

The rationale for the use of vasodilators in patients with IPH is to


counteract vasoconstriction. The various drugs used are oral calcium
set 4 answers_set 4 answers.qxd 22-04-2013 19:20 Page 119

Set 4 answers

channel blockers (e.g. nifedipine) and continuous intravenous


prostacyclin. The latter may be recommended for the patient with right
heart failure and/or symptoms that may include syncope. Endothelin
receptor blockers and phosphodiesterase-5 inhibitors have also been
advocated.

Further reading
1. Rashid A, Ivy D. Severe paediatric pulmonary hypertension: new
management strategies. Arch Dis Child 2005; 90(1): 92-8.
2. Melson H, Sykes E, et al. Perioperative implications of pulmonary
hypertension. CPD Anaesthesia 2008; 10: 39-66.

18
119
Answer: C. Hydralazine and magnesium.

This lady has severe pre-eclampsia with a diastolic blood pressure


³110mmHg and systolic blood pressure ³170mmHg with significant
proteinuria (3+ is approximately equal to 3g/L). The presence of any of the
following clinical features constitutes severe pre-eclampsia (in addition to
hypertension and proteinuria):

w Symptoms of severe headache.


w Visual disturbance.
w Liver tenderness.
w Platelet count falling to below 100 x 106/L.
w Epigastric pain and/or vomiting.
w Abnormal liver enzymes (ALT or AST rising to above 70I.U./L).
w Clonus.
w HELLP syndrome.
w Papilloedema.

Antihypertensive treatment should be started in women with a systolic


blood pressure over 160mmHg or a diastolic blood pressure over
110mmHg or other markers of severe disease. Labetalol should be
avoided in women with known asthma.

Hydralazine can be used for the acute management of severe


hypertension.
set 4 answers_set 4 answers.qxd 22-04-2013 19:20 Page 120

Single Best Answer MCQs in Anaesthesia

In addition to hydralazine, magnesium is likely to be beneficial in view of


the signs of cerebral irritability and prevention of eclamspsia. The patient
should be closely monitored, treated with antihypertensives and stabilised,
and delivery should be considered depending on the maternal response to
treatment and foetal age.

Further reading
1. RCOG Green-Top 10A guideline: The management of severe pre-
eclampsia/eclampsia. (www.rcog.org.uk).

19 Answer: B. Tension pneumothorax.


120
The two main complications of positive pressure ventilation in a patient with
asthma are pneumothorax and haemodynamic collapse. The reported
incidence of pneumothorax during manual ventilation following endotracheal
intubation is as high as 75%. During manual bagging extremely high airway
pressures can be generated. To prevent a pneumothorax, slow rate low tidal
volume ventilation should be used. Sepsis and myocardial infarction are also
possible diagnoses. Patients with severe asthma usually present with
tachycardia and hypertension due to increased sympathetic drive and
release of catecholamine. Associated fever, hyperventilation and sweating
can lead to dehydration and hypovolaemia. Consequently, soon after
sedation is administered, the vasodilatory effect of sedation and reduced
catecholamine levels can result in cardiovascular collapse.

Further reading
1. De Mendoza D, Lujan M, Rello J. Mechanical ventilation for asthma
Exacerbations. In: Yearbook of intensive care medicine. Vincent LL,
Eds. New York, USA: Springer, 2008; VII: 256-68.
2. Rodrigo GJ, Rodrigo C, Hall JB. Acute asthma in adults: a review.
Chest 2004; 125: 1081-102.

20 Answer: A. Intravenous dexamethasone.

Insertion of a nasogastric tube can result in oropharyngeal trauma. In this


patient it resulted in trauma to the uvula and subsequent oedema leading
to difficulty in breathing. The other cause of trauma to the uvula could have
set 4 answers_set 4 answers.qxd 22-04-2013 19:20 Page 121

Set 4 answers

been from the blind Yankauer suction. Dexamethasone is the treatment of


choice for uvular oedema. The anti-inflammatory potency of
dexamethasone is 30 times greater than hydrocortisone. Dexamethasone
also has a long half-life of 36-72 hours. Steroids decrease the capillary
permeability and therefore reduce the oedema. I.V. hydrocortisone is less
effective than dexamethasone in reducing oedema.

Further reading
1. Holden JP, Vaughn WC, Brock-Utne JG. Airway complications
following functional endoscopic sinus surgery. J Clinical Anesthesia
2002; 14: 154-7.
2. Postoperative airway complications after sinus surgery. In: Clinical
anesthesia: near misses and lessons learned. Brock-Utne JG, Ed. 121
New York, USA: Springer, 2008; Case 22: 54-6.
3. Hawkins DB, Crockett DM, Shum TK. Corticosteroids in airway
management. Otolaryn Head Neck Surg 1983; 91: 593-6.

21 Answer: C. Amputation of limb.

Chronic post-surgical pain (CPSP) is a serious and relatively common


complication following surgery. It is defined as pain occurring after a
surgical procedure, at least 2 months after the procedure, when no other
cause for the pain can be found (e.g. malignancy, infection). The incidence
of CPSP following specific surgical procedures is shown in Table 2.

Table 2. The incidence of chronic post-surgical pain (CPSP)


following specific surgical procedures.

Cholecystectomy 3-56%
Mastectomy 11-57%
Amputation of limb 30-85%
Thoracotomy 5-67%
Hernia repair 0-63%

Further reading
1. Searle RD, Simpson KH. Chronic post-surgical pain. British Journal of
Anaesthesia CEACCP 2010; 10: 12-4.
set 4 answers_set 4 answers.qxd 22-04-2013 19:20 Page 122

Single Best Answer MCQs in Anaesthesia

22 Answer: B. Haloperidol 0.5-1mg I.V.

The most likely diagnosis is postoperative delirium (POD). The incidence


in elderly patients with hip fracture is high, ranging from 16-62% with an
average of 35%. The primary treatment is identification and correction of
any underlying causes such as pain, hypoxia and dehydration.
Pharmacological treatment may be necessary when agitation puts the
patient and staff at risk of harm.

Haloperidol is the drug of choice for treating POD. It is administered at


a dose of 0.5-1mg I.V. every 5-10 minutes until the agitation is controlled.
122 It has a long half-life of up to 72 hours; it is therefore essential to titrate
the dose carefully to avoid over sedation. Benzodiazepines such as
midazolam and diazepam can result in sedation, respiratory depression
and hypoxia, which may aggravate agitation. In patients undergoing
cardiac surgery, ketamine has been shown to reduce the incidence of
POD when administered at a dose of 0.5mg/kg at induction.

Further reading.
1. Deiner S, Silverstein JH. Postoperative delirium and cognitive
dysfunction. British Journal of Anaesthesia 2009; 103 (Suppl. I): i41-
6.

23 Answer: D. Uterine atony.

The most common cause for intra and postpartum haemorrhage following
prolonged labour is uterine atony. Other less common causes include a
retained placenta, other causes of mechanical obstruction to contraction,
and genital tract trauma. Rarer causes include coagulopathies,
endometritis and intra-uterine sepsis.

Further reading
1. RCOG Green-Top 10A guideline: The management of severe pre-
eclampsia/eclampsia. (www.rcog.org.uk).
set 4 answers_set 4 answers.qxd 22-04-2013 19:20 Page 123

Set 4 answers

24 Answer: A. Platelet count of 102 x 109/L 6 days


after starting heparin.

It is important to distinguish heparin-induced thrombocytopaenia (HIT)


from other causes of a low platelet count. HIT usually occurs 5 days after
starting heparin, unless the patient has been exposed to heparin in the
previous 30 days, in which case it can occur immediately. HIT is
characterized by a fall in the platelet count to less than 50% of its previous
level. Extremely low platelet counts (typically <15 x 109/L) are not usually
associated with HIT.

123
Not all platelet-heparin antibodies are associated with the development of
HIT. Only 1% of patients with detectable antibodies will develop clinically
relevant disease. IgA and IgM antibodies are unimportant - only those with
IgG antibodies will develop the disease.

In true HIT the platelet count will start to recover 2 to 3 days after stopping
heparin, and is usually normal by day 14. A recovery more rapid than this
does not support the diagnosis of HIT.

Other causes of thrombocytopaenia must be ruled out in order for the


diagnosis of HIT to be made. Disseminated intravascular coagulation
(DIC) is associated with severe sepsis and will lead to consumption of
platelets. A low fibrinogen level supports the diagnosis of DIC rather than
HIT.

Further reading
1. Hall A, Thachil J, Martlew V. Heparin-induced thrombocytopenia in the
intensive care unit. J Intensive Care Soc 2010; 11: 20-5.

25 Answer: E. Mallory-Weiss syndrome.

Mallory-Weiss syndrome is upper gastro-intestinal bleeding due to linear


mucosal tears at the gastro-oesophageal junction, and typically occurs
following episodes of retching and vomiting. There is usually a history of
excessive alcohol intake preceding the episode of vomiting. Massive
bleeding is uncommon in reflux oesophagitis and acute gastritis. The
set 4 answers_set 4 answers.qxd 22-04-2013 19:20 Page 124

Single Best Answer MCQs in Anaesthesia

history of alcohol abuse and absence of occult blood in the stool


differentiates Mallory-Weiss syndrome from a bleeding peptic ulcer. A
history of epigastric pain and the passage of black stools (melaena)
suggest peptic ulcer disease. A hiatus hernia can cause reflux
oesophagitis. Reflux oesophagitis usually causes epigastric pain which
often radiates to the back.

Further reading
1. Diseases of the gastro-intestinal system. In: Anesthesia and co-
existing diseases, 4th ed. Stoelting RK, Dierdorf S, Eds. Philadelphia,
USA: Churchill Livingstone, 2002; Chapter 19: 337-8.

26
124
Answer: B. Stellate ganglion block.

This patient has refractory angina. The various treatment options for this
condition include analgesic drugs such as opioids, TENS therapy, stellate
ganglion block, and spinal cord stimulation. In this patient opioids may not
be safe as he suffers from obstructive sleep apnoea. TENS therapy is
contraindicated due to the presence of a pacemaker. Refractory angina
pain is vascular pain and responds poorly to NSAIDs. A stellate ganglion
block is therefore the most suitable option in this patient.

The stellate ganglion is formed by the fusion of the inferior cervical and the
first thoracic ganglion as they meet at the neck of the first rib. It is present
in 80% of subjects. The structures anterior to the ganglion include the skin
and subcutaneous tissue, the sternocleidomastoid and the carotid sheath.
The dome of the lung lies anterior and inferior to the ganglion. The
prevertebral fascia, vertebral body of C7, oesophagus and thoracic duct
lie medially. Structures posterior to the ganglion include the longus colli
muscle, anterior scalene muscle, vertebral artery, brachial plexus sheath
and neck of the first rib.

The indications for a stellate ganglion block include complex regional pain
syndrome types I and II, refractory angina, phantom limb pain, post-
herpetic neuralgia and vascular insufficiency, such as Raynaud’s
syndrome.
set 4 answers_set 4 answers.qxd 22-04-2013 19:20 Page 125

Set 4 answers

A successful stellate ganglion block is indicated by the development of


Horner’s syndrome. The classic clinical findings associated with Horner’s
syndrome are ptosis, enophthalmos, pupillary miosis and facial anhidrosis.
The benefits of stellate ganglion block may only be temporary; the duration
of analgesia is very variable.

Further reading
1. Chester M, Hammond C, Leach A. Long-term benefits of stellate
ganglion block in severe chronic refractory angina. Pain 2000; 87:
103-5.

27 Answer: A. The axillary nerve. 125

The axillary nerve arises from the posterior cord of the brachial plexus and
carries nerve fibres from C5 and C6. It travels through a quadrilateral
space bounded above by teres major, below by teres minor, medially by
the long head of triceps and laterally by the surgical neck of the humerus.
It divides into anterior and posterior branches. The anterior branch of the
axillary nerve winds round the surgical neck of the humerus. The axillary
nerve provides sensory innervation to the lower part of the deltoid region
and motor innervation to the deltoid and teres minor muscles. Weakness
of the deltoid limits abduction of the arm (0-30°). Contraction of
supraspinatus causes initial abduction up to 30°, and further abduction
from 30-90° is achieved by contraction of the deltoid.

The median nerve supplies the pronator teres and flexors of the wrist
except flexor carpi ulnaris and part of flexor digitorum profundus. Median
nerve injury above the level of the elbow results in weakness of pronation
and flexion of the wrist.

The suprascapular nerve supplies the supraspinatus and infraspinatus


muscles. Injury to the musculocutaneous nerve results in weakness of
flexion of the forearm (biceps brachii, coracobrachialis and brachialis).
Radial nerve injury results in weakness of extension at the elbow and wrist.

Further reading
1. Kroll DA, Caplan RA, Posner K, et al. Nerve injury associated with
anesthesia. Anesthesiology 1990; 73: 202-7.
set 4 answers_set 4 answers.qxd 22-04-2013 19:20 Page 126

Single Best Answer MCQs in Anaesthesia

28 Answer: D. Ruptured abdominal aortic


aneurysm.

The majority of abdominal aortic aneurysms (AAA) are asymptomatic and


are incidentally discovered during routine physical examination or during
an unrelated diagnostic imaging study.

The classic triad of ruptured AAA is hypotension, a pulsatile abdominal


mass, and flank or back pain. Back pain is the most common symptom,
due to leak of blood into the retroperitoneal space. The combination of a
past history of aneurysm and the sudden onset of severe back ache
126
should lead to the diagnosis of ruptured aortic aneurysm.

Pain in the right hypochondrium and at the inferior angle of the right
scapula is common in cholecystitis. In pancreatitis, pain is in the epigastric
region, and often radiates to the back. In severe acute pancreatitis the
patient may present in shock. Nausea and vomiting are common; there
may be a past history of dyspepsia, biliary colic or transient jaundice. In
lumbar disc prolapse, the back pain usually radiates to the legs and shock
is uncommon.

Further reading
1. Tan WA, Makaroun MS. Abdominal aortic aneurysm rupture.
(http://emedicine.medscape.com/article/416397-overview).

29 Answer: C. Serum calcium.

A rapid decrease in serum calcium can produce skeletal muscle spasm


manifesting as laryngospasm. The parathyroid glands may be accidentally
removed or may be injured during total thyroidectomy resulting in
hypocalcaemia. The serum thyroid stimulating hormone (TSH) level is not
usually altered immediately following this surgery. TSH measurement may
be needed at a later stage to assess thyroid replacement therapy.
Hypocalcaemia can result in a prolonged QT interval on the ECG but this
is not a reliable guide to the presence of hypocalcaemia.
set 4 answers_set 4 answers.qxd 22-04-2013 19:20 Page 127

Set 4 answers

Hypoalbuminaemia results in low measured serum calcium but the ionised


calcium can be normal. Other causes include acute pancreatitis, vitamin D
deficiency, hypomagnesaemia, malnutrition, sepsis, parathyroidectomy
and renal failure. Hyperventilation and metabolic alkalosis result in low
ionised calcium.

The clinical signs and symptoms of hypocalcaemia are caused by


increased neuronal irritability leading to neurological, respiratory,
cardiovascular and psychiatric manifestations. Neurological manifestations
include paraesthesia of the distal extremities and circum-oral area,
Chvostek’s and Trousseau’s signs, muscle cramps, tetany, and seizures.
Acute hypocalcaemia can result in laryngospasm and bronchospasm,
particularly after parathyroidectomy. Cardiovascular manifestations include 127
hypotension, bradycardia and precipitation of digitalis toxicity.

Further reading
1. Aguilera IM, Vaughan RS. Calcium and anaesthetist. Anaesthesia
2001; 55: 779-90.
2. Water, electrolyte and acid-base disturbances. In: Anesthesia and co-
existing diseases, 4th ed. Stoelting RK, Dierdorf S, Eds. Philadelphia,
USA: Churchill Livingstone, 2002; Chapter 21: 385-90.

30 Answer: B. Lingual nerve.

The lingual nerve is a branch of the mandibular nerve, and carries the
fibres from the chorda tympani (branch of the facial nerve). The lingual
nerve provides sensory innervation to the mucous membrane of the
anterior two-thirds of the tongue, and to the side wall and floor of the
mouth. The fibres from the chorda tympani are secretomotor to the
submandibular and sublingual salivary glands and also carry taste
sensation from the anterior two-thirds of the tongue.

The inferior alveolar nerve is the largest branch of the mandibular nerve. It
provides sensory innervation to the molar teeth and premolar teeth.

The glossopharyngeal nerve carries sensation from the pharynx, the


tonsillar region and the posterior one third of the tongue. It provides motor
set 4 answers_set 4 answers.qxd 22-04-2013 19:20 Page 128

Single Best Answer MCQs in Anaesthesia

innervation to the stylopharyngeus muscle and to the secretomotor fibres


of the parotid gland. It also innervates the carotid sinus and body.

The buccal nerve supplies the skin over the anterior part of the cheek, the
mucous membrane of the inner aspect of the cheek and the lateral aspect
of the gum adjacent to the molar teeth of the mandible.

The hypoglossal nerve supplies all the intrinsic and extrinsic muscles of the
tongue (with the exception of palatoglossus).

Further reading

128
1. The cranial nerves. In: Anatomy for anaesthetists. 8th ed. Ellis H,
Feldman S, Harrop-Griffiths W. Oxford: Blackwell Science Ltd, 2004;
Part 6: 235-83.
2. Graff-Radford SB, Evans RW. Lingual nerve injury.
(http://www.medscape.com/viewarticle/462066).
set 5_set 5.qxd 22-04-2013 19:21 Page 129

Set 5 questions

Set 5
1 A 44-year-old female presents with generalized body pain. She feels
lethargic and also complains of depression. She has generalized 129
muscle tenderness all over her body. She is known to have irritable
bowel syndrome. What is the most likely diagnosis?

a. Muscular dystrophy.
b. Polymyalgia rheumatica.
c. Fibromyalgia.
d. Chronic fatigue syndrome.
e. Rheumatoid arthritis.

2 A 26-year-old primigravida diagnosed with severe pre-eclampsia


was treated with magnesium sulphate. She subsequently had a
Caesarean section performed under epidural anaesthesia. No
adverse events were recorded intra-operatively and she was
transferred to the high dependency unit where she was making a
good recovery. 24 hours later she becomes lethargic and confused.
She has reduced muscle tone and reflexes. Her ECG shows a
prolonged P-R interval and widened QRS. What is the most likely
diagnosis?

a. Pulmonary embolism.
b. Amniotic fluid embolism.
c. Hypermagnesaemia.
d. Hyponatraemia.
e. Hyperkalaemia.
set 5_set 5.qxd 22-04-2013 19:21 Page 130

Single Best Answer MCQs in Anaesthesia

3 A 78-year-old man underwent a hemi-arthroplasty of the left hip


under general anaesthesia. He was given atropine in recovery for
severe bradycardia. Following this he become extremely restless,
agitated and confused. Which of the following is likely to be the most
suitable treatment for this patient's restlessness and agitation?

a. Intravenous midazolam.
b. Intravenous physostigmine.
c. Intravenous neostigmine.
d. Intravenous haloperidol.
e. Oral clonidine.

4
130

A 46-year-old male patient is admitted to the high dependency unit


following severe myalgia, diarrhoea, sweating, cough and fever with
a temperature of 39.1°C. He is fit and well otherwise and has a
history of recent travel abroad on a business trip. A diagnosis of
severe community-acquired pneumonia is made and culture yields a
gram negative bacillus. Which of these organisms is most likely to
be the cause of his infection?

a. Streptococcus pneumoniae.
b. Legionella pneumophilia.
c. Staphylococcus aureus.
d. Haemophilus influenzae.
e. Mycoplasma pneumoniae.

5 A 66-year-old female is scheduled for major surgery. She suffers


from Parkinson’s disease for which she is treated with levodopa.
She has a history of PONV. Which of the following anti-emetic
regimes would be most suitable for this patient?

a. Domperidone and ondansetron.


b. Metoclopramide and ondansetron.
c. Droperidol and prochlorperazine.
d. Ondansetron and prochlorperazine.
e. Prochlorperazine and metoclopramide.
set 5_set 5.qxd 22-04-2013 19:21 Page 131

Set 5 questions

6 A 72-year-old male who is a chronic smoker complains of constant


leg pain. Clinical examination has revealed ischaemic ulcers in both
his legs. He has been assessed by a vascular surgeon and has been
referred to the pain clinic for the management of his pain. Which of
the below is likely to be the most effective treatment for his pain?

a. Lumbar epidural steroids.


b. Nerve root block.
c. Chemical lumbar sympathectomy.
d. Regular morphine.
e. Superior hypogastric plexus block.
131

7 A 30-year-old male patient is undergoing a laparoscopic


cholecystectomy. Halfway through the surgery, despite adequate
inspired oxygen and ventilation, his SpO2 decreases to 90%.
Clinical examination reveals distended neck veins, and reduced
movement and breath sounds on the left side of the chest. What
should be the next step in the management of this patient?

a. Increasing minute ventilation.


b. Arterial blood gas analysis.
c. Endotracheal suction.
d. Needle decompression and insertion of a chest drain.
e. Application of PEEP.

8 A 65-year-old male patient with a history of heavy smoking presents


with chest pain and a cough of 2 months’ duration. On clinical
examination there are enlarged supraclavicular lymph nodes on the
left side. Chest X-ray demonstrates a 2cm lesion in the left upper
lobe. The biochemistry reveals normal potassium, urea and
creatinine but serum sodium is 124mmol/L. Which of following is the
most likely cause for his hyponatraemia?
set 5_set 5.qxd 22-04-2013 19:21 Page 132

Single Best Answer MCQs in Anaesthesia

a. Cerebral salt wasting syndrome.


b. Small cell carcinoma of the lung.
c. Adrenal insufficiency.
d. Hypothyroidism.
e. Pulmonary tuberculosis.

9 A 58-year-old gentleman with a history of recent recovery from a flu-


like illness has been admitted to the medical ward with a history of
progressive ascending weakness of the lower limbs with preserved
bladder function. On examination he has flaccid paralysis and

132
peripheral neuropathy in the lower limbs. CSF examination shows a
normal cell count but a raised protein concentration. The most likely
diagnosis is:

a. Myasthenia gravis.
b. Transverse myelitis.
c. Severe electrolyte disturbances.
d. Motor neurone disease.
e. Guillain-Barré syndrome.

10 A 70-year-old male patient is scheduled for a right carotid


endarterectomy under a cervical plexus block. A deep cervical
plexus block is performed using 12ml of 0.5% levobupivacaine. Five
minutes after completing the injection, the patient complains of
difficulty in breathing and soon becomes unconscious and apnoeic
requiring intubation and ventilation. When he is still able to speak he
denies circumoral numbness or tingling. The blood pressure
decreases from 170/84 to 70/44mm Hg and responds to
intravenous fluids and ephedrine. The ECG shows sinus rhythm with
a heart rate of 64 bpm. About an hour later the patient wakes up and
is extubated. The neurological examination is completely normal and
he is cardiovascularly stable. The most likely cause for the
respiratory arrest in this patient is:

a. Local anaesthetic toxicity.


b. Epidural block.
c. Subarachnoid block.
set 5_set 5.qxd 22-04-2013 19:21 Page 133

Set 5 questions

d. Intravascular injection.
e. Right phrenic nerve palsy.

11 A 70-year-old female with myopia is scheduled to have a cataract


operation. She is known to have COAD and has been on a home
nebuliser for the last 6 months. She also takes warfarin for atrial
fibrillation; her most recent INR was 2.2. Which one of the following
is the most appropriate anaesthetic technique?

a. Retrobulbar block.
b. Peribulbar block.
c. Sub-Tenon’s block. 133
d. General anaesthetic with endotracheal intubation.
e. General anaesthetic with LMA.

12 A 60-year-old man with hypertension presents for an inguinal hernia


repair. His past medical history includes a heart transplant 3 years
before for idiopathic cardiomyopathy. During the intra-operative
period which of the following drugs would be most suitable for
treating bradycardia?

a. Glycopyrrolate.
b. Atropine.
c. Ephedrine.
d. Isoprenaline.
e. Norephedrine.

13 For patients presenting for thoracic surgery in which of the following


is the use of a double-lumen tube for lung isolation most strongly
indicated?

a. Thoracic aortic aneurysm.


b. Giant unilateral lung cyst.
c. Lobectomy.
d. Oesophagectomy.
e. Spinal thoracic surgery.
set 5_set 5.qxd 22-04-2013 19:21 Page 134

Single Best Answer MCQs in Anaesthesia

14 A 37-year-old female patient is admitted to the intensive care unit


with shortness of breath and hypotension. Despite vigorous
resuscitation, she remains hypotensive. Her heart rate is 110bpm
and BP is 92/50mmHg. The results of pulmonary artery flotation
catheter measurements are shown in Table 1.

Table 1. Results of pulmonary artery flotation catheter measurements.

RV pressure PA pressure PA wedge pressure


134
65/12mmHg 80/30mmHg 36mmHg

Which of the following drug combinations would be the most


appropriate in this patient?

a. Noradrenaline and dobutamine.


b. Noradrenaline and propranolol.
c. Furosemide and dobutamine.
d. Nitroglycerine and dobutamine.
e. Noradrenaline and nitroglycerine.

15 A 70-year-old male patient with type II diabetes, hypertension and


ischaemic heart disease is undergoing a laparotomy for carcinoma
of the sigmoid colon. Which of the following monitors would be the
most sensitive detector of intra-operative myocardial ischaemia?

a. Electrocardiography.
b. Transosesophageal echocardiography.
c. Pulmonary capillary wedge pressure measurement.
d. ECG monitoring with CM5 configuration.
e. Dipyridamole-thallium scanning.
set 5_set 5.qxd 22-04-2013 19:21 Page 135

Set 5 questions

16 An 82-year-old lady presents with unilateral burning pain in the T7


dermatome on the right side. She complains of severe pain
associated with a tingling and itching sensation. She is unable to
tolerate light touch in the area. Her medical history includes
hypertension, COAD, and end-stage renal failure. She recently had
a course of steroids. What would be the most appropriate initial
treatment for her pain?

a. Gabapentin.
b. Amitriptyline.
c. Morphine sulphate.
d. Lidocaine 5% plasters. 135
e. Capsaicin 0.025% cream.

17 A 47-year-old female is receiving intravenous antibiotic therapy for


subacute bacterial endocarditis. She gradually becomes more
breathless. Clinical examination reveals a pulse rate of 112/minute,
a BP of 118/52mmHg, raised jugular venous pressure and diastolic
and systolic murmurs at the left sternal edge. The most likely
diagnosis in this patient is:

a. Ruptured aneurysm of the ascending aorta.


b. Mitral stenosis.
c. Cardiac failure.
d. Ruptured valve cusp of the mitral valve.
e. Acute myocardial infarction.

18 A 32-year-old woman with a history of asthma presents to the


emergency department with shortness of breath and wheeze. On
arrival to the emergency department her respiratory rate is
34/minute with an SpO2 of 90%. She is immediately given oxygen
at 15L/minute followed by nebulised salbutamol 5mg and
ipratropium bromide 500µg. Fifteen minutes after the nebulisers
have been administered, her respiratory rate is still rapid at
32/minute and her SpO2 is 92%. Clinical examination reveals no
set 5_set 5.qxd 22-04-2013 19:21 Page 136

Single Best Answer MCQs in Anaesthesia

wheezing. An arterial blood gas measurement shows a PaO2 of


8kPa and a PaCO2 of 6.5kPa. Which of the following should be
performed next?

a. Rapid sequence induction, intubation and commencement of


positive pressure ventilation.
b. Intravenous salbutamol by infusion at 5µg/min.
c. Intravenous aminophylline 5mg/kg.
d. Intravenous magnesium sulphate 2g over 20 minutes.
e. Administration of Heliox (70:30 helium/oxygen mixture) by face mask.

19 A 45-year-old patient is undergoing a craniotomy and evacuation of


136

an intracranial haematoma. Anaesthesia is induced and maintained


with target-controlled infusions of remifentanil and propofol. An
arterial blood sample, prior to craniotomy, has shown a PaO2 of
42kPa and PaCO2 of 4.7kPa. On opening the skull the
neurosurgeon comments that the appearance suggests significantly
raised intracranial pressure. Which of the following therapeutic
options should be carried out first to reduce the ICP?

a. Intravenous infusion of mannitol 0.5g/kg.


b. Intravenous administration of 8mg of dexamethasone.
c. Use of a vasopressor infusion to achieve a mean arterial pressure of
90mmHg.
d. Administration of 0.9% saline to achieve a haematocrit of 30%.
e. Increase in minute volume to achieve a PaCO2 of 3.5kPa.

20 A 37-year-old man presents for surgery on his hand. He has no


medical problems, and wants to have his surgery done under a
regional block. A brachial plexus block via the axillary approach is
performed using a total of 30ml of 0.375% levobupivacaine. The
block is tested before the start of surgery. Sensation is maintained
over the thumb and posterior surface of the hand, but there is good
anaesthesia over the rest of the hand. How should this situation be
managed?
set 5_set 5.qxd 22-04-2013 19:21 Page 137

Set 5 questions

a. Conversion to general anaesthesia.


b. Repeat the axillary brachial plexus block with the same dose of local
anaesthetic.
c. Fentanyl 1µg/kg by intravenous injection.
d. Infiltration of 5ml 0.5% levobupivacaine at the elbow between the
brachioradialis and the biceps tendon.
e. Midazolam 2-3mg by intravenous injection.

21 A 42-year-old male has been suffering from radicular pain in the L4


nerve distribution of his right leg. He had a discectomy at the L4/L5

137
level 2 years ago; his pain, however, still remains a significant
problem. He has tried neuropathic and opioid medications, TENS, a
lumbar epidural and physiotherapy without satisfactory pain relief.
TENS relieves his pain for a short duration only. He works as a
security officer and is quite keen to continue working. His family and
employer are sympathetic and supportive. What would be the next
most suitable therapy for his pain?

a. Cognitive behavioural therapy.


b. Physiotherapy.
c. Intrathecal drug delivery using opioids.
d. Spinal cord stimulation.
e. Acupuncture.

22 A 73-year-old man is undergoing an emergency laparotomy for a


perforated duodenal ulcer. His blood pressure is low at 73/46mm
Hg, and his heart rate is 103 bpm. Which one of these clinical
measurements would suggest that he is likely to respond to a fluid
bolus?

a. Central venous pressure of 9mmHg.


b. Pulmonary capillary wedge pressure of 16mmHg.
c. Stroke volume variation of 12%.
d. Urine output of 18ml in the last hour.
e. Capillary refill time of 5 seconds.
set 5_set 5.qxd 22-04-2013 19:21 Page 138

Single Best Answer MCQs in Anaesthesia

23 A 63-year-old woman is receiving a continuous epidural infusion of


local anaesthetic for analgesia following gastrectomy. It is the
second postoperative day. She is pain-free but complains of loss of
sensation in her legs as well as motor block. On examination she has
a sensory block from T4-S5 on both sides. There is a dense motor
block in both lower limbs. Her temperature is 36.5°C. The clinical
record reveals that the patient is receiving 20mg of enoxaparin once
daily and her INR is 1.1. What should be your next course of action?

a. Reassure the patient that everything is normal.


b. Stop the epidural infusion and reassess after 2 hours.
138 c. Give 2 units of fresh frozen plasma.
d. Prescribe intravenous flucloxacillin 2g 6-hourly.
e. Arrange an immediate MRI scan of the spine.

24 A 24-year-old female presents to the emergency department with


confusion, tachycardia and convulsions gradually worsening for the
past 2 hours. She has a history of drug abuse and has been recently
started on citalopram for depression. Which of the following would
be the most appropriate immediate management?

a. Lorazepam 4mg I.V.


b. Activated charcoal.
c. Cyproheptadine.
d. 225mmol of 8.4% sodium bicarbonate.
e. Haemodialysis.

25 A 3-year-old boy is brought to the accident and emergency


department by his parents with a 1-week history of malaise and
excessive thirst. He is drowsy but responsive to voice. His
respiratory rate is 50 breaths per minute, heart rate is 180 bpm and
BP is 70/40mm Hg. Initial investigations reveal serum glucose
25mmol/L, sodium 130mmol/L, potassium 4.0mmol/L, pH 7.08 and
lactate 3.1mmol/L. The appropriate initial treatment is:
set 5_set 5.qxd 22-04-2013 19:21 Page 139

Set 5 questions

a. Intravenous insulin bolus.


b. Intravenous bicarbonate.
c. 20ml/kg Hartmann's solution.
d. 10ml/kg of 0.9% saline.
e. Mannitol 0.5g/kg.

26 A 28-year pregnant woman in her first trimester undergoes surgery


for a left tibial fracture following a road traffic accident. On the third
postoperative day she develops swelling and pain in the right calf. A
deep vein thrombosis (DVT) is suspected and confirmed following a
139
Doppler scan. What is the most appropriate treatment for this
condition in this patient?

a. 5000 units of unfractionated heparin subcutaneously twice a day.


b. Intravenous heparin infusion.
c. Oral warfarin therapy.
d. Placement of an inferior vena caval filter.
e. Enoxaparin 1mg/kg subcutaneously every 12 hours.

27 A 52-year-old woman is brought into the emergency department with


a history of shortness of breath and central chest pain. She was
discharged home 5 days before following a total knee replacement.
On clinical examination she is hypoxic and tachycardic and her BP
is 86/45mm Hg following infusion of 1L of Hartmann's solution.
Which of the following would be the most suitable diagnostic test for
pulmonary embolism?

a. Troponin.
b. Echocardiography.
c. D-dimers.
d. CT pulmonary angiogram.
e. Isotope lung scan.
set 5_set 5.qxd 02-05-2013 20:55 Page 140

Single Best Answer MCQs in Anaesthesia

28 An 82-year-old female patient is receiving a blood transfusion on her


first postoperative day following a right hemi-arthroplasty. Her
baseline observations are normal. Five minutes after the blood
transfusion is commenced her observations are as shown in Table 2.

Table 2. Results of observations.

Pulse rate BP Respiratory rate SpO2 Temp

140 132 bpm 85/45mm Hg 16 bpm 97% 38.8°C

What is the most likely cause of her condition?

a. Postoperative chest infection.


b. Pyrexia of unknown origin.
c. Transfusion-related acute lung injury.
d. ABO incompatability.
e. Non-haemolytic febrile transfusion reaction.

29 A 72-year-old gentleman is scheduled for a total knee replacement.


At the pre-operative assessment clinic, clinical examination reveals a
small scar below the left clavicle. On questioning, the patient admits
to having 'something' inserted in that area about 6 years ago. He
has no particular symptoms and could not remember what the
device was inserted for, but did confirm that it was checked to be in
working condition in the clinic last month. No further documentation
was available in the patient's notes. Which of the following is the
most important next step in this patient's peri-operative
management?

a. A plain chest radiograph should be performed to confirm the


presence of a pacemaker.
set 5_set 5.qxd 02-05-2013 20:55 Page 141

Set 5 questions’

b. The pacemaker clinic should be contacted to confirm the indication


for the pacemaker, type of pacemaker, and degree of pacemaker
dependency.
c. The diathermy plate should be placed as far as possible from the
chest.
d. A magnet should be placed over the pacemaker to ensure inhibition
of shock therapy.
e. Invasive blood pressure monitoring should be considered as an
alternative to ECG monitoring.

30 A 26-year-old primigravida had an epidural for labour analgesia


which was inserted uneventfully and remained in situ for 12 hours. 141
Seven days postpartum the patient presented to the accident and
emergency department with a low-grade fever, backache and
weakness of the legs. The results of all routine blood investigations
are within the normal range. The most likely diagnosis in this patient
is:

a. Epidural abscess.
b. Intervertebral disc prolapse.
c. Subdural haematoma.
d. Obstetric palsy.
e. Spinal arachnoiditis.
set 5_set 5.qxd 22-04-2013 19:21 Page 142

Single Best Answer MCQs in Anaesthesia

142
set 5 answers_set 5 answers.qxd 02-05-2013 21:47 Page 143

Set 5

Set 5 answers
answers

1 Answer: C. Fibromyalgia.
143
Fibromyalgia is characterised by chronic widespread pain and muscle
tenderness. Other associated symptoms include fatigue, poor sleep,
functional bowel disturbances, and joint stiffness. It is frequently
associated with psychiatric conditions such as anxiety and depression.

The diagnosis of fibromyalgia is difficult as, in most cases, laboratory


testing appears normal and many of the symptoms mimic those of other
rheumatic conditions such as arthritis or osteoporosis. The most widely
accepted criteria (stated by the American College of Rheumatology) for
diagnosing fibromyalgia are a history of widespread pain lasting more than
3 months affecting both sides of body, above and below the waist, and the
patient must feel pain at 11 or more tender points, of a designated 18
points on the surface of the body. Cognitive behavioural therapy (CBT),
along with exercise, provides the greatest benefit in this group of patients.

In muscular dystrophy, there is progressive, symmetrical skeletal muscle


weakness and wasting with intact sensation and reflexes. It is often
associated with mental retardation.

Polymyalgia rheumatica is characterized by pain and stiffness in proximal


muscles such as the pelvic and shoulder girdle, which is worse in the
morning. Clinical examination usually demonstrates absence of tenderness
and normal muscle strength. It is often associated with giant cell arthritis.
set 5 answers_set 5 answers.qxd 02-05-2013 21:47 Page 144

Single Best Answer MCQs in Anaesthesia

Further reading
1. Goldenberg DL. Multidisciplinary modalities in the treatment of
fibromyalgia. Journal of Clinical Psychiatry 2008; 69: 30-4.
2. Nochimson G. Polymyalgia rheumatica. (http://emedicine.medscape.
com/article/808755-overview).

2 Answer: C. Hypermagnesaemia.

Magnesium sulphate is used for seizure prophylaxis in severe pre-


eclampsia and is often continued postpartum for several hours. This

144
patient had features suggestive of hypermagnesaemia. The clinical
features range from nausea and vomiting to respiratory and cardiac arrest,
depending on the serum magnesium level. A marked reduction in tendon
reflexes indicates impending magnesium toxicity. Monitoring should
include the regular assessment of tendon reflexes. A marked depression
of patellar reflexes is an indication of impending magnesium toxicity. The
ECG may show a prolonged PR interval and intraventricular conduction
defects. Respiratory rate, tendon reflexes and urine output should be
monitored during magnesium therapy. If prolonged infusion or higher
doses are used, serum magnesium levels should be monitored to prevent
magnesium toxicity.

The classic ECG changes in pulmonary embolism include the S1Q3T3


pattern with a prominent S wave in lead I, and prominent Q and T waves
in lead III. Amniotic fluid embolism usually occurs during labour, and can
also occur during Caesarean section.

Further reading
1. Diseases associated with pregnancy. In: Anesthesia and co-existing
disease, 4th ed. Stoelting RK, Dierdorf SF, Eds. Philadelphia, USA:
Churchill Livingstone, 2002: 662.
2. Magnesium sulphate. In: Analgesia, anaesthesia and preganancy, a
practical guide. Yentis SM, Grighouse D, May A, Bogod D, Elton C,
Eds. London: W.B. Saunders, 2002: 198-200.
3. Novello NP, Blumstein HA. Hypermagnesemia. (http://emedicine.
medscape.com/article/766604-overview).
set 5 answers_set 5 answers.qxd 02-05-2013 21:47 Page 145

Set 5 answers

3 Answer: B. Physostigmine.

Atropine has numerous side effects including dizziness, nausea, blurred


vision, loss of balance, dilated pupils, photophobia, supraventricular or
ventricular tachycardia, ventricular fibrillation, and, notably in the elderly,
extreme confusion, dissociative hallucinations, and agitation. The CNS
effects of atropine are due to its ability to cross the blood brain barrier,
being a tertiary amine compound. The central anticholinergic symptoms
are relatively common in the elderly. These symptoms range in severity
from mild confusion to severe agitation, hallucinations, psychosis, seizures
and coma (‘anticholinergic syndrome’). Physostigmine is an
anticholinesterase which crosses the blood brain barrier and therefore 145
acts as an antidote to atropine. It is specifically indicated when the patient
develops tachydysrhythmias with subsequent haemodynamic
compromise, intractable seizures, severe agitation or psychosis.

Agitation and convulsions associated with anticholinergic toxicity can be


controlled with intravenous midazolam, but physostigmine is the most
specific antidote. Neostigmine does not cross the blood brain barrier and
therefore is not useful in the treatment of anticholinergic syndrome.

Haloperidol is a member of the butyrophenone group of antipsychotic


drugs used in the treatment of delirium, acute psychosis, and an adjunctive
treatment during alcohol and opioid withdrawal.

Clonidine is a a2 agonist; it can be used as a supplement for sedation in


intensive care for controlling agitation.

Further reading
1. Burns MJ, Linden CH, Graudins A, et al. A comparison of
physostigmine and benzodiazepines for the treatment of
anticholinergic poisoning. Ann Emerg Med 2000; 35: 374-81.

4 Answer: B. Legionella pneumophilia.

The most common pathogens causing community-acquired pneumonia


(CAP) are Streptococcus pneumoniae, Legionella pneumophilia and
set 5 answers_set 5 answers.qxd 02-05-2013 21:47 Page 146

Single Best Answer MCQs in Anaesthesia

Staphylococcus aureus. A history of recent travel should be an alert to the


possibility of Legionella pneumophilia, which is an aerobic gram negative
bacillus. It is present in natural habitats such as fresh water ponds and
lakes, and also reservoirs and artificial sources such as cooling towers
and air conditioning systems. It is most commonly seen in younger patients
and smokers. Patients may present with altered mental status, elevated
liver enzymes, and diarrhoea, in addition to multilobar pneumonia.

Further reading
1. Sadashivaiah B, Carr B. Severe community-acquired pneumonia.
British Journal of Anaesthesia CEACCP 2009; 9: 87-91.

5
146
Answer: A. Domperidone and ondansetron.

Parkinson’s disease is a neurological condition involving the


extrapyramidal system. It occurs due to loss of dopaminergic neurones in
the substantia nigra, causing an imbalance of acetylcholine and dopamine.
It is characterised by tremor, rigidity and postural instability. It is usually
treated using dopamine precursors (e.g. levodopa), dopamine agonists
(e.g. apomorphine), and MAO inhibitors (e.g. selegiline). Surgical
treatment (deep brain stimulation) is reserved for those patients suffering
from refractory disease with severe disablement.

Several drugs used during the peri-operative period may have an adverse
effect on Parkinsonism. Opiates are often necessary after major surgery,
but may worsen muscle rigidity. Pethidine should be avoided as it can
cause hypertension and muscle rigidity in patients on selegiline. Patients
may not be able to physically use patient-controlled analgesia (PCA).
Antipsychotics, e.g. phenothiazines and butyrophenones, used as enti-
emetics, may worsen symptoms of Parkinsonism, as they have
antidopaminergic actions. Propofol may have dopamine-like effects, and
thus helps to reduce tremor and muscle rigidity. Anticholinergic drugs
which cross the blood brain barrier, such as atropine, can precipitate
central anticholinergic syndrome. Glycopyrrolate is the anticholinergic of
choice. Anti-emetics, such as metoclopramide, droperidol and
prochlorperazine, may worsen the symptoms of Parkinsonism and cause
extra-pyramidal effects. The anti-emetic of choice is domperidone as it
set 5 answers_set 5 answers.qxd 02-05-2013 21:47 Page 147

Set 5 answers

does not cross the blood brain barrier and thus does not cause extra-
pyramidal effects. 5-HT3 antagonists (e.g. ondansetron, granisetron) and
cyclizine can safely be used.

Further reading
1. Nicholson G, Pereira AC, Hall GM. Parkinson’s disease and
anaesthesia. British Journal of Anaesthesia 2002; 89: 904-16.

6 Answer: C. Chemical lumbar sympathectomy.

147
Chemical sympatholysis is commonly performed for palmar or plantar
hyperhidrosis, Buerger’s disease, and critical lower limb ischaemia where
there is no revascularization option available for palliative treatment of the
pain.

The lumbar sympathetic chain lies on the anterolateral border of the


vertebral body. The sympathetic chain is separated from the main lumbar
sensory / motor nerves by the psoas muscle.

The block is performed using X-ray screening, intravenous sedation if


necessary, and local anaesthetic infiltration, with the patient in the prone or
lateral position. The needle is inserted about 8-10cm from the midline, and
advanced so that it initially touches the side of the L2 or L3 vertebral body.
It is then withdrawn slightly and readvanced until it slips past the
anterolateral border of the vertebral body. Radio-opaque dye is injected to
confirm the correct needle position. Complications include abdominal
organ puncture, bleeding due to aortic and inferior vena cava injury, and
genitofemoral neuralgia. Genitofemoral neuralgia is thought to be due to
bruising of the L1 nerve root by the needle passing by it. More than 90%
of cases recover spontaneously after 6 weeks. It is dangerous to perform
the block in the presence of a large aortic aneurysm. Intravascular injection
should be avoided by checking the needle tip position using a radio-
opaque dye.

Lumbar epidural and nerve root blocks are useful for treatment of radicular
pain in the legs but are not effective in ischaemic vascular pain. A superior
hypogastric plexus block may be effective for the management of pelvic
pain.
set 5 answers_set 5 answers.qxd 02-05-2013 21:47 Page 148

Single Best Answer MCQs in Anaesthesia

Further reading
1. Nesargikar P, Ajit M, Eyers P, et al. Lumbar chemical sympathectomy
in peripheral vascular disease: does it still have a role? International
Journal of Surgery 2009; 7: 145-9.

7 Answer: D. Needle decompression and insertion


of a chest drain.

Pneumothorax is a recognised complication of laparoscopy. Gas passes


from the peritoneum directly into the pleural space either through pre-

148
existing channels, or it may spread from the mediastinum along the bronchi
until it breaks through the weak spot into the pleural cavity. The distended
neck veins (increased CVP), hypotension and desaturation suggest
development of a tension pneumothorax. The specific treatment is needle
decompression followed by insertion of a chest drain.

Increasing minute ventilation may improve the desaturation due to


hypoventilation. An endotracheal tube blocked by a mucous plug can
result in high airway pressures and desaturation which may be improved
by endotracheal suction. PEEP is useful in improving oxygenation due to
atelectasis.

Further reading
1. Farn J, Hammerman A, Brunt LM. Intraoperative pneumothorax during
laparoscopic cholecystectomy: a complication of prior
transdiaphragmatic surgery. Surgical Laparoscopy Endoscopy &
Percutaneous Techniques 1993; 3: 219-22.
2. Complications and contraindications of laparoscopic surgery. In:
Anaesthesia for minimally invasive surgery. Crozier TA, Ed.
Cambridge: Cambridge University Press, 2004.

8 Answer: B. Small cell carcinoma of the lung.

The clinical features in this patient are suggestive of bronchial carcinoma.


The syndrome of inappropriate ADH secretion (SIADH) from the posterior
pituitary is a recognised manifestation of para-neoplastic syndrome
set 5 answers_set 5 answers.qxd 02-05-2013 21:47 Page 149

Set 5 answers

associated with bronchial carcinoma (more commonly with small cell


carcinoma). SIADH is one of the most common causes of hyponatraemia.

Other endocrine disorders associated with lung cancer include increased


ACTH secretion, hyperparathyroidism and carcinoid syndrome.

Cerebral salt wasting syndrome causes hyponatraemia, and is seen in


patients with traumatic brain injury. Both hypothyroidism and adrenal
insufficiency can cause hyponatraemia but in this patient, the clinical
history does not suggest these as a cause. SIADH secretion can also be
associated with respiratory inflammatory diseases and infections such as
tuberculosis, pneumonia, asthma and lung abscess.
149
Further reading
1. Craig S. Hyponatremia. (http://emedicine.medscape.com/article/
767624-overview).
2. Reid PT, Innes JA. Respiratory disease. In: Davidson’s principles and
practice of medicine, 20th ed. Boon AN, Colledge NR, Walker BR,
Hunter JAA, Eds. London: Churchill Livingstone, 2006; Chapter19:
705-11.
3. Ellison DH, Berl T. Clinical practice. The syndrome of inappropriate
antidiuresis. N Engl J Med 2007; 356: 2064-72.

9 Answer: E. Guillain-Barré syndrome.

Guillain-Barré syndrome is an acute inflammatory demyelinating


polyneuropathy affecting the peripheral nervous system. It is usually
triggered by an acute infection. Most patients present with an acute
neuropathy with ascending paralysis, hyporeflexia or areflexia, and raised
protein concentrations in CSF.

Myasthenia gravis is an autoimmune disease affecting the neuromuscular


system characterised by fluctuating muscle weakness commonly involving
voluntary muscles, often the facial and eye muscles. The reflexes are not
lost and the weakness generally improves with rest. The diagnosis can be
confirmed by the edrophonium test.
set 5 answers_set 5 answers.qxd 02-05-2013 21:47 Page 150

Single Best Answer MCQs in Anaesthesia

Transverse myelitis is a neurological disorder caused by inflammation


across both sides of one level, or segment, of the spinal cord. Symptoms
of transverse myelitis include a loss of spinal cord function occurring over
several hours to several weeks. It usually begins as a sudden onset of
lower back pain, muscle weakness, or abnormal sensations in the toes
and feet, and can rapidly progress to more severe symptoms, including
paralysis, urinary retention, and loss of bowel control. Bladder and bowel
dysfunction is common. Many patients also report muscle spasms, a
general feeling of discomfort, headache, fever, and loss of appetite. Pain
is the primary presenting symptom of transverse myelitis in approximately
one third to one half of all patients. Severe electrolyte disturbances can

150
cause generalised muscle weakness. EMG and neurological studies are
normal.

Motor neurone disease (MND) is a progressive neurodegenerative


disease that attacks the upper and lower motor neurones. Degeneration of
the motor neurones leads to weakness and wasting of muscles, causing
increasing loss of mobility in the limbs, and difficulties with speech,
swallowing and breathing.

Further reading
1. Hughes RAC, Cornblath DR. Guillain-Barré syndrome. Lancet 2005;
366: 1653-66.

10 Answer: C. Subarachnoid block.

Complications of cervical plexus block include subarachnoid block,


epidural block, intravascular injection resulting in local anaesthetic toxicity,
phrenic nerve palsy, recurrent laryngeal nerve palsy, Horner’s syndrome,
haematoma and airway obstruction. Subarachnoid block may also occur
during retrobulbar block, interscalene brachial plexus block and stellate
ganglion block.

Epidural block is a possible complication but it is slower in onset and


unlikely to produce profound hypotension. Accidental intravenous injection
of local anaesthetic agent may result in local anaesthetic toxicity. The
clinical features of local anaesthteic toxicity include tinnitus, circumoral
set 5 answers_set 5 answers.qxd 02-05-2013 21:47 Page 151

Set 5 answers

numbness, seizures, respiratory depression, apnoea, bradycardia,


conduction abnormalities and hypotension. As this patient recovered
rapidly, this is unlikely to be local anaesthetic toxicity. Phrenic nerve palsy
may result in shortness of breath due to diaphragmatic weakness.

Further reading
1. Carling A, Simmonds M. Complications from regional anaesthesia for
carotid endarterectomy. British Journal of Anaesthesia 2000; 84: 797-
800.
2. Ross S, Scarborough CD. Total spinal anaesthesia following brachial
plexus block. Anesthesiology 1973; 39: 458.

151
3. Edde RR, Deutsch S. Cardiac arrest after interscalene brachial plexus
block. Anesthesia Analgesia 1977; 56: 446-7.

11 Answer: C. Sub-Tenon’s block.

Anaesthesia for cataract surgery can be provided using local and general
techniques. Local anaesthetic techniques can be divided into superficial
eye blocks such as sub-Tenon’s, or topical anaesthesia alone, and deep
eye blocks via the peribulbar and retrobulbar routes.

Prophylactic anticoagulation is common in patients presenting for cataract


surgery, and there may be significant risks arising from cessation of such
therapy. Topical anaesthesia or sub-Tenon’s block can be safely used in
the anticoagulated patient. Since this patient has COAD, it would be wise
to avoid a general anaesthetic. With peribulbar and retrobulbar blocks the
risk of retrobulbar haemorrhage is quite significant. Patients with myopia
are likely to have an axial length of more than 25mm and are at risk of globe
perforation during retrobulbar block.

Further reading
1. Kumar C, Dodds C. Ophthalmic regional block. Ann Acad Med
Singapore 2006; 35: 158-67.
set 5 answers_set 5 answers.qxd 02-05-2013 21:47 Page 152

Single Best Answer MCQs in Anaesthesia

12 Answer: D. Isoprenaline.

The transplanted heart does not receive autonomic input because it is


denervated during harvesting. It does not respond to extrinsic neural
signals although intrinsic myocardial mechanisms and reflexes are intact.
It responds to circulating catecholamines and directly acting
sympathomimetic agents. Vagolytic drugs such as atropine and
glycopyrrolate will not increase the heart rate. Ephedrine and
norephedrine are indirectly-acting sympathomimetic drugs and do not
have any response on the denervated heart. Isoprenaline is a potent
synthetic directly-acting catecholamine. It stimulates both b1 and b2
152 receptors. It increases the heart rate, myocardial contractility and cardiac
output.

Further reading
1. Stone ME. Non-cardiac surgery after heart transplantation. In: Clinical
cases in anaesthesia, 3rd ed. Reed AP, Yudkowitz FS, Eds.
Philadelphia, USA: Elsevier Churchill Livingstone, 2005; Case 12: 59-
64.

13 Answer: B. Giant unilateral lung cyst.

Absolute indications for lung isolation include a giant unilateral lung cyst or
bulla, broncho-pleural fistula, open surgery on the main bronchus,
unilateral massive haemorrhage and infection in one of the lungs where
contamination of the other lung is to be avoided. The use of a double-
lumen tube facilitates the ventilation of the normal lung in conditions such
as a unilateral lung cyst and bulla, thereby avoiding barotrauma. Relative
indications for lung isolation include lobectomy, thoracic spinal surgery,
thoracic aneurysm repair and pneumonectomy.

Further reading
1. Eastwood J, Mahajan R. One lung anaesthesia. British Journal of
Anaesthesia CEACCP 2002; 2: 83-7.
set 5 answers_set 5 answers.qxd 02-05-2013 21:47 Page 153

Set 5 answers

14 Answer: D. Nitroglycerine and dobutamine.

The clinical picture is suggestive of pulmonary hypertension. The ideal


drug combination in this patient is one which increases myocardial
contractility, and decreases the pulmonary and systemic vascular
resistance. A combination of dobutamine and nitroglycerine would be the
most suitable. Dobutamine has agonist action on b1 receptors, acts as a
positive inotrope and therefore increases the cardiac output.
Nitroglycerine reduces pulmonary vascular resistance and systemic
vascular resistance and thereby reduces pulmonary congestion and
augments cardiac output. Noradrenaline will increase the systemic
vascular resistance and may worsen the pulmonary congestion. 153
Propranolol will further increase the pulmonary vascular resistance by its
b-antagonist action and also will reduce cardiac contractility.

Further reading
1. Rudarakanchana N, Trembath RC, Morrell NW. New insights into the
pathogenesis and treatment of primary pulmonary hypertension.
Thorax 2001; 56: 888-90.

15 Answer: B. Transoesophageal echocardiography.

The ECG is most commonly used to detect intra-operative ischaemia and


rhythm disturbances but the ST segment analysis on standard monitors
has poor sensitivity and specificity. Transoesophageal echocardiography is
the most sensitive monitor for detection of myocardial ischaemia. It
demonstrates regional wall motion abnormalities, an early sign of
myocardial ischaemia. These changes are seen before the ECG changes
develop. Pulmonary artery diastolic pressures increase during myocardial
ischaemia. Dipyridamole-thallium scanning may be used to detect peri-
operative myocardial perfusion defects. There are, however, limited data
on the use of this technique during the intra-operative period.

Further reading
1. Shore-Lesserson LJ. Coronary artery bypass grafting. In: Clinical cases
in anaesthesia, 3rd ed. Reed AP, Yudkowitz FS, Eds. Philadelphia,
USA: Elsevier Churchill Livingstone, 2005; Case 13: 65-8.
set 5 answers_set 5 answers.qxd 02-05-2013 21:47 Page 154

Single Best Answer MCQs in Anaesthesia

2. Edwards ND, Reilly CS. Detection of peri-operative myocardial


ischaemia, review article. British Journal of Anaesthesia 1994; 72:
104-15.

16 Answer: D. Lidocaine 5% plasters.

The clinical presentation of this patient’s pain, her age, and recent course
of steroids, all indicate that she is suffering from post-herpetic neuralgia
(PHN). The reactivation of the Herpes zoster virus occurs due to
immunosuppression and therefore is more commonly seen in patients with
old age, poor nutrition, malignancy, and immunosuppression due to any
154 cause. PHN is treated using lidocaine 5% plasters, tricyclic
antidepressants (amitriptyline), calcium channel blockers (gabapentin,
pregabalin), sodium channel blockers (phenytoin and carbamazepine) and
non-pharmacological therapies such as TENS, acupuncture and cognitive
behavioural therapy (CBT). In this particular patient, lidocaine 5% plasters
would be most appropriate. The soothing effect of the plaster will minimise
the pain from mechanical allodynia and will also provide local analgesia
without any systemic side effects. This patient has multiple comorbidities
so it would be wise to avoid any systemic therapy.

Further reading
1. Guy H, et al. Efficacy and tolerability of a 5% lidocaine medicated
plaster for the topical treatment of post-herpetic neuralgia: results of a
long-term study. Current Medical Research and Opinion 2009; 25:
1295-305.

17 Answer: D. Ruptured valve cusp of the mitral


valve.

Infective bacterial endocarditis is most commonly observed in adults, but


the incidence in children with congenital heart disease or central
indwelling venous catheters continues to rise.

Patients with acute bacterial endocarditis present with an acute, toxic,


febrile illness and symptoms that have lasted less than 2 weeks. There
set 5 answers_set 5 answers.qxd 02-05-2013 21:47 Page 155

Set 5 answers

may be a history of intravenous drug use. Staphylococcus aureus is the


most common cause of acute bacterial endocarditis. Patients with
subacute bacterial endocarditis present with more non-specific flu-like
symptoms that have lasted more than 2 weeks. Subacute bacterial
endocarditis is more common in patients with an underlying congenital
heart defect, a bicuspid aortic valve being the most common example.
Staphylococcus aureus is the primary organism causing endocarditis.
Other organisms include Streptococcus viridans, Streptococcus
intermedius and Pseudomonas aeruginosa.

Acute heart failure may occur due to valve destruction or distortion and/or
rupture of the chordae tendinae. Chronic heart failure may be due to
progressive valvular insufficiency with worsening ventricular function. 155
Heart failure with aortic insufficiency is associated with a high mortality
rate.

Further reading
1. Bayer AS, Bolger AF, Taubert KA, et al. Diagnosis and management
of infective endocarditis and its complications. Circulation 1998; 98:
2936-48.

18 Answer: A. Rapid sequence induction,


intubation and commencement of positive
pressure ventilation.

The clinical information along with the raised respiratory rate and low PaO2
suggests the diagnosis of acute life-threatening asthma. Appropriate first-
line treatment has already been started, but there are no signs of a
response.

There is little good quality evidence regarding drug therapy for the further
management of severe asthma. The use of aminophylline is limited due to
its side effects (arrhythmias, restlessness, vomiting, and convulsions),
related to a narrow therapeutic window. Intravenous salbutamol has no
benefit compared to nebulised salbutamol provided administration of
nebulisers is not a problem. In practice this means that intravenous b2
agonists should generally be reserved for patients who are ventilated.
set 5 answers_set 5 answers.qxd 02-05-2013 21:47 Page 156

Single Best Answer MCQs in Anaesthesia

There is some evidence that magnesium sulphate may be of benefit in


severe asthma, and importantly has few side effects when given as a
single dose. Although further research is undoubtedly required, the best
supporting evidence for all second-line agents suggests the use of
magnesium.

Intubation and ventilation of patients with severe asthma is difficult and can
have deleterious effects both at the time of intubation and during
mechanical ventilation. It should be used in the presence of life-threatening
hypoxia, cardiac or respiratory arrest or after all other medical treatment
has failed.

156 Heliox (79% helium and 21% oxygen) will reduce the work of breathing in
acute upper airway obstruction by improving the turbulent flow due to low
density of helium. It has been suggested in severe asthma but it limits the
FiO2 that can be achieved and has no role in acute life-threatening asthma.

Further reading
1. British guideline in the management of asthma. British Thoracic
Society, 2009.
2. Stanley D, Tunnicliffe W. Management of life-threatening asthma in
adults. British Journal of Anaesthesia CEACCP 2008; 8: 95-9.

19 Answer: A. Intravenous infusion of mannitol


0.5g/kg.

Mannitol is an osmotic diuretic that reduces ICP by drawing fluid from the
brain. It has been shown to improve surgical access and reduce ICP
within a few minutes of administration. It can also temporarily improve
cerebral perfusion by increasing intra-vascular volume.

Hyperventilation is a recognised method of reducing ICP as cerebral


blood flow follows an almost linear relationship with PaCO2. However,
hyperventilation to sub-normal levels of CO2 can cause significant
vasoconstriction and has the potential to cause ischaemia in vulnerable
brain tissue. The PaCO2 should be maintained >4.0kPa unless other
methods of reducing ICP have failed.
set 5 answers_set 5 answers.qxd 02-05-2013 21:47 Page 157

Set 5 answers

Steroids have no role in reducing ICP in acute head injury; they are only
useful in reducing the bulk of tumours and intracranial infective lesions. A
large-scale study has found evidence of increased mortality with the
administration of steroids in acute head injury.

Hypertension and hypervolaemia are important in order to maintain


cerebral perfusion in the presence of cerebral vasospasm, but other
treatments will be needed to reduce intracranial pressure.

Further reading
1. Mishra LD, Rajkumar N, Hancock SM. Current controversies in
157
neuroanaesthesia, head injury management and neurocritical care.
British Journal of Anaesthesia CEACCP 2006; 6: 79-82.

20 Answer: D. Infiltration of 5ml 0.5%


levobupivacaine at the elbow between the
brachioradialis and the biceps tendon.

In this case the axillary nerve block has spared part of the radial nerve
distribution. The patient is very keen to have surgery performed under a
regional block so general anaesthesia should be used only if other
measures fail.

Repeating the axillary block has a high chance of being successful,


although response to nerve stimulation will be unreliable; this could be
performed under ultrasound guidance. However, by performing the block
again with the same amount of levobupivacaine, the total dose will exceed
the maximum safe dose.

Nerve blocks at the elbow, while unlikely to provide adequate anaesthesia


alone, are useful to supplement a problematic brachial plexus block.
Blocking the radial nerve at the elbow will most likely resolve the problem
and allow the procedure to be performed.
set 5 answers_set 5 answers.qxd 02-05-2013 21:47 Page 158

Single Best Answer MCQs in Anaesthesia

21 Answer: D. Spinal cord stimulation.

This patient is suffering from failed back surgery syndrome (post-


laminectomy syndrome). This manifests as a combination of chronic
neuropathic and nociceptive pain mainly involving the legs and/or the
lower back following successful spinal surgery. This patient’s history
suggests minimal psycho-social issues and therefore spinal cord
stimulation would be the most appropriate next step in his treatment. A
spinal cord stimulator (SCS) provides pulsed electrical signals to the
spinal cord to relieve pain from various conditions. In the simplest form, it
consists of stimulating electrodes implanted in the extradural space, an
158 implanted electrical pulse generator, wires connecting the electrodes to
the generator, and the generator remote control.

Electrotherapy of pain by neurostimulation was first used shortly after


proposal of the gate control theory. The theory proposed that nerves
carrying painful peripheral stimuli and nerves carrying touch and vibratory
sensation both terminate in the dorsal horn (the gate) of the spinal cord. It
was hypothesized that input to the latter could be manipulated in order to
‘close’ the gate. The mechanisms of action of spinal cord stimulation in
relieving neuropathic pain may be very different from that involving
ischaemic pain. In neuropathic pain, evidence shows that SCS alters the
local neurochemistry in the dorsal horn, suppressing the hyper-excitability
of the neurones. There is some evidence that there are increased levels of
serotonin and GABA, and suppressed levels of excitatory amino acids
such as glutamate. In the case of ischaemic pain, analgesia seems to be
mediated by inhibition of the sympathetic nervous system and
vasodilatation. SCS is used mostly in the treatment of radicular pain in the
lower limb (failed back surgery syndrome), complex regional pain
syndrome affecting the limbs, and ischaemic pain (e.g. ischaemic leg pain,
refractory angina).

Further reading
1. Oakley J, Prager J. Spinal cord stimulation: mechanism of action.
Spine 2002; 27: 2574-83.
set 5 answers_set 5 answers.qxd 02-05-2013 21:47 Page 159

Set 5 answers

22 Answer: C. Stroke volume variation of 12%.

Although central venous pressure (CVP) is widely used for assessing fluid
status, in reality it is a poor guide. A single reading of CVP bears little
relationship to the right ventricular end-diastolic volume, and as such does
not measure the degree of filling. The same is true of the pulmonary
capillary occlusion pressure as an assessment of left ventricular filling.
These non-linear relationships are thought to be due to dynamic changes
in ventricular wall compliance.

A low urine output may be suggestive of hypovolaemia, but on its own is

159
not diagnostic. There are several other possible causes of acute renal
failure in the sick surgical patient. Equally, prolonged capillary refill time
represents poor peripheral perfusion, but this is not necessarily due to
hypovolaemic shock.

Stroke volume variation can be measured by pulse contour analysis or by


Doppler ultrasound of the aorta. In clinical trials a stroke volume variation
of >9.5% has been shown to be highly predictive of a response to a fluid
bolus.

Further reading
1. Eyre L, Breen A. Optimal volaemic status and predicting fluid
responsiveness. British Journal of Anaesthesia CEACCP 2010; 10:
59-62.

23 Answer: B. Stop the epidural infusion and


reassess after 2 hours.

The differential diagnosis in this case includes excessive administration of


an epidural local anaesthetic, an epidural abscess and an epidural
haematoma. The latter two are serious complications and must be acted
upon as soon as they are diagnosed. They are, however, much less likely
than an excessive block so this should be ruled out first. It would be
appropriate to stop the infusion first and reassess within a fairly short time
period. If there is no sign of the block receding then an immediate imaging
and neurosurgical referral should be arranged.
set 5 answers_set 5 answers.qxd 02-05-2013 21:47 Page 160

Single Best Answer MCQs in Anaesthesia

A normal temperature makes an abscess unlikely, although it cannot be


ruled out. It would also be unlikely for an abscess to form this soon after
insertion. It is unnecessary to start antibiotics without objective evidence
of abscess formation. Equally, the facts in this case do not entirely support
the diagnosis of an epidural haematoma; a prophylactic dose of low-
molecular-weight heparin is unlikely to cause haematoma formation and
the INR of 1.1 is within the normal range. There is no indication for the
administration of blood products or clotting factors in this case.

Further reading
1. Grewal S, Hocking G, Wildsmith JA. Epidural abscesses. British
Journal of Anaesthesia 2006; 96: 292-302.

24
160

Answer: A. Lorazepam 4mg I.V.

The clinical features and history of drug abuse, and the intake of
citalopram suggests serotonin syndrome. This is characterised by the triad
of altered level of consciousness, neuromuscular hyperactivity and
autonomic instability. The treatment is mainly supportive and should follow
an ABC approach. Benzodiazepines or phenytoin are used for the
management of convulsions. Activated charcoal is useful but only within
the first hour. Cyproheptadine and chlorpromazine have been used but
there is no evidence to support their use. Alkalinization of urine is useful in
reducing the incidence of renal failure.

Further reading
1. Ward W, Sair M. Oral poisoning: an update. British Journal of
Anaesthesia CEACCP 2010; 10: 6-11.

25 Answer: D. 10ml/kg of 0.9% saline.

This child has severe diabetic ketoacidosis (DKA) with signs of marked
intravascular fluid depletion (severe tachycardia, hypotension and raised
lactate). A cautious 10ml/kg fluid bolus should be given (10-20ml/kg 0.9%
saline over the first 1-2 hours) and the response assessed. Hypotonic
fluids should not be used. Children with DKA are at high risk of developing
cerebral oedema. DKA results in low 2,3-diphosphoglycerate (2,3 DPG),
set 5 answers_set 5 answers.qxd 02-05-2013 21:47 Page 161

Set 5 answers

which reduces the oxygen delivery to the tissues. This effect of low 2,3
DPG is opposed by a low pH which shifts the oxygen dissociation curve
to the right. Intravenous bicarbonate shifts the oxygen dissociation curve
to the left, and along with low 2,3 DPG it reduces the oxygen delivery to
the tissues. Hypertonic saline or mannitol should be given if cerebral
oedema is suspected. Insulin is best administered as a low dose I.V.
infusion rather than a bolus dose. A low-dose infusion of 0.1unit/kg/hour
should achieve an adequate steady state plasma level within an hour. An
I.V. bolus dose may increase the risk of cerebral oedema.

Further reading
1. Steel S, Tibby SM. Paediatric diabetic ketoacidosis. British Journal of
Anaesthesia CEACCP 2009; 9: 194-9. 161

26 Answer: E. Enoxaparin 1mg/kg subcutaneously


every 12 hours.

DVT carries considerable clinical risk during pregnancy, and treatment


should be started prior to the Doppler scan if the diagnosis has been made
on clinical grounds. Warfarin should be avoided during pregnancy due to
the risk of teratogenicity. As the clearance of heparin is increased during
pregnancy, the dose of enoxaparin in pregnancy is raised to 1.0mg/kg
twice daily. Monitoring should include anti-Xa activity (target 0.6-1.0
units/ml) and platelet count. Administration of enoxaparin should cease
once labour begins. The duration of treatment should normally be 6
months, but if started early in pregnancy then enoxaparin should continue
at the full treatment dose until 6 weeks post-delivery.

Further reading
1. Thromboembolic disease in pregnancy and the puerperium.
Guidelines and Audit Committee of RCOG, April 2001.

27 Answer: B. Echocardiography.

This patient has risk factors for and clinical features of a massive
pulmonary embolism (PE). Because of her haemodynamic compromise a
set 5 answers_set 5 answers.qxd 02-05-2013 21:47 Page 162

Single Best Answer MCQs in Anaesthesia

bedside echocardiogram would be the test of choice to confirm diagnosis


prior to consideration of thrombolytic therapy. A CT pulmonary angiogram
would be the initial investigation in a stable patient. D-dimers are only
recommended in patients with a low to intermediate clinical risk of PE.
Isotope lung scanning may be considered in stable patients with a normal
chest X-ray and no concurrent cardiopulmonary disease. Troponin may
rise and be a prognostic indicator but is not diagnostic.

Further reading
1. van Beek EJR, Elliot CA, Kiely DG. Diagnosis and initial treatment of
patients with suspected pulmonary embolism. British Journal of
Anaesthesia CEACCP 2009; 4: 119-24.
162 2. British Thoracic Society guidelines for the management of suspected
pulmonary embolism. Thorax 2003; 58: 470-83.

28 Answer: D. ABO incompatability.

This patient did not have respiratory distress or hypoxaemia; this makes
postoperative chest infection and transfusion-related acute lung injury
unlikely causes for this clinical presentation. A non-haemolytic febrile
transfusion reaction can develop up to several hours afterwards but is
more common around 30 minutes into the transfusion. It is very common
and only rarely leads to more severe symptoms such as hypotension,
vomiting and respiratory distress. ABO incompatability should be
suspected when the symptoms occur within a few minutes of commencing
transfusion. It is important that the diagnosis is made rapidly so that
transfusion is stopped immediately and supportive management is
commenced promptly.

Further reading
1. Serious Hazards of Transfusion Annual Report 2008.
2. Maxwell MJ, Wilson MJA. Complications of blood transfusion. British
Journal of Anaesthesia CEACCP 2006; 6: 225-9.
3. Blood transfusion and the anaesthetist. Red cell transfusion 2. AAGBI
guidelines, 2008.
4. British Committee for Standards in Haematology (BSCH). Guidelines
for administration of blood and blood components and the
management of transfused patients. Transfusion Medicine 1999; 9:
227-38.
set 5 answers_set 5 answers.qxd 02-05-2013 21:47 Page 163

Set 5 answers

29 Answer: B. The pacemaker clinic should be


contacted to confirm the indication for the
pacemaker, type of pacemaker, and degree
of pacemaker dependency.

In all planned surgical procedures, both the anaesthetist and the surgeon
should be made aware of the presence of a pacemaker. In this instance,
the patient is unable to give the indication for the use of the pacemaker nor
the device type; therefore, the pacemaker clinic should be contacted for
more clinical information. A chest X-ray will give some device information
but would not confirm whether the device has been checked recently.
Interference from monopolar diathermy remains a risk even if the operative 163
site is far from the pacemaker and this mode should only be used as a last
resort. Bipolar diathermy should be used whenever possible but
interference is still possible. A magnet will in many cases inhibit delivery of
shock therapy but this is not guaranteed and may vary between
manufacturers. In the presence of electromagnetic interference (EMI), the
magnet may alter the programmability of the pacemaker, resulting in
malfunction. Its use is not indicated for programmable pacemakers. In
addition to an ECG, an alternative method of monitoring heart rate should
be considered in the presence of pacemaker spikes on the ECG.

Further reading
1. Guidelines for perioperative management of patients with implantable
pacemakers or implantable cardioverter defibrillators. (http://www.
mhra.gov.uk/Safetyinformation/General safety information and advice).

30 Answer: A. Epidural abscess.

The classical presentation of an epidural abscess is a triad of fever,


backache and neurological deficit. Other signs or symptoms are epidural
site redness, tenderness and purulent discharge at the site of insertion.
Though these are the common symptoms, patients may present with just
one or two of the above symptoms, hence, a high index of suspicion is
necessary. The most common organism causing an epidural abscess is
Staphylococcus aureus, which tracks from the skin along the catheter site.
set 5 answers_set 5 answers.qxd 02-05-2013 21:47 Page 164

Single Best Answer MCQs in Anaesthesia

The symptoms of intevertebral disc prolapse can vary depending on the


location and severity. It can range from no pain to severe pain, radiating to
regions supplied by the affected nerve root. Other symptoms may include
sensory changes such as numbness, tingling, muscular weakness, and
paralysis and sphincter disturbances. The presence of fever should raise
suspicion of an epidural infection.

A subdural haematoma usually develops soon after the insertion of the


epidural or after removal of the catheter. Symptoms begin with local or
radicular back pain and compression of lumbar spinal roots may cause
cauda equina syndrome. Neurological deficit progresses over minutes to

164
hours.

Obstetric palsy can result in unilateral foot drop due to compression of the
lumbosacral plexus by the large foetus or by forceps-assisted delivery.
Maternal obstetric palsy is a common transient weakness in the
distribution of nerves anywhere along the lumbosacral plexus. The
condition can present commonly as meralgia paraesthetica (neuropathy of
the lateral femoral cutaneous nerve), femoral neuropathy or sacral
numbness. It occurs during the immediate postpartum period and
gradually improves over a period of weeks.

Local inflammation of the arachnoid mater resulting in fibrosis, adhesions


and scarring is known as arachnoiditis. The presence of blood in the
subarachnoid space due to traumatic puncture, a high concentration of local
anaesthetic (2% lidocaine, 0.75% bupivacaine), steroid injection and the
preservatives in local anaesthetics, such as methyl-paraben or prophyl-
paraben, can cause arachnoiditis. Usually the patient presents several
weeks to months after the procedure with a neurological deficit in the lower
extremity.

Further reading
1. Green LK, Paech MJ. Obstetric epidural catheter-related infections at
a major teaching hospital: a retrospective case series. International
Journal of Obstetric Anaesthesia 2010; 19: 38-43.
2. Sghirlanzoni A, Marrazzi R, Pareyson R, et al. Epidural anaesthesia
and spinal arachnoiditis. Anaesthesia 1989; 44: 317-21.
3. Aldrete JA. Neurological deficit and arachnoiditis following neuroaxial
anaesthesia. Acta Anaesthesiol Scand 2003; 47: 3-12.
set 6_set 6.qxd 22-04-2013 19:22 Page 165

Set 6 questions

Set 6
1 A 42-year-old female is being seen in the pre-operative assessment
clinic prior to an elective laparoscopic cholecystectomy. She gives a 165
history of shortness of breath and fatigue on minimal exertion.
Cardiovascular examination reveals a heart rate of 62 bpm, BP of
110/70mmHg and a mid-diastolic murmur. A postero-anterior view
of the chest X-ray shows straightening of the left heart border. What
is the most likely diagnosis?

a. Mitral stenosis.
b. Tricuspid stenosis.
c. Mitral regurgitation.
d. Aortic stenosis.
e. Pulmonary stenosis.

2 A previously fit and well 2-year-old boy, weighing 12kg, presents to


the emergency department with convulsions. He has been febrile
and irritable during the day and started to fit in the evening, and an
ambulance was called. Diazepam 6mg and paraldehyde 4.8ml were
given rectally en route to hospital by the paramedics. His
temperature is 38°C, and his SpO2, BP and blood glucose are
within normal limits. He is breathing spontaneously and maintaining
his airway, and is receiving high-flow oxygen. Despite the above
measures he has been fitting continuously for the last 20 minutes.
You have managed to secure venous access. Which of the below
would be the most appropriate medication to be given next?
set 6_set 6.qxd 22-04-2013 19:22 Page 166

Single Best Answer MCQs in Anaesthesia

a. Oral diazepam 0.5mg/kg.


b. Oral midazolam 0.5mg/kg.
c. Intravenous phenytoin 18mg/kg, given over 30 minutes.
d. Rapid sequence induction with thiopentone and suxamethonium.
e. Oral paracetamol 240mg.

3 You are called to the emergency department to assess a 35-year-old


man who has been admitted with a suspected drug overdose. On
examination, he appears confused and agitated and unable to give a
clear history. His temperature is 38.6°C, pulse rate is 120 bpm, BP

166
is 143/87mm Hg, respiratory rate is 18/minute and SpO2 is 97% in
room air. He also has a tremor, muscle weakness and hyper-reflexia.
Assuming the diagnosis of overdose is correct, which of the
following medications is most likely to be responsible for his
symptoms?

a. Aspirin.
b. Lorazepam.
c. Edrophonium.
d. Citalopram.
e. Amitriptyline.

4 A 27-year-old man has been sedated and ventilated in the intensive


care unit, following a head injury. Five days after admission it is
noticed that his serum sodium concentration has dropped to
125mmol/L. Further tests reveal a haemoglobin concentration of
15g/dL, a haematocrit of 44% and a urea of 14mmol/L. Plasma and
urine osmolarities are 295 and 600mosm/L, respectively. On clinical
examination he appears dehydrated and it is noticed that his
vasopressor requirements have increased. Which of the following is
the most appropriate treatment for this problem?

a. Fluid restriction to 1L/24hours.


b. Titrated doses of intravenous desmopressin.
c. Intravenous infusion of 0.9% saline solution.
d. Administration of furosemide 20mg intravenously.
e. Demeclocycline 900mg daily.
set 6_set 6.qxd 22-04-2013 19:22 Page 167

Set 6 questions

5 A 46-year-old woman is due to have an elective total abdominal


hysterectomy and oopherectomy for stage one endometrial cancer.
General anaesthesia with a transversus abdominis plane block and
postoperative patient-controlled analgesia have been planned.
Which of the following would be the best means of providing venous
thrombo-embolism (VTE) prophylaxis?

a. Anti-embolism stockings alone.


b. Mechanical VTE prophylaxis using a combination of anti-embolism
stockings and an intermittent pneumatic device.
c. Mechanical VTE prophylaxis and low-molecular-weight heparin for 3
days post-surgery. 167
d. Mechanical VTE prophylaxis and low-molecular-weight heparin
continued for 28 days post-surgery.
e. Low-molecular-weight heparin alone.

6 A 39-year-old patient, who has a 2-year history of opioid addiction,


has had a below-knee amputation following a road traffic accident.
On the third postoperative day, despite initial satisfactory pain
control, he starts to suffer from severe increasing generalized stump
pain. Increasing morphine dosage has been ineffective. Which one
of the following analgesic regimes is the most likely to be effective in
relieving his pain?

a. Epidural morphine infusion.


b. Intrathecal morphine infusion.
c. Intravenous ketamine infusion.
d. Intravenous morphine infusion.
e. Intravenous gabapentin infusion.

7 A 3-year-old boy known to be diabetic presents with irritability and


disorientation. His blood glucose is 12mmol/L and his urine is
positive for ketones. Which of the following is the most common
cause of death in a patient presenting with these clinical features?
set 6_set 6.qxd 22-04-2013 19:22 Page 168

Single Best Answer MCQs in Anaesthesia

a. Sepsis.
b. Cerebral oedema.
c. Arrhythmias.
d. Acute pancreatitis.
e. Hypokalaemia.

8 A 45-year-old female patient is scheduled for arthroscopy of the


shoulder joint. During pre-operative assessment, she gives a history
of cold intolerance and weight gain. On examination her heart rate is
58 per minute and blood pressure is 130/72mmHg. Her blood test

168
reveals an elevated TSH level and low T3 and T4 levels. Which of
the following is the most likely cause for her symptoms?

a. Pituitary failure.
b. Hypothalamic failure.
c. Iodine deficiency.
d. Autoimmune hypothyroidism.
e. Hypophysectomy.

9 A 21-year-old university student has been admitted to the intensive


care unit with a diagnosis of meningitis. He has no significant past
medical history and presents with acute confusion after onset of a
severe headache. He has been sedated and intubated because he
became extremely agitated on arrival to the emergency department.
A CT scan of the head is normal and the results of lumbar puncture
are shown in Table 1.

Table 1. Results of lumbar puncture.

Neutrophils Lymphocytes Protein Glucose

10/mm3 300/mm3 0.35g/L 5.9mmol/L


set 6_set 6.qxd 22-04-2013 19:22 Page 169

Set 6 questions

Which of the following organisms is most likely to have caused the


meningitis?

a. Neisseria meningitidis.
b. Streptococcus pneumoniae.
c. Haemophilus influenzae type B.
d. Myobacterium tuberculous.
e. Herpes simplex virus.

10 A 23-year-old male is admitted to the accident and emergency


169
department following a motorcycle accident. He is found to have a
GCS of 9/15. He is sedated, intubated and ventilated, and an urgent
CT scan is performed. Which of the following findings in the CT
scan would indicate the worst prognosis in the first 14 days?

a. Subarachnoid bleed.
b. Subdural haematoma.
c. Depressed skull fracture.
d. Obliteration of the third ventricle.
e. An extradural haematoma.

11 You are assessing a new patient in the pain clinic. He is a 49-year-


old patient who had an amputation of the right upper limb at the
mid-humeral level 3 years ago. He is complaining of pain in the
right wrist, which is associated with burning and a spasmodic
sensation. What would be the most appropriate initial therapy for
him?

a. Morphine sulphate.
b. Cognitive behavioural therapy.
c. Image-guided physiotherapy.
d. Gabapentin.
e. Stellate ganglion block.
set 6_set 6.qxd 22-04-2013 19:22 Page 170

Single Best Answer MCQs in Anaesthesia

12 A 23-year-old male undergoes a closed reduction of a fracture of his


right femur. In the immediate postoperative period he becomes
tachypneoic and confused. Which of the following findings most
strongly suggest fat embolism?

a. Axillary petechiae.
b. Emboli present in the retina.
c. Fat present in urine.
d. Fat globules present in the sputum.
e. Increasing ESR.

170
13 A 12-year-old girl with a history of cerebral palsy underwent insertion
of a cochlear implant into the right ear under general anaesthesia.
She had not received her morning dose of regular medications. The
procedure took 3 hours with no adverse events during the intra-
operative period and recovery. On return to the ward she
experienced nausea and one episode of vomiting. About 6 hours
later she became very disorientated and developed dystonia and
painful muscle spasms. Which of the following regular medications
would be the most likely to lead to these clinical features?

a. Ondansetron.
b. Diazepam.
c. Ibuprofen.
d. Sodium valproate.
e. Baclofen.

14 A 18-year-old female presents to the accident and emergency


department having taken about 50 tablets of paracetamol. She says
that she swallowed the tablets within the last 20 minutes. Which of
the following treatments would be the most effective in reducing
absorption of paracetamol?

a. Induced emesis.
b. Gastric lavage.
c. Activated charcoal.
d. N-acetyl cysteine.
e. Methionine.
set 6_set 6.qxd 22-04-2013 19:22 Page 171

Set 6 questions

15 A 24-year-old African man is scheduled for an urgent


appendicectomy. His blood results and observations during pre-
operative assessment are shown in Table 2.

Table 2. Observations and results of blood testing.


Observations

Heart rate BP Temperature SpO2

120 bpm 106/56mm Hg 38.2°C 95%


171
Blood results

Hb WCC Urea Creatinine

7.3g/dL 16.0 8.9mmol/L 143µmol/L

The most appropriate immediate management is:

a. Laparotomy.
b. Oxygen and intravenous Hartmann’s solution.
c. Sickledex testing.
d. Blood transfusion to achieve Hb >10g/dl.
e. Pre-optimisation in the anaesthetic room.

16 A 6-year-old child weighing 20kg is scheduled on a day-surgery list


for circumcision. He is mildly asthmatic and has had ibuprofen in the
past. His mother is quite worried about the postoperative pain.
Which one of the following is most likely to provide adequate pain
relief safely?

a. Caudal block using 10ml of 0.5% bupivacaine.


b. Penile block and regular paracetamol and ibuprofen.
c. Regular ibuprofen and 4-hourly Oramorph for the first 24 hours.
d. Regular paracetamol and 4-hourly Oramorph for the first 24 hours.
e. Caudal block using 0.125% bupivacaine and ketamine 2mg/kg.
set 6_set 6.qxd 22-04-2013 19:22 Page 172

Single Best Answer MCQs in Anaesthesia

17 A 32-year-old female is scheduled for a laparoscopic


cholecystectomy. Her medical history includes long QT syndrome
which is treated with bisoprolol 5mg per day. In the past she
suffered a cardiac arrest and now has a history of recurrent
syncope. Which of the following should be the next step in her pre-
operative management?

a. Increasing the dose of bisoprolol.


b. Antibradycardia pacing.
c. Intravenous magnesium sulphate 2g.
d. Insertion of an automatic implantable cardioverter defibrillator
172 (AICD).
e. Oral amiodarone 300mg per day.

18 A 60-year-old ASA1 male patient is undergoing a radical


prostatectomy under general anaesthesia. During the procedure the
estimated total blood loss is 2L. He receives 4 units of blood and 4
units of fresh frozen plasma over a period of 30 minutes. About 2
hours following the blood transfusion, the patient develops
tachycardia and hypotension requiring inotropic support with an
epinephrine infusion. The peak airway pressure increases from 15 to
35cm H2O. Clinical examination reveals bilateral lung crepitations.
The most likely cause for his clinical deterioration is:

a. Sepsis.
b. Volume overload.
c. Myocardial infarction.
d. Transfusion-related acute lung injury (TRALI).
e. Cardiogenic shock.

19 A 62-year-old man collapses on the third postoperative day after


undergoing an oesophagectomy. His medical history includes
hypertension, angina, chronic smoking, and type II diabetes. His vital
parameters after collapse are shown in Table 3.
set 6_set 6.qxd 22-04-2013 19:22 Page 173

Set 6 questions

Table 3. Vital parameters.

Temperature BP Pulse rate CVP

39.2°C 82/30mm Hg 114 bpm 2mmHg

Which of the following is the most likely cause of this clinical


presentation?
173
a. Septicaemia.
b. Myocardial infarction.
c. Severe dehydration.
d. Haemorrhage.
e. Cardiac tamponade.

20 A 35-year-old male patient is scheduled for a laparoscopic


cholecystectomy. He gives a history of muscular weakness affecting
his breathing. He also has a history of obstructive sleep apnoea.
Clinical examination reveals frontal baldness, ptosis and an inability
to relax his hand grip. His muscle tone is increased significantly by
exercise and cold. Which of the following pre-operative
investigations would be the most important one to perform given this
patient’s condition?

a. Serum electrolytes.
b. Haemoglobin.
c. Ultrasound of his gall bladder.
d. Electromyography.
e. ECG.
set 6_set 6.qxd 22-04-2013 19:22 Page 174

Single Best Answer MCQs in Anaesthesia

21 A 6-week-old baby boy is admitted to hospital with projectile


vomiting. Pyloric stenosis is diagnosed and he is scheduled for
pyloromyotomy. He was born 2 weeks prematurely. Which of the
following would provide the best postoperative analgesia in this
patient?

a. Local infiltration and regular paracetamol.


b. Local infiltration and regular ibuprofen.
c. Local infiltration and morphine infusion.
d. Local infiltration and Oramorph.
e. Epidural analgesia.

22 Which
174

of the following surgical procedures has the highest


incidence of postoperative respiratory complications?

a. Abdominal aortic surgery.


b. Peripheral vascular surgery.
c. Abdominal surgery for bowel resection.
d. Neurosurgery.
e. Major head and neck surgery.

23 A 45-year-old female patient is scheduled for arthroscopy of the


knee joint. During the pre-operative assessment she gives a history
of sweating, dizziness and visual disturbances during the previous 2
months. During the previous week she had two such episodes
lasting for a few minutes. She denies any history of diabetes or any
other medical illness. She is not taking any regular medication. A
random blood glucose level in the pre-operative clinic is 2.5mmol/L.
The next most useful investigation in establishing the clinical
diagnosis in this patient is:

a. Fasting blood glucose.


b. Urine ketone body levels.
c. Plasma insulin level.
d. CT scan of the abdomen.
e. MRI scan of the abdomen.
set 6_set 6.qxd 22-04-2013 19:22 Page 175

Set 6 questions

24 A 55-year-old woman presents to the emergency department with


chest pain and shortness of breath. She was discharged from the
hospital a week ago following fixation of a fractured ankle. On
examination she has a respiratory rate of 30/minute, with a SpO2 of
93% on 15L/min of oxygen. Her heart rate is 117 bpm and blood
pressure is 110/46mm Hg. An ECG shows T-wave inversion in lead
III. A bedside transthoracic echocardiogram suggests raised
pulmonary artery pressure and right ventricular strain. Which of the
following would be the most appropriate immediate management?

a. Enoxaparin 1.5mg/kg administered subcutaneously.


b. Urgent Doppler ultrasound of the leg veins. 175
c. Alteplase 50mg intravenous bolus.
d. Warfarin 10mg orally.
e. Aspirin 300mg and clopidogrel 300mg orally.

25 A 45-year-old female patient with myasthenia gravis underwent a


laparoscopic cholecystectomy. Her pre-operative medications
included pyridostigmine 440mg and prednisolone 10mg per day. At
induction vecuronium 0.05mg/kg was administered which resulted
in loss of twitch responses to train of four stimulation. At the end of
surgery neuromuscular blockade was reversed with neostigmine
and glycopyrrolate. The trachea was extubated following
confirmation of adequate reversal of blockade by both nerve
stimulation and clinical signs. On transfer to recovery the patient
develops muscle weakness and hypoventilation. Which of the
following would be the most useful drug in deciding further
management of this patient?

a. Neostigmine.
b Edrophonium.
c. Pyridostigmine.
d. Naloxone.
e. Doxapram.
set 6_set 6.qxd 22-04-2013 19:22 Page 176

Single Best Answer MCQs in Anaesthesia

26 A 65-year-old male patient with severe COAD is scheduled for a


total knee replacement. Routine pre-operative investigations reveal
elevated blood levels of urea and creatinine. Which of the following
is the most appropriate for postoperative pain relief?

a. Regular paracetamol and diclofenac sodium.


b. Intravenous morphine infusion.
c. Femoral nerve block and infiltration of the wound with lignocaine
0.5%.
d. Intrathecal morphine and regular paracetamol.
e. Combined femoral and sciatic nerve block and regular paracetamol.

27 A 25-year-old male patient is admitted to the intensive care unit


176

following a thoracotomy for a stab injury of the chest. Six hours after
surgery, following extubation, he suddenly starts complaining of
shortness of breath. His core body temperature is 37°C, his pulse
is irregular with a rate of 150 bpm, his respiratory rate is 25/minute
and his blood pressure is 105/82mmHg. A 12-lead ECG shows
atrial fibrillation with a ventricular rate of 160 per minute. Arterial
blood gas shows a respiratory acidosis. On examination the patient
is pale and sweaty. His JVP is 12mm Hg. There are no clinical signs
of a pneumothorax or haemothorax. Which of the following would be
the most appropriate immediate management in this situation?

a. Echocardiography and cardiology opinion.


b. Commence treatment with anticoagulants.
c. Start patient on digoxin and then consider electrical cardioversion.
d. Electrical cardioversion and start patient on aspirin and heparin.
e. Electrical cardioversion and consider cardiothoracic/cardiology
opinion.

28 A 52-year-old man is scheduled for resection of an adrenal


pheochromocytoma. His current treatment includes phenoxy-
benzamine 40mg twice daily, commenced 10 days before. Over the
past 3 days his heart rate has varied between 110-120 bpm. His blood
pressure is 146/86mm Hg in the supine position and 110/68mm Hg
when erect. The next step in his management should be:
set 6_set 6.qxd 22-04-2013 19:22 Page 177

Set 6 questions

a. Proceed with surgery.


b. Increase the dose of phenoxybenzamine.
c. Start propranolol 30mg twice daily.
d. Stop phenoxybenzamine and commence phentolamine instead.
e. Start enalapril 5mg once daily.

29 A 35-year-old female patient with a history of postoperative nausea


and vomiting (PONV) is scheduled for a laser stapedectomy as a
day-case procedure. Which of the following in the anaesthetic
management is most likely to reduce PONV?
177
a. Avoiding neuromuscular blocking agents.
b. Premedication with anxiolytics.
c. Head-up tilt of 10-15°.
d. Total intravenous anaesthesia using propofol and remifentanil.
e. Induced hypotension.

30 A previously hypertensive patient is admitted to the intensive care


unit after an emergency laparotomy for an anastomotic leak following
an oesophagectomy. He was hypertensive and hypothermic in the
recovery room, but after active warming, he is now normothermic,
with a BP of 95/39mmHg, and heart rate of 110 bpm. His cardiac
index is 3.2L/m2, PCWP is 3mmHg, SVR is 550dynes/cm/sec-5,
and base excess is -7mmol/L. Over the last 2 hours, his total urine
output is 35ml. Which of the following would be the most
appropriate treatment to improve his renal function?

a. Administration of norepinephrine.
b. Administration of dobutamine.
c. Intravenous furosemide 40mg.
d. Intravenous mannitol 0.5g/kg.
e. Expansion circulating volume using a fluid challenge.
set 6_set 6.qxd 22-04-2013 19:22 Page 178

Single Best Answer MCQs in Anaesthesia

178
set 6 answers_set 6 answers.qxd 22-04-2013 19:21 Page 179

Set 6

Set 6 answers
answers

1 Answer: A. Mitral stenosis.


179
The normal area of the mitral valve orifice is about 4 to 6cm2. When the
mitral valve area is reduced below 2cm2, there is an impediment to the
flow of blood into the left ventricle, creating a pressure gradient across the
mitral valve. The first heart sound is unusually loud and may be palpable
(tapping apex beat) because of the increased force required to close the
mitral valve. Stenosis of the mitral valve causes turbulent diastolic flow
during diastole resulting in a mid-diastolic murmur, which is best heard at
the apical region. Tricuspid stenosis causes a low rumbling diastolic
murmur, which is best heard at the lower sternal border. Mitral
regurgitation causes a pansystolic murmur. Aortic stenosis causes an
ejection systolic murmur, which is best heard in the aortic area. Pulmonary
stenosis results in a systolic murmur which is best heard in the pulmonary
area. In mitral stenosis, enlargement of the left atrium produces
straightening of the left border of the heart on a plain chest X-ray.

Further reading
1. Valvular heart disease. In: Anesthesia and co-existing diseases, 4th
ed. Stoelting RK, Dierdorf S, Eds. Philadelphia, USA: Churchill
Livingstone, 2002; Chapter 2: 33-5.

2 Answer: C. Intravenous phenytoin 18mg/kg,


given over 30 minutes.

This child is suffering from status epilepticus; the priority is to stop the
seizures. Initial supportive management includes ensuring a patent airway,
set 6 answers_set 6 answers.qxd 22-04-2013 19:21 Page 180

Single Best Answer MCQs in Anaesthesia

administering 100% oxygen and assisting breathing. Rectal and buccal


medications should only be administered in the pre-hospital setting. Senior
help should also be sought promptly. If both benzodiazepines and
paraldehyde have been given with no effect, the next line of therapy after
intravenous access has been established is phenytoin. Although
paracetamol is used to reduce the temperature in febrile convulsions,
there is no evidence that antipyretics reduce the risk of subsequent febrile
convulsions. In the above clinical scenario it may not be possible to
administer medications orally.

Further reading
1. CG20 Epilepsy in adults and children: full guideline, appendix C
180 (corrected). NICE clinical guidelines, October 2004.
2. Chapman MG, Smith M, Hirschz NP. Status epilepticus, review
article. Anaesthesia 2001; 51: 648-59.
3. Appleton R, Choonara I, Martland T, et al. The treatment of convulsive
status epilepticus in children. Arch Dis Child 2000; 83: 415-9.
4. El-Radhi AS, Barry W. Do antipyretics prevent febrile convulsions?
Arch Dis Child 2003; 88: 641-2.
5. Young GM. Paediatric status epilepticus: treatment and medication.
(http://emedicine.medscape.com/article/804189-treatment).

3 Answer: D. Citalopram.

This patient is exhibiting signs of serotoninergic syndrome commonly


caused by overdose of selective serotonin reuptake blocking agents.
Overdose of edrophonium will cause cholinergic syndrome (confusion,
bradycardia, salivation, emesis and weakness). Overdose of
benzodiazepines will result in respiratory depression, hypotension,
hypothermia and hyporeflexia. Amitriptyline (tricyclic anti-depressant)
overdose causes metabolic acidosis, a wide QRS complex, prolonged PR
interval arrhythmias, convulsions and coma.

Further reading
1. http://www.toxbase.co.uk.
2. Mokhlesi B, Leiken JB, Murray P, et al. Adult toxicology in critical care:
Part II Specific poisoning. Chest 2003; 123: 897-922.
set 6 answers_set 6 answers.qxd 22-04-2013 19:21 Page 181

Set 6 answers

4 Answer: C. Intravenous infusion of 0.9% saline


solution.

This patient has developed hyponatraemia during the first week following
head injury. Clinical examination and biochemical tests suggest that he is
dehydrated, but the usual hypernatraemia seen with body water deficit is
not apparent.

Hyponatraemia is common after traumatic brain injury and it is important to


recognise the cause as the treatments differ significantly. The most
common causes are, firstly, the syndrome of inappropriate anti-diuretic
181
hormone secretion (SIADH) and, secondly, cerebral salt wasting
syndrome (CSWS). Both are characterised by hyponatraemia but are
differentiated by the intravascular volume status. This case has features
suggestive of hypovolaemia and so is likely to be CSWS.

SIADH is a normo- or hyper-volaemic state caused by excessive re-


absorption of free water. This causes haemodilution and is characterised
by a low plasma osmolality with a normal or high urine osmolality. It is also
important to assess the haemodynamic state; the key to the diagnosis of
SIADH is the absence of clinical signs of dehydration. Treatment of SIADH
is initially by restriction of water intake, followed by the administration of
drugs such as demeclocycline which inhibits the renal response to ADH.
It is also possible to use diuretics to increase fluid excretion, whilst
supplementing sodium by other means.

CSWS is often misdiagnosed as SIADH and treated incorrectly.


Biochemical criteria for CSWS include a low serum sodium with a high or
normal plasma and urine osmolality. The key distinguishing feature is
hypovolaemia. This should be assessed clinically; signs such as
hypotension, low central venous pressure and dry mucous membranes are
likely to be present. Examination of the fluid chart will often reveal a
persistently negative fluid balance. Blood tests will show features of
dehydration including a high urea and high haematocrit.

The pathophysiology of CSWS is still unclear, but is likely to involve the A-


type and B-type natriuretic peptides. Treatment is by replacement of
sodium and water, in most cases by the use of 0.9% saline solution.
set 6 answers_set 6 answers.qxd 22-04-2013 19:21 Page 182

Single Best Answer MCQs in Anaesthesia

Symptomatic hyponatraemia can be treated by the use of more


concentrated saline solutions (1.8% or 3%) alongside diuretics to avoid
circulatory overload.

Further reading
1. Bradshaw K, Smith M. Disorders of sodium balance after brain injury,
British Journal of Anaesthesia CEACCP 2008; 8: 129-33.

5 Answer: D. Mechanical VTE prophylaxis and


low-molecular-weight heparin continued for 28
days post-surgery.
182

According to NICE guidelines, the VTE risk of each patient should be


assessed individually. This patient is at significantly increased risk of VTE
as she has cancer, and is undergoing pelvic surgery with a period of
immobility in bed postoperatively. Mechanical VTE prophylaxis and low-
molecular-weight heparin should be prescribed and continued for 28 days
post-surgery.

Further reading
1. CG92 Venous thromboembolism - reducing the risk. National Institute
for Health and Clinical Excellence guidelines, 2010. (www.nice.org.uk).

6 Answer: C. Intravenous ketamine infusion.

This patient’s pain is generalised in the stump area and is therefore unlikely
to be neuroma-related pain. He did initially respond to opioid analgesics,
but seemed to have developed acute tolerance to them. As he has a
history of opioid addiction, acute tolerance to opioids is most likely.
Administering epidural or intrathecal opioids is effective in patients who
are responsive to opioid analgesia but cannot tolerate it due to side
effects. Gabapentin may be useful in this patient but it is not available
intravenously. Ketamine has been shown to reverse, at least partly reverse,
acute opioid tolerance in doses that are not large enough to provide a
direct antinociceptive effect. Therefore, intravenous infusion in the dose
range of 10 to 20mg/hour is likely to be the most effective.
set 6 answers_set 6 answers.qxd 22-04-2013 19:21 Page 183

Set 6 answers

Further reading
1. Yamauchi M, Asano M, Watanabe M, et al. Continuous low-dose
ketamine improves the analgesic effects of fentanyl patient-controlled
analgesia after cervical spine surgery. Anesth Analg 2008; 107: 1041-
4.

7 Answer: B. Cerebral oedema.

Cerebral oedema occurs in up to 1% of all paediatric diabetic


ketoacidosis (DKA). It is the most common cause of mortality in children

183
with DKA, accounting for 60-90% of all paediatric DKA deaths. Other
causes of mortality include hypokalaemia and hyperkalaemia with
associated arrhythmias, sepsis, aspiration pneumonia, acute pancreatitis,
intracranial venous thrombosis and rhabdomyolysis.

Further reading
1. Steel S, Tibby SM. Paediatric diabetic ketoacidosis. British Journal of
Anaesthesia CEACCP 2009; 9: 194-9.

8 Answer: D. Autoimmune hypothyroidism.

The clinical features are suggestive of hypothyroidism. Elevated TSH


levels together with low T3 and T4 levels suggest that she is experiencing
primary hypothyroidism. Primary hypothyroidism is due to intrinsic thyroid
gland failure. The most common cause is autoimmune hypothyroidism. It
is six times more common in females. Other causes of primary
hypothyroidism include irradiation, thyroid surgery and iodine deficiency or
excess.

Secondary hypothyroidism is due to inadequate levels of TSH with a


normal thyroid gland. This can be due to pituitary failure or pituitary
surgery. All levels of TSH, T3 and T4 will be low. Hypothalamic failure
results in tertiary hypothyroidism, due to low levels of TRH.

Further reading
1. Farling PA. Thyroid disease. British Journal of Anaesthesia 2000; 85:
15-28.
set 6 answers_set 6 answers.qxd 22-04-2013 19:21 Page 184

Single Best Answer MCQs in Anaesthesia

2. Howlett TA. Endocrine disease. In: Clinical medicine, 6th ed. Kumar P,
Clark M. Philadelphia, USA: Elsevier Saunders, 2005; Chapter 18:
1073-80.

9 Answer: E. Herpes simplex virus.

The results of CSF analysis suggest viral meningitis. Common viruses that
can cause meningitis include arbovirus, cytomegalovirus and the Herpes
simplex virus. This patient should be treated with I.V. acyclovir (10mg/kg
8- hourly). CSF findings in early bacterial meningitis and partially treated

184
bacterial meningitis may be similar to those found with viral meningitis.
Viral meningitis is also known as aseptic meningitis due to the inability to
isolate pathogens in CSF. In viral meningitis, the CSF biochemistry is likely
to reveal low protein and an elevated white cell count, predominantly
monocytes (lymphocytes). The CSF biochemistry in meningitis is shown in
Table 1.

Table 1. CSF biochemistry in meningitis.

Parameter Normal value Bacterial Viral


meningitis meningitis

WCC <5/mm3 >1000/mm3 <1000/mm3

Glucose 3.3-4.4mmol/L <2/3 plasma level >2/3 plasma level

Protein 0.2-0.4g/L 0.5-2g/L 0.4-0.8g/L

Further reading
1. Steiner I, Budka H, Chaudhuri A, et al. Viral encephalitis: a review of
diagnostic methods and guidelines for management. European Journal
of Neurology 2005; 12: 331-45.
2. Van de Beek D, de Gans J, Tunkel AR, et al. Community-acquired
bacterial meningitis in adults. N Engl J Med 2006; 354: 44-53.
set 6 answers_set 6 answers.qxd 22-04-2013 19:21 Page 185

Set 6 answers

10 Answer: D. Obliteration of the third ventricle.

All of the above findings on the CT scan are associated with a poor
prognosis. Obliteration of the third ventricle and midline shift are, however,
the strongest predictors of mortality at 14 days.

Further reading
1. The MRC CRASH Trial Collaborators. Predicting outcome after
traumatic brain injury: practical prognostic models based on a large
cohort of international patients. BMJ 2008; 336: 425-9.

11 Answer: D. Gabapentin.
185

This patient is suffering from post-amputation limb pain. Following


amputation, the incidence of chronic pain is very high. There are three
types of pain problems which can occur after amputation. These are
stump pain, neuroma pain or phantom limb pain. Stump pain usually
occurs as a result of infection, scar tissue or bony stump irritation due to
inadequate padding by muscle. Neuroma pain is due to abnormal
outgrowth of the nerve from the cut end. It is generally localised pain and
elicited by touch or pressure over the neuroma. Phantom limb pain is pain
experienced in the missing part of the limb. One of the proposed theories
is that when a limb is amputated many severed nerve endings outgrow
abnormally and/or become inflamed and send anomalous signals to the
brain. These signals are thought to be interpreted by the brain as pain. At
times, the patient may feel as if they are gesturing, feel itches, twitch, or
even feel as if they are trying to pick things up.

This particular patient has phantom limb pain and therefore gabapentin
would be the most appropriate initial therapy. If this fails, intervention
therapy or, more recently recommended, image-guided therapy could be
considered. Image-guided therapy is a novel treatment for phantom limb
pain. Through the use of artificial visual feedback (mirror box) it becomes
possible for the patient to ‘move’ the phantom limb, and to unclench it from
potentially painful positions. Repeated training in some subjects has led to
long-term improvement.
set 6 answers_set 6 answers.qxd 22-04-2013 19:21 Page 186

Single Best Answer MCQs in Anaesthesia

Further reading
1. Halbert J, Crotty M, Cameron ID. Evidence for the optimal
management of acute and chronic phantom pain: a systematic review.
Clin J Pain 2002; 18: 84-92.

12 Answer: A. Axillary petechiae.

The diagnosis of fat embolism is usually made on the basis of clinical


findings. The major criteria are based on the classic triad of respiratory
insufficiency, neurological impairment and a petechial rash. Although a
186
petechial rash is present in only 20-50% of cases, it is considered
pathognomonic of fat embolism. To establish the diagnosis of fat embolism
syndrome, at least one major and four minor criteria must be present.

Major criteria include:

w Axillary or sub-conjunctival petechiae.


w Hypoxaemia PaO2 <60mm Hg.
w Central nervous system depression disproportionate to the
hypoxaemia.
w Pulmonary oedema.

Minor criteria include:

w Tachycardia.
w Pyrexia.
w The presence of fat in the urine.
w The presence of retinal emboli.
w Increased ESR.

Further reading
1. Gupta A, Reilly CS. Fat embolism. British Journal of Anaesthesia
CEACCP 2007; 7: 148-51.
set 6 answers_set 6 answers.qxd 22-04-2013 19:21 Page 187

Set 6 answers

13 Answer: E. Baclofen.

The symptoms described in the postoperative period are those due to


acute withdrawal of baclofen. The majority of patients with cerebral palsy
are likely to be taking a combination of anti-epileptics, antidepressants,
baclofen and analgesics. As this patient developed postoperative nausea
and vomiting she may have omitted her regular oral baclofen during the
postoperative period as well. Sudden cessation of baclofen may result in
acute withdrawal symptoms such as disorientation, painful muscle
spasms, dystonia, seizures, bradycardia and hypotension.

Baclofen is a GABAA receptor agonist, which acts as an inhibitory 187


neurotransmitter in the central nervous system. It can be given orally or
intrathecally through a subcutaneously implanted continuous infusion
device. Ondansetron has very mild side effects on I.V. administration, such
as headache, a sensation of warmth, occasional visual disturbances and
transient dizziness.

Further reading
1. Prosser DP, Sharma N. Cerebral palsy and anaesthesia. British
Journal of Anaesthesia CEACCP 2001; 10: 72-6.

14 Answer: C. Activated charcoal.

Activated charcoal is used in the management of many orally ingested


toxins due to its adsorptive capacity. It is effective if administered within the
first hour of ingestion. A recent Cochrane review concluded that activated
charcoal is more effective than gastric lavage or ipecac-induced emesis in
preventing absorption of paracetamol. N-acetyl cysteine and methionine
are used in the treatment of paracetamol overdose but do not reduce the
absorption of paracetamol.

Further reading
1. Ward W, Sair M. Oral poisoning: an update. British Journal of
Anaesthesia CEACCP 2010; 10: 6-11.
set 6 answers_set 6 answers.qxd 22-04-2013 19:21 Page 188

Single Best Answer MCQs in Anaesthesia

15 Answer: B. Oxygen and intravenous Hartmann’s


solution.

This man may have sickle cell disease (SCD), an inherited


haemoglobinopathy resulting from a mutation in chromosome 11 which
leads to the substitution of valine for glutamate at position 6 of the beta Hb
chain, producing HbS. HbS precipitates in the deoxygenated form and
may lead to sickling of the red blood cell. Sickling of red blood cells is
exacerbated by stasis, cold, dehydration and sepsis. Acute ‘crises’ are
most often vaso-occlusive, presenting with an acute abdomen, pulmonary
infarction, bone infarction, stroke and priapism. Aplastic crises lead to
188 marrow shutdown usually caused by parvovirus B19, while sequestration
crises are seen usually in children with massive pooling of red cells in the
spleen. Patients are usually anaemic (Hb 6-9g/dL) and develop
progressive organ damage including asplenism, renal impairment,
cardiomegaly, pulmonary hypertension, retinal damage and
cerebrovascular disease. Heterozygotes (HbAS) produce normal Hb and
about 30-40% HbS. Diagnosis is by electrophoresis which distinguishes
type and quantity of different haemoglobins. The Sickledex test is a
screening test which can rapidly detect the presence of HbS if greater
than 10%, and is positive in both disease and trait. The blood film in SCD
will usually show anaemia with reticulocytosis, target and sickle cells,
whereas it is often normal in trait.

In this scenario, the acute abdomen may represent a crisis, therefore


immediate measures should include rehydration and oxygen as well as
analgesia. The need for surgery may then be reassessed in light of the
response and Sickledex results. If surgery is required, it may be advised
to transfuse to an Hb of >10g/dL following advice from a haematologist.

Further reading
1. Haemoglobinopathy and sickle cell disease. British Journal of
Anaesthesia CEACCP 2010; 10: 24-7.
set 6 answers_set 6 answers.qxd 22-04-2013 19:21 Page 189

Set 6 answers

16 Answer: B. Penile block and regular


paracetamol and ibuprofen.

Postoperative pain after circumcision is usually well controlled using


regular paracetamol and ibuprofen. It is a simple regime, which can safely
be administered at home. Caudal block is effective in providing analgesia
after circumcision and is commonly used. However, 10ml of 0.5%
bupivacaine (50mg) in this child will exceed the maximum dose and is likely
to cause urinary retention and leg weakness. The dose of ketamine for
caudal use is in the range of 0.5 to 1mg/kg and the higher dose is likely to
cause unpleasant CNS side effects. Regular Oramorph is rarely needed
for circumcision and has side effects such as nausea, vomiting and 189
sedation. Penile block with regular paracetamol and ibuprofen would be
safe and effective pain relief in this patient.

Further reading
1. Joyce BA, Keck JF, Gerkensmeyer J. Evaluation of pain management
interventions for neonatal circumcision pain. Journal of Pediatric
Health Care 2001; 15: 105-14.

17 Answer: D. Insertion of an automatic


implantable cardioverter defibrillator (AICD).

The aim of treatment is to prevent sudden death from fatal arrhythmias


such as Torsades de Pointes. Beta-blockers are the first line in the
treatment of long QT syndrome (LQTS), but they may be ineffective in
about 25% of patients. Anti-bradycardia pacing is indicated in patients
with bradycardia and pauses. An AICD is the treatment of choice when
the initial presentation is a cardiac arrest, in symptomatic patients despite
beta-blockers, and in patients with documented arrhythmia despite beta-
blockers.

Long QT syndrome complicated by Torsades de Pointes can be treated


with intravenous magnesium sulphate. Amiodarone is not effective in
preventing arrhythmias associated with long QT syndrome.
set 6 answers_set 6 answers.qxd 22-04-2013 19:21 Page 190

Single Best Answer MCQs in Anaesthesia

Further reading
1. Al-Refai A, Gunka V, Douglas J. Spinal anaesthesia for Caesarean
section in a parturient with long QT syndrome. Canadian Journal of
Anaesthesia 2004; 51: 993-6.
2. Drake E, Preston R, Douglas J. Brief review: anaesthetic
implications of long QT syndrome in pregnancy. Canadian Journal of
Anaesthesia 2007; 54: 561-72.

18 Answer: D. Transfusion-related acute lung


injury (TRALI)
190
Since the clinical deterioration occurred following blood transfusion, it is
most likely to be related to the blood transfusion. The possibility of
TRALI should be considered in any patient developing hypoxaemia and
pulmonary oedema within a few hours of transfusion of any blood
product or plasma derivative containing plasma.

TRALI is a form of acute respiratory distress syndrome due to


transfusion of blood products containing plasma. The onset is usually
within the first 6 hours, though it can occur up to 24 hours after
transfusion. The mechanism of TRALI involves activation of recipient
leucocytes by donor anti-leucocyte antibodies.

Cardiogenic shock can occur secondary to a myocardial event in a


patient with significant cardiovascular disease. Similarly, volume
overload may occur in patients with pre-existing cardiorespiratory
disease.

Further reading
1. Teague G, Hughes A, Gaylard D. Transfusion-related acute lung
injury. Anesthesia Intensive Care 2005; 33: 124-7.
2. Rajan GR. Severe transfusion-related acute lung injury in the
intensive care unit secondary to transfusion of fresh frozen plasma.
Anaesthesia Intensive Care 2005; 33: 400-2.
set 6 answers_set 6 answers.qxd 22-04-2013 19:21 Page 191

Set 6 answers

19 Answer: A. Septicaemia.

Oesophagectomy is associated with multiple potential complications,


including oesophageal anastomotic leak, chest infection, deep vein
thrombosis, myocardial infarction, gastric necrosis, and prolonged ileus.
The postoperative mortality rate associated with oesophagectomy ranges
from 5% to 13%. The most common causes of morbidity and mortality are
cardiopulmonary complications. The rise in temperature, hypotension,
tachycardia, and a low CVP indicate sepsis as the most likely cause of his
collapse. In severe dehydration or haemorrhage or myocardial infarction,
the patient’s temperature is likely to be normal. In cardiac tamponade, the
CVP will be high. 191

Further reading
1. Nozoe T, Kimura Y, Ishida M, et al. Correlation of pre-operative
nutritional condition with postoperative complications in surgical
treatment for oesophageal carcinoma. Eur J Surg Oncol 2002; 28:
396-400.

20 Answer: E. ECG.

The history is suggestive of myotonia dystrophica. It is a multisystem


disease inherited as an autosomal dominant trait. Patients usually present
at the age of 15-35 years. Increased muscle tone on exercise, respiratory
muscle weakness, cataracts, testicular atrophy, frontal balding, obstructive
sleep apnoea, and cardiomyopathy are associated features. First-degree
heart block is a common finding on the ECG. Sudden death can occur
due to complete heart block. Up to 20% of patients may have evidence of
mitral valve prolapse on echocardiography.

Although ultrasound of the gall bladder is a useful investigation to evaluate


the presence of gall stones, it is more relevant to surgical management.
Electromyography is not helpful in this situation but may be useful in the
diagnosis of myasthenia gravis.

Anaesthetic implications in these patients include poor cardiorespiratory


reserve, delayed gastric emptying, and increased sensitivity to I.V.
set 6 answers_set 6 answers.qxd 02-05-2013 21:55 Page 192

Single Best Answer MCQs in Anaesthesia

anaesthetics and non-depolarising agents. Suxamethonium may cause


prolonged muscle contraction.

Further reading
1. Russell SH, Hirsch P. Anaesthesia and myotonia, review article. British
Journal of Anaesthesia 1994; 72: 210-6
2. Myotonic dystrophy In: Anesthesia and co-existing disease, 4th ed.
Stoelting RK, Dierdorf SF, Eds. Philadelphia, USA: Churchill
Livingstone, 2002; 519-20.

21 Answer: A. Local infiltration and regular


192 paracetamol.
The surgical procedure of pyloromyotomy involves a small sub-costal
incision and pain following this can be managed by local infiltration and
regular paracetamol. NSAIDs should be avoided below the age of 6
months due to their effect on pulmonary circulation. Opioids are generally
avoided below the age of 6 months due to their side effects. There is a risk
of postoperative apnoea in neonates and the biochemical changes
associated with pyloric stenosis can also predispose to opioid-induced
postoperative respiratory depression.

Further reading
1. Fell D, Chelliah S. Infantile pyloric stenosis. British Journal of
Anaesthesia CEPD Reviews 2001; 1: 85-8.

22 Answer: A. Abdominal aortic surgery.

The risk of overall postoperative respiratory complications is about 25%


with open abdominal aortic surgery. The incidence of postoperative
pneumonia is higher with abdominal aortic surgery compared to thoracic
surgery (odds ratio 4.3 versus 3.9).

Endovascular repair is associated with a much lower risk of postoperative


pneumonia as compared to an open procedure.
set 6 answers_set 6 answers.qxd 02-05-2013 21:55 Page 193

Set 6 answers

Further reading
1. Moppett IK. Respiratory risk. In: Consent, benefit, and risk in
anaesthetic practice. Hardmann JG, Moppett IK, Aitkenhead AR, Eds.
Oxford: Oxford University Press, 2009; Chapter 12: 173-87.

23 Answer: C. Plasma insulin level.

The possible diagnosis in this patient is insulinoma; this is an insulin-


secreting tumour arising from the islet cells of the pancreas. It may secrete
insulin in short bursts, causing wide fluctuations in the blood levels. About
90% of insulinomas are benign, and 10% are malignant. Roughly 10% of
patients have multiple insulinomas. 193

The diagnosis is based mainly on the presence of inappropriately high


levels of insulin (>10 micro units), a low plasma glucose (<2.5mmol/L),
and plasma c-peptide levels >2.5ng during overnight or supervised fasting
for 72 hours.

Imaging studies such as an ultrasound scan, a CT scan and an MRI scan


are only indicated once the biochemical tests confirm the diagnosis,
because the majority of tumours are smaller than 2cm, which are difficult
to detect using imaging studies.

Further reading
1. Ali AZ, Radebold K. Insulinoma. (http://emedicine.medscape.com/
article/283039-diagnosis).

24 Answer: A. Enoxaparin 1.5mg/kg administered


subcutaneously.
The most likely diagnosis is pulmonary embolism (PE). The patient is
relatively stable but demonstrates signs of right heart strain on the ECG
and echocardiogram. Current evidence suggests that the risks of
thrombolysis (such as GI bleed, haemorrhagic stroke) outweigh the
benefits unless there is significant haemodynamic instability or cardiac
arrest. It is therefore not indicated in this case.
set 6 answers_set 6 answers.qxd 02-05-2013 21:55 Page 194

Single Best Answer MCQs in Anaesthesia

Initial treatment should be with a ‘treatment-dose’ low-molecular-weight


heparin, and this should be continued while long-term anticoagulation is
established. Warfarin is ineffective for 2-3 days following the start of
therapy and so is not the correct initial treatment.

The most likely source of embolus is a vein in the leg that has recently
been treated for the fracture. If there are clinical signs of DVT then a
Doppler ultrasound should be arranged, but treatment should be started in
the meantime. If a DVT is not found then a CT pulmonary angiogram will
be necessary to confirm the diagnosis, but again this should not delay
treatment.

Further reading
194
1. Van Beek EJR, Elliot CA, Kiely DG. Diagnosis and initial treatment of
patients with suspected pulmonary thromboembolism. British Journal
of Anaesthesia CEACCP 2009; 9: 119-24.

25 Answer: B. Edrophonium.

The likely cause for muscle weakness and hypoventilation in this patient is
either a cholinergic crisis or a myasthenic crisis. Both can result in muscle
weakness. A cholinergic crisis occurs as a result of excessive
acetylcholine, due to an excessive dose of anticholinesterase. In a
cholinergic crisis, the pupils are constricted, whereas in a myasthenic
crisis they are dilated. In a myasthenic crisis a small dose of edrophonium
improves muscle strength; in a cholinergic crisis it does not improve
muscle weakness. A myasthenic crisis should be treated with
pyridostigmine. Any further doses of neostigmine should be used
cautiously to avoid a cholinergic crisis. Doxapram is a respiratory stimulant
and naloxone reverses opioid-induced respiratory depression.

Further reading
1. Myasthenic syndrome. In: Anesthesia and co-existing diseases, 4th
ed. Stoelting RK, Dierdorf S, Eds. Philadelphia, USA: Churchill
Livingstone, 2002; Chapter 26: 527-8.
2. Abel M. Myasthenia gravis. In: Clinical cases in anaesthesia, 3rd ed.
Reed AP, Yudkowitz FS, Eds. Philadelphia, USA: Elsevier Churchill
Livingstone, 2005; Case 27: 137-42.
set 6 answers_set 6 answers.qxd 02-05-2013 21:55 Page 195

Set 6 answers

26 Answer: E. Combined femoral and sciatic nerve


block and regular paracetamol.

A combined femoral and sciatic nerve block is effective in providing


adequate postoperative analgesia for up to 24 hours in the postoperative
period. Both systemic and intrathecal opioids can cause respiratory
depression and this risk is increased in a patient with severe COAD.
Although NSAIDs are generally recommended for postoperative
analgesia, they would be inappropriate in a patient with deranged renal
function.

Further reading 195


1. Fischer HBJ, Simanski CJP, Sharp C. A procedure-specific
systematic review and consensus recommendations for post-operative
analgesia following total knee arthroplasty. Anaesthesia 2008; 63:
1105-23.

27 Answer: E. Electrical cardioversion and consider


cardiothoracic/cardiology opinion.

Unless contraindicated, a rhythm-control strategy should be the initial


management option for the treatment of postoperative atrial fibrillation (AF)
following cardiothoracic surgery. Any underlying electrolyte imbalance
should be corrected and prophylaxis with anti-thrombotic therapy (in
consultation with cardiology) should be considered.

In general, the evidence suggests that combined therapeutic


anticoagulation with antiplatelet therapy does not reduce the incidence of
stroke or thrombo-embolism when compared to therapeutic
anticoagulation alone, and it may increase the incidence of bleeding.
Preloading with anti-arrhythmic drugs prior to electrical cardioversion does
not appear to have any long-term efficacy in maintaining sinus rhythm.

Further reading
1. NICE guidelines for the management of atrial fibrillation, June 2006.
set 6 answers_set 6 answers.qxd 02-05-2013 21:55 Page 196

Single Best Answer MCQs in Anaesthesia

28 Answer: C. Start propranolol 30mg twice daily.

Adequate pre-operative preparation is essential prior to surgery in order to


reduce mortality and morbidity. Alpha-blockade with oral
phenoxybenzamine is the treatment of choice for pheochromocytoma. It
should be started at 10mg per day and gradually increased until postural
hypotension develops. As the alpha-blockade is established, the
intravascular volume expands and tachycardia may develop which should
be treated with adequate hydration. Beta-blockade can be used to treat
associated tachycardia. Beta-blockers should not be started until
adequate alpha-blockade is established, as an unopposed alpha-
196 adrenergic effect may precipitate a hypertensive crisis and cardiac failure.

This patient has adequate alpha-blockade as indicated by his postural


hypotension. His tachycardia should be treated with propranolol. Both
drugs should be continued. Enalapril is an angiotensin-converting enzyme
inhibitor and is not usually used in the management of pheochromocytoma.

Further reading
1. Prys-Roberts C. Phaeochromocytoma - recent progress in its
management. British Journal of Anaesthesia 2000; 85: 44-57.
2. Pace N, Buttigieg M. Phaeochromocytoma. British Journal of
Anaesthesia CEPD review 2003; 3: 20-3.
3. Malhotra V, Garland TA. Pheochromocytoma. In: Decision making in
anesthesiology - an algorithmic approach, 3rd ed. Bready LL, Mullins
RM, Noorily SH, Smith RB, Eds. Missouri, USA: Mosby, 180-1.

29 Answer: D. Total intravenous anaesthesia using


propofol and remifentanil.

Middle ear surgery is associated with a high incidence of PONV. The


reported incidence is as high as 80% in adults without anti-emetic
prophylaxis. The use of total intravenous anaesthesia using propofol is a
popular technique for middle ear surgery, as a reduced incidence of
PONV, reduced bleeding and better operating conditions have been
observed.
set 6 answers_set 6 answers.qxd 02-05-2013 21:55 Page 197

Set 6 answers

There is no contraindication to the use of neuromuscular blocking agents


at induction of anaesthesia, though avoiding further doses of
neuromuscular blocking agents facilitates monitoring of the facial nerve
using a nerve stimulator. This patient may be very anxious due to the
previous experience of nausea and vomiting, so premedication with
benzodiazepines may be helpful in reducing the anxiety. Induced
hypotension and a slight head-up tilt reduces bleeding at the surgical site.

Further reading
1. Herlich A. Tympanomastoidectomy. In: Clinical cases in anaesthesia,
3rd ed. Reed AP, Yudkowitz FS, Eds. Philadelphia, USA: Elsevier
Churchill Livingstone, 2005; Case 45: 243-5.
197

30 Answer: E. Expansion circulating volume using a


fluid challenge.

This patient has low systemic vascular resistance (SVR) and hypovolaemia
as indicated by the low PCWP. Clinical signs also indicate that the patient
is volume depleted probably due to sequestration of the fluid in the
peritoneal cavity and increased capillary permeability due to sepsis.
Mannitol will increase urine output but will worsen the hypovolaemia.
Furosemide can cause a diuresis but will not improve renal function.
Administration of dobutamine would be appropriate if the urine output
does not improve despite restoration of the circulating volume.

Further reading
1. Carcillo JA, Tasker RC. Fluid resuscitation of hypovolemic shock:
acute medicine’s great triumph for children. Intensive Care Med 2006;
32: 958-61.
2. Sturm JA, Wisner DH. Fluid resuscitation of hypovolaemia. Intensive
Care Medicine 1985; 11: 227-30.
set 6 answers_set 6 answers.qxd 22-04-2013 19:21 Page 198

Single Best Answer MCQs in Anaesthesia

198
SBA cover.qxd 02/07/2010 15:30 Page 1

Single Best Answer MCQs in Anaesthesia


This book comprises six sets of single best answer practice papers. Each set

contains 30 single best answer questions on clinical anaesthesia, pain and

Single Best Answer MCQs in


intensive care. The scenarios are based on common peri-operative problems

encountered during anaesthetic practice and intensive care medicine. The

best possible answer to a given clinical scenario is substantiated by detailed

explanation drawn from recent review articles and textbooks in clinical

ANAESTHESIA
anaesthesia. These questions enable the candidates to assess their knowledge

and problem-solving ability.

This book is an ideal companion for candidates sitting postgraduate

examinations in anaesthesia and intensive care medicine. It will also be a

valuable educational resource for all trainees and practising anaesthetists.

Volume I Clinical Anaesthesia

ISBN 978-1-903378-75-5

9 781903 378755
tf m Cyprian Mendonca, Mahesh Chaudhari, Josephine James

You might also like

pFad - Phonifier reborn

Pfad - The Proxy pFad of © 2024 Garber Painting. All rights reserved.

Note: This service is not intended for secure transactions such as banking, social media, email, or purchasing. Use at your own risk. We assume no liability whatsoever for broken pages.


Alternative Proxies:

Alternative Proxy

pFad Proxy

pFad v3 Proxy

pFad v4 Proxy